巨大数探索スレッド過去ログ


巨大数探索スレッド11.75 [転載禁止]©2ch.net

1 :
132人目の素数さん
2015/11/23(月) 02:05:24.86 ID:xCIHApRS
大きな実数を探索するスレッドです。 

巨大数探索スレッド10
 http://wc2014.2ch.net/test/read.cgi/math/1384444271/
巨大数探索スレッド11b
 http://wc2014.2ch.net/test/read.cgi/math/1447237446/
巨大数研究室
 http://www.geocities.co.jp/Technopolis/9946/
巨大数 (Wikipedia)
 http://ja.wikipedia.org/wiki/%E5%B7%A8%E5%A4%A7%E6%95%B0
ふぃっしゅっしゅ氏の巨大数論PDF
 http://gyafun.jp/ln/
たろう氏のまとめ
 http://gyafun.jp/ln/archive/7-571.txt
Dmytro Taranovsky の順序数表記
 http://web.mit.edu/dmytro/www/other/OrdinalNotation.htm
mixi 巨大数コミュ (要 mixi アカウント)
 http://mixi.jp/view_community.pl?id=2771859
巨大数研究Wiki
 http://ja.googology.wikia.com/wiki/
2 :
132人目の素数さん
2015/11/23(月) 02:17:01.15 ID:xCIHApRS
数列
(0)=1
(0 0)=2
(1 0)=ω
(1 0 0)=ω+1
(1 0 1 0)=ω2
(0 1 0)=ω^2
(0 1 0 1 0 0)=ω^2+ω+1
(0 0 1 0)=ω^3
(1 1 0)=ω^ω
(2 1 0 0)=ω^(ω+1)
(2 0 1 0)=ω^ω^2
極限はε_0
3 :
132人目の素数さん
2015/11/23(月) 06:45:23.24 ID:xCIHApRS
P a={a}
A a b=a∩b
N a=¬a
U a b=a∪b
I a b={c|a∈c∧c⊆b}
E={}
これでなんかできないか
4 :
2015/11/23(月) 21:11:23.14 ID:j7D50RId
保守 そして乙
3行のバシク行列の解析結果は間違ってると思う。wikiaのやつ
v(ω,0,0)がI_ωと同等(等しいとは言ってない)というのが勘違いかなと
5 :
132人目の素数さん
2015/11/24(火) 20:20:36.91 ID:HIBjlwsf
前スレなぜ消えたん?
6 :
2015/11/24(火) 20:21:34.30 ID:OiYH/djC
運営の陰謀
7 :
2015/11/24(火) 21:15:48.40 ID:OiYH/djC
再投稿

巨大数を返すプログラムが停止することを証明するには
例えばどのようなテクニックがあるでしょうか。
8 :
2015/11/24(火) 22:07:46.33 ID:hQnIig8o
無限ループに突入するパターンなら簡単に証明できる。
カオスな振るまいをするやつはなんか頑張る。
任意のは不可能でも特定のプログラムが停止するかどうかは計算可能だから

・・・だったよな(自信ない)
9 :
2015/11/24(火) 22:37:15.65 ID:hQnIig8o
(0,0,0,0)(1,1,1,1)までの計算結果はもうあれで確定でいいけれど、これより先は怪しいので、検算のためにwikiaの編集は少し待っていただきたいです。

今年中にすくなくとも4行は「確実丁寧に」解明したい
Cでもかなわなくなったら後は海外勢にまかせる。でも英語版にはバシク行列の記事は無かったな。

(・・・算術的には1行でATR_0、2行でΠ^1_1-CA_0・・・)
10 :
2015/11/25(水) 02:08:08.52 ID:OGFQOmH0
バシク行列は計算終了の証明がされていないし、直感的にも終了することが自明でもないので、
そこをうまく説明できないと英語版の作成は難しい。
11 :
2015/11/25(水) 02:10:40.30 ID:OGFQOmH0
手っ取り早いのは、バシク行列の計算についてわかっている人が、
Googology wiki にアカウント作って、ブログで説明を書くこと。
そうすれば、興味を持った人が解析してくれる。
12 :
2015/11/25(水) 02:13:14.40 ID:OGFQOmH0
ちなみに、ペア数列数までは英語版の記事ができている。
13 :
132人目の素数さん
2015/11/25(水) 17:32:55.50 ID:TkLRuCun
チューリングボンベはチューリングマシンより強いって聞いたことがあるような
なんか恒等式を解けるとか
14 :
2015/11/25(水) 18:57:45.55 ID:ZS7/TrMk
強いっていうか速度が上がる可能性がある、程度のことだろ。
本質的に解けなかった問題が解けるようになるわけじゃないだろ。
15 :
2015/11/25(水) 20:09:17.62 ID:1xveQ4sJ
ATR_0じゃなくてACA_0だった。毎度毎度微妙なとこ間違えてすまん。
この流れだと3行はΠ^1_2-CA_0と予想されるけど、どうなんだろ

0列については推移律が成り立たないという話があるけれど、これについては関数を適用させて(1を引いて)消すというルールが特別に設けられているから問題ないです。
標準形のようなものを作って、それが計算終了することを証明。つぎに任意の行列が標準形の形に直すことができることを証明。こういう流れかね
16 :
2015/11/25(水) 21:15:37.08 ID:OGFQOmH0
バシク行列の計算が終了する = バシク行列は整列集合である
 = バシク行列は順序数と対応可能である

ということで、もしこの照明ができれば、バシク行列をそれなりに大きな可算順序数を表記
できる順序数の記法とすることができるので、けっこう面白いはず。
17 :
2015/11/26(木) 23:48:16.70 ID:MsvMxaCP
3行のおさらい

(0,0,0)(1,1,1)(2,2,1)(3,2,0)=C(C(Ω+C(Ω*2,0),0),0)=v(v(1,0,0))=ψ(ψ_I(0))
(0,0,0)(1,1,1)(2,2,1)(3,2,0)(2,2,1)=C(C(Ω+C(Ω*2,0)+1,0),0),0)=v(v(1,0,0)*2)=ψ(ψ_I(ω))

v関数 v(1,0,0)より大きい引数を持ってくることで、構成可能な範囲で任意の2変数vを引数とすることができる。その2変数vの引数も同様
v(v(v(1,0,0),0))でもv(v(1,0,0))でもどちらでもいいです。
これ以上については検討中
18 :
2015/11/27(金) 00:01:49.52 ID:9fdghPrT
v関数とバシク行列の関係についてはあれであってると思うけれど、一緒に消されちゃったな
そろそろv関数の記事ができてもいいだろう。
19 :
2015/11/27(金) 19:24:35.57 ID:9fdghPrT
探索スレッド10の定義に則って、

v(v(1,0,0)+v(v(1,0,0),v(1,0,0)))>v(v(1,0,0)+v(v(1,0,0),v(v(1,0,0),v(1,0,0)))>...

こうして無限下降列ができて計算が終了しません。

変数の少ない方から入れ子構造をつくっていくと、

v(v(v(1,0,0),v(v(1,0,0),v(v(1,0,0,),...))))

これ以上計算できません。
前スレの定義はもう少し洗練しないと使えないな。感覚でどうしたいかは何となくわかるけど
20 :
2015/11/28(土) 00:34:08.55 ID:2Z9mTxSf
昔計算結果をメモした紙を捨ててしまったので再計算・・・

v(v(v(1,0,0)+v(1,0,0),0))=ψ(ψ_I(1))
v(v(v(1,0,1),0))=ψ(ψ_I(ω))
v(v(v(1,0,v(1,0,0)),0))=ψ(ψ_I(I))
v(v(v(1,0,v(1,1,0)),0))=ψ(ψ_I(ε_{I+1}))
...

v(n,0,0)がI_nとなって思ったほどの強さに達していない
まずいことにそもそも昔の自分の計算が間違えていたようだ
v関数の定義はまだまだ遠い。私はもう諦めた

せめてもの対応表

v(n,0,0) I_n C(Ω*(n+1))
v(n,0,0,0) χ(1,n-1) C(Ω^2*n)
v(n,0,0,0,0) χ(2,n-1) C(Ω^3*n)
21 :
132人目の素数さん
2015/11/30(月) 00:39:12.18 ID:WsfxmPRr
初心に返って
この世に存在する0より大きく1より小さいすべての数をかけた数の逆数
22 :
2015/11/30(月) 11:05:31.03 ID:SStpBv11
なんで初心に返るんだ
23 :
2015/11/30(月) 11:06:48.00 ID:yLx4cCSm
数学初心者に退化してる
24 :
132人目の素数さん
2015/11/30(月) 18:42:51.89 ID:x1oQ/D0B
(n↑2)/n!
25 :
2015/11/30(月) 22:56:51.41 ID:BQHSQ7WV
あれは惰性で表面滑ってるだけなのを諦めないという言葉で美化していたんだろう

(0,0,0,0)(1,1,1,1)(2,2,1,0)=C(C(ε_{Ω+1}+1,0),0)
(0,0,0,0)(1,1,1,1)(2,2,1,0)(3,3,2,1)=C(C(ε_{Ω+1}*2,0),0)
(0,0,0,0)(1,1,1,1)(2,2,1,1)=C(C(ε_{Ω+2},0),0)=C(C(C(Ω_2,Ω),0),0),C(C(C(Ω_2,Ω),0),0))
(0,0,0,0)(1,1,1,1)(2,2,1,1)(3,2,1,1)=C(C(C(Ω_2,Ω),0)+C(C(C(Ω_2,Ω),0),0),0),0)
(0,0,0,0)(1,1,1,1)(2,2,2,0)=C(C(Ω_2,Ω)^ω,0)
(0,0,0,0)(1,1,1,1)(2,2,2,1)=C(C(C(C(Ω_2,C(Ω_2,Ω)),0),0),0)

3行の解析結果とか、誤りがあれば指摘ほしいです。定期的にあれで本当にいいのか不安になる
26 :
2015/12/01(火) 02:08:36.06 ID:79GFDpvA
結局、ここでは誰も分からないんだから、検証して欲しいなら GWiki に突撃しないと意味ないって
27 :
2015/12/01(火) 02:09:30.12 ID:79GFDpvA
もちろん英語版の方ね
28 :
2015/12/01(火) 02:13:37.99 ID:79GFDpvA
GWiki はプログラム読める人多いんで、計算方法はプログラム読んでね、で十分で、
あとはどのように順序数を計算したかを説明するだけ
29 :
132人目の素数さん
2015/12/01(火) 09:54:29.55 ID:Hxid+zsa
チェーンまでは理解出来たがアッカーマン関数から高卒じゃキツくなってくるわ
ぱっと見アッカーマン以上の式が出てきてるようだけど、

タワー表記

チェーン表記

アッカーマン


て感じに増大具合並べていった時のアッカーマン以降を教えて下さい。
30 :
2015/12/01(火) 21:18:07.68 ID:G+wtwvIa
wakiaの関数の一覧にまとめられとるだ
突撃の件は、勉強や解析が一段落してから考えるです。
不完全性定理の無矛盾への拡張とか、命題と証明のゲーデル数の間のみに成り立つ算術的関係の存在とか、
Π^1_n-CA_0とバシク行列の関係とか。課題がいろいろ

任意の順序数の後者ではないとか、なんらか特定の不動点とかいう条件は一階の算術で記述できます。
算術的内包公理で集合を作り、その最小値をとれば最初の極限を得られます。
具体的にどのような極限を得ることができるかは公理系によってしぼられます。
そんなこんなで1行の強さはACA_0であり、その証明論的順序数はε_0
31 :
132人目の素数さん
2015/12/01(火) 21:49:52.04 ID:Semh/LDE
括弧の中はFGHの順序数
↑弱い
足し算(0) < 掛け算(1) < べき関数(1) < 指数関数(2)
タワー表記(ω)≒アッカーマン関数(ω)
チェーン表記(ω^2)

1行のバシク行列(ε_0=ω↑↑ω)
32 :
132人目の素数さん
2015/12/01(火) 22:01:47.69 ID:Semh/LDE
TREE関数 θ(Ω^ω・ω)

ペア数列システム θ(Ω_ω)
↓強い

http://ja.googology.wikia.com/wiki/%E9%96%A2%E6%95%B0%E3%81%AE%E4%B8%80%E8%A6%A7
33 :
132人目の素数さん
2015/12/01(火) 22:07:19.72 ID:Semh/LDE
>>22
初心に返ったのはちょっと別のことをしていてブランクが空いてしまったし、
それにずっとむつかしい数学やってちゃ初見さんどっか行っちゃいますよ

自然数(0, 1, 2, 3, ...)
100も自然数、グラハム数も自然数

この自然数をずっと続けても届かないのが最初の無限、ω

この辺は寿司 虚空編に書いてあったけど初めはよく分からなかった
34 :
2015/12/01(火) 22:14:51.37 ID:kGtJV6AW
>>21はいわゆるポエムに属する発言なのでは
35 :
132人目の素数さん
2015/12/01(火) 22:18:56.49 ID:Semh/LDE
順序数同士も計算はできる。

ω, ω+1, ω+2, ... といってまた届かないのがω×2=ω+ω。
そして0, ω, ω×2, ω×3, ω×4, ... といって届かないのがω×ω=ω^2

そして、ある順序数未満の順序数と足し算を使って表せない(additive principal)順序数は、
1, ω, ω^2, ω^3, ...


1は、それより小さいものは0しかなく、0と足し算では1を表せない。
ωは、最初の無限だから、自然数同士を足してもωより小さい。
ω^2は、ω+ω+・・・の極限。

そしてφ(0,β)はβ番目のadditive principalな順序数。
ω番目もある。(ω^ω)
36 :
132人目の素数さん
2015/12/01(火) 22:20:34.34 ID:Semh/LDE
>>34
まあ、「このスレで出た一番大きい数+1」と同じようなことだけど、
明らかに冬おこじょ数や可融差関数なども含まれるので、かなり大きな数になると予想される。
37 :
2015/12/02(水) 03:28:00.81 ID:B2TQOKeV
ω^ω/ω!
0〜1のω個の数をかけた逆数
38 :
2015/12/02(水) 14:14:31.54 ID:XvzutfaU
>>36
いや、マジレスするなら非可算の数が含まれているのだから非可算無限で「大きな実数(有限)」ではない。
39 :
132人目の素数さん
2015/12/02(水) 22:02:40.49 ID:lEKrYlHC
>>38

すまん 正しくは
「今までに誰かが一度でも定義または書いたことがある0より大きく1より小さい実数をすべて掛けた数の逆数」
だった
40 :
132人目の素数さん
2015/12/02(水) 22:10:30.33 ID:ZWWss0wP
素数の逆数を1/2から順番に足していって、総和が5を超える瞬間の分母。
41 :
132人目の素数さん
2015/12/04(金) 10:58:16.77 ID:SLPBLRKF
この考え方って既出?

StI(a)=a→a→a

StI(a)n=StI(a)nー1→StI(a)nー1→StI(a)nー1
42 :
2015/12/04(金) 14:31:57.96 ID:SLPBLRKF
追記
StI(a)'=a→a→a→(a回チェーン拡張)

軽くふぃっしゅ数越えね?
43 :
132人目の素数さん
2015/12/04(金) 19:31:04.01 ID:v+Ncq/8l
>>39 f(x)=1/x と定義するとその数は∞
>>41 StI(a)n≒a→a→n→2
44 :
2015/12/04(金) 20:00:17.39 ID:SLPBLRKF
訂正します

1→1
2→2→2
3→3→3→3
4→4→4→4→4
こういった感じのを式にする考えです。

StI(a)=a→a→<aー1の回数だけチェーン拡張>
45 :
2015/12/04(金) 20:12:32.60 ID:SLPBLRKF
>>43
訂正前のですが、
a=2の場合StI(2)1=16
StI(2)2=StI(2)→StI(2)→StI(2)=16→16→16
なので近似しません。わかりづらくてすみません。
46 :
2015/12/04(金) 20:54:59.08 ID:UiFuE/6P
チェーンてあんま強くないんじゃなかったっけ
47 :
2015/12/04(金) 21:26:25.26 ID:SLPBLRKF
>>44追記
StI(a)=a→a→<aー1の回数分チェーン拡張>

この()の中にStI(a)を入れるとチェーンの長さも大爆発する。

StI(StI(a))=StI(a)→StI(a)→<StI(a)ー1の回数分チェーン拡張>

()の中にさらにStI(StI(a))を入れるとかっこが増えてかさ張るので、

StI(StI(StI(a)))
=StI(a)3

て感じにStI(a)nでまとめられるが代入する数値は1つだけにしたいので、
かっこを重ねる回数もa回にする式を作る。

StI((a))=StI(a)a

StI((2))=StI(StI(2))
=StI(16)=16→16→16→…(→計16)

StI((3))=StI(StI(StI(3)))
=StI(StI(3→3→3→3))
=StI((3→3→3→3)→(3→3→3→3)→<(3→3→3→3)の回数だけチェーン延長>

StI((3))でこれならStI((64))とかもうふぃっしゅ余裕越えじゃね?甘い?
48 :
2015/12/04(金) 21:50:57.55 ID:zTRMks5w
よくわからないけど、Stl(a)は拡張チェーンでa(→2)aくらい?
49 :
2015/12/04(金) 22:08:27.38 ID:SLPBLRKF
Stl(3)=3→3→3→3

3(→2)3=?


拡張表記ググってもガラケーじゃ式の図が表示されなかったり小さすぎてわからんからスルーしてたが、
誰か教えて下さい。
50 :
2015/12/04(金) 23:03:19.98 ID:SLPBLRKF
訂正
2→2→2は16じゃなく4だった
これひっかけだよな
51 :
2015/12/04(金) 23:11:31.51 ID:hO+vBRY3
http://ja.googology.wikia.com/wiki/%E6%8B%A1%E5%BC%B5%E3%83%81%E3%82%A7%E3%83%BC%E3%83%B3%E8%A1%A8%E8%A8%98

一番下から「デスクトップ版」を選べば数式が出る
52 :
2015/12/04(金) 23:25:43.17 ID:SLPBLRKF
貼ってもらって申し訳ないがガラケーだと文字重なってたり文中にタグ表記化けがあったりでそのリンクも式も見つからないんですわ
53 :
2015/12/05(土) 01:50:20.29 ID:tBAJaGbt
a(→c)b = a(→cー1)a(→cー1)…(→cー1)a(→cー1)a
        b 個の (→cー1)

他のチェーンの規則は、そのままであり、→の下についている数字を無視して計算できる。
したがって、 3(→2)3→3 のような表記はできない。
つまり、矢印のタイプ(→の下についている数字)は、1通りでなければならない。
f(n)=n(→n)n が急増加関数で f[ω^3](n) 程度
54 :
2015/12/05(土) 02:00:13.58 ID:tBAJaGbt
巨大数Wikiによれば、ふぃっしゅ数は
バージョン1が 3(→2)63(→2)2 程度
バージョン2が 3(→64)3(→64)2 程度
55 :
2015/12/05(土) 02:13:36.86 ID:tBAJaGbt
StI((a)) = StI(a)a が a(→2)a(→2)2 くらいかな?
StI((64)) が 64(→2)64(→2)2 で、ちょうどふぃっしゅ数V1を超えるくらいか。
56 :
132人目の素数さん
2015/12/05(土) 08:25:14.34 ID:8CHjWAgt
おお、ありがとうございます
ただ三つ組の場合がよくわからないのですが、
3(→2)63(→2)2なら、
63→63の回数だけ3→3→…を繰り返すってことですか?
57 :
2015/12/05(土) 08:49:59.23 ID:8CHjWAgt
StI(a)=a(→2)a

StI((a))=StI(a)a

StI((3))=StI(StI(StI(3)))
=StI(StI(3→3→3→3))
=StI(3→3→3→3)(→2)StI(3→3→3→3)


>>56が正しければこういう事か
58 :
2015/12/05(土) 09:06:08.44 ID:7TNQfcCI
アホテスト

アホテスト

アホテスト
59 :
132人目の素数さん
2015/12/05(土) 09:29:49.72 ID:8CHjWAgt
ああ63(→2)2は63→63→63か
60 :
2015/12/05(土) 10:02:02.30 ID:aJUpXbBV
>>56
「他のチェーンの規則は、そのままであり、→の下についている数字を無視して計算できる。 」
3(→2)63(→2)2 = 3(→2)(3(→2)62(→2)2)(→2)1 = 3(→2)(3(→2)62(→2)2
= 3(→2)(3(→2)(3(→2)61(→2)2)(→2)2
= 3(→2)(3(→2)(3(→2)(3(→2)(3(→2)60(→2)2)(→2)2)(→2)2)
= ...
61 :
132人目の素数さん
2015/12/05(土) 10:06:39.95 ID:8CHjWAgt
a(→a)aこれ反則だろ、勝ち目ないわ
3の時点で、
3(→2)3(→2)3(→2)3
=3(→2)3(→2)(3→3→3→3)
とかさらに
C(a)=a(→a)a
C(C(a))=C2(a)
C(C(C(a)))=C3(a)
Ca(a)
とかやったら
最新版ふぃっしゅってC64(64)よりでかいのか?
62 :
2015/12/05(土) 10:08:07.47 ID:aJUpXbBV
f(n) = 3(→2)n(→2)2 とすると

f(n)= 3(→2)f(n-1)(→2)1 = 3(→2)f(n-1)
 = f(n-1)の回数だけ3→3→…を繰り返す

ということだから、>>56 の解釈は大きさが全く違う
63 :
2015/12/05(土) 10:09:42.33 ID:8CHjWAgt
>>60
どうもです
拡張むずい
64 :
2015/12/05(土) 10:09:55.55 ID:aJUpXbBV
>>61
f(n)=n(→n)n が急増加関数で f[ω^3](n) 程度
だから、バージョン3以降はC64(64)よりでかい
65 :
2015/12/05(土) 10:11:45.38 ID:8CHjWAgt
マジかよ…ふぃっしゅ甘く見過ぎていましたわ…
66 :
2015/12/05(土) 10:15:40.88 ID:8CHjWAgt
最期に恥を晒して帰りますわ
StIはStayaway to Infinityの略です
67 :
132人目の素数さん
2015/12/06(日) 14:41:15.40 ID:lKkpODsm
p(0)=1
p(a+1): P(a)={x|p(a)<x≦y;xは素数}、P(a+1)の総和≧p(a)になるときの最小のy

p(0)=1, p(1)=5, p(2)>10000
68 :
2015/12/07(月) 18:19:56.39 ID:wb9Eoq6l
問一 Σ_1論理式でωを定義せよ
69 :
132人目の素数さん
2015/12/07(月) 18:40:35.21 ID:2tEdA2Nn
∀x<ω S(x)<ω
70 :
2015/12/07(月) 18:41:56.59 ID:nEGhqiqo
それは極限数の定義
71 :
132人目の素数さん
2015/12/07(月) 18:48:34.27 ID:2tEdA2Nn
∀ω<x S(x)<ω ∧ ?∃y<ω∀x<y S(x)<y
72 :
132人目の素数さん
2015/12/08(火) 18:40:20.95 ID:17Z768kx
?ω?>(^ω^)<(・∀・)>
73 :
2015/12/08(火) 20:22:44.02 ID:GjmOCCy0
一階部分が自然数の集合になっているものとします。Σ_1-帰納法を用いて
∃a[(0<a<α∧(x<a<α→x+1<a<α))→x<α]
αは超限順序数ですが、この際どの順序数かは気にしません

∃a[(α<a<α*2∧(α+x<a<α*2→α+x+1<a<α*2))→α+x<α*2]
∃a[(α<a<α^2∧(α*x<a<α^2→α*(x+1)<a<α^2))→α*x<α^2]

積分や無限数列なんかで極限をとるのと同じ要領です。
そういうわけでRCA_0の証明論的順序数はω^ω

Σ_1論理式でωを定義する方法はわからん。たぶんbig fiveの公理系じゃ無理
寿司は近いうち更新されそう
74 :
132人目の素数さん
2015/12/09(水) 20:15:32.23 ID:+AC0b1Lb
Π^1_1-CA_0って何を使える公理系なの?
75 :
2015/12/09(水) 22:07:54.26 ID:WwP9mdhG
∃a[(α<α+a<α*2∧(α+x<α+a<α*2→α+x+1<α+a<α*2))→α+x<α*2]
∃a[(α<α*a<α^2∧(α*x<α*a<α^2→α*(x+1)<α*a<α^2))→α*x<α^2]
でしたね。失礼

Π^1_1-CA_0はロビンソン算術の公理系からはじめていろいろあって、
Π^1_1に関する内包公理(comprehension axiom)をつかえるようにした体系です。
と言ってもよく分からないと思うので、big fiveの弱いほうから自分でいろいろ試してその強さを実感したほうがいいと思います。
76 :
2015/12/09(水) 22:56:35.92 ID:WwP9mdhG
算術的内包公理
∀x∀y[x,y∈A_t∧x>y∧?x+y=α←→α∈A_{t+1}]
A_s:s∈A_tとか
A_tのtがTaranovskyのCでいうところの順序数の次数に対応しています。
本当はもっと厳密にやらないとあれだけど、こんな感じでε_0まで
77 :
2015/12/10(木) 09:20:27.70 ID:9vqQlERG
スレ違いなのわかっているんですがどうしてもわからないので教えてください

損益分岐点の問題です
工場である商品を製造する場合、100個製造すると1個あたり200円が採算ライン、400個製造するとき1個あたり120円が、採算ラインになる。

(1)800個製造すれば1個あたりいくらが、採算ラインになるか
   答えは、106円

 (2)1個あたり100円を採算ラインにするためには、何個発注すればよいか
    答えは、1529個

  計算方法を教えて下さい。お願いします
78 :
2015/12/10(木) 13:16:33.89 ID:LphI+Sqv
総費用関数の定め方によって、任意の自然数を答えにすることができる。
79 :
2015/12/10(木) 14:15:45.95 ID:Hn/egwyi
よく知らんけど線形関数でいーんじゃねーのそこは
80 :
2015/12/11(金) 00:36:09.51 ID:XCEYabrS
3次式とか普通に使われてるよ。問題文に書かないのが悪い。
81 :
2015/12/11(金) 18:41:20.25 ID:Peo0PYFc
まずは(0,0,...,0)(1,1,...,1)を展開してあらわれる形を標準形として、計算が終了することを証明。これはCを利用すれば比較的簡単にできそう。
それ以外の形は超準形として拡張すればいい。
多次元やテトーレーション配列へはそれからでもいい
82 :
2015/12/11(金) 21:10:14.41 ID:Ldx+2Edf
まずは (0,0,...,0)(1,1,...,1) が計算終了することを証明できれば十分だよ。
83 :
2015/12/12(土) 14:27:18.84 ID:j+I0yB3F
より洗練されたかたちへ

(0,1)→(0,0,...,0)(1,1,...,1)
(0,1)(1,0)→(0,1)(0,1)...(0,1) 右から展開
(0,1)(1,0)(2,1)→(0,1)(1,0)(2,0)...(n,0)
(0,1)(1,1)→(0,1,0,...,0)(1,0,0,...,0)(2,1,...,1)
(0,1)(1,2)→(0,1)(1,1)(2,1)...(n,1)
(0,1,0)(1,2,1)→(0,1)(1,2)...(n,n+1)
(0,1,0,1)→(0,1,0,0,...,0)(1,2,1,1,...1)
(0,1,1)→(0,1,0,1,...,0,1)
(0,1,2)→(0,1,1,...,1)
(0(0)(0)(0)0(0)(0)1(0)(0)(0)1)→(0,1,...,n)

(0(X^(m+1))0(X^m)1(X^(m+1))1)→(0(X^m)1(X^m)...(X^m)n) 2つの次元のなかで標準形となっている
84 :
2015/12/12(土) 14:37:25.12 ID:j+I0yB3F
この調子で(0(1))→(0(0(...(0(0)1)...)))まで

バシク表記法のテトレーション配列という名称ですが、よく考えたら(0(0,0)(1,1)1)の時点で構造が
テトレーション配列になっているのでこの言い方はあまり意味がないと思っただ
そしてレギオン配列も取り込まれ・・・恐ろしい子
85 :
2015/12/12(土) 17:39:51.23 ID:C4QUzQyq
>>84 さらにバシクレギオン自体を圧縮して第二バシクにできれば・・・
86 :
2015/12/12(土) 21:11:28.63 ID:j+I0yB3F
(0(1))でレギオンの役割は果たせています。新しく記号を追加して
(0[0]1)→(0(1...(n)...))
としてミーミーミーなんたらまで定義してもいいけど後はだれかが考えてくれるだろう

以下、3行のまとめ
(0,0,0)(1,1,1)=C(C(Ω+1,0),0)=ψ(Ω_ω)
(0,0,0)(1,1,1)(1,1,1)=C(C(Ω+1,0)*2,0)=ψ(Ω_ω*2)
(0,0,0)(1,1,1)(2,2,0)(3,3,1)=C(C(Ω+1,C(Ω+1,0)),0)=ψ(Ω_{ω*2})
(0,0,0)(1,1,1)(2,2,1)=C(C(Ω+2,0),0)=ψ(Ω_{ω^2})
(0,0,0)(1,1,1)(2,2,1)(2,2,1)=C(C(Ω+3,0),0)=ψ(Ω_{ω^3})
(0,0,0)(1,1,1)(2,2,1)(3,1,1)(4,2,1)=C(C(Ω+C(Ω+2,0),0),0)=ψ(Ω_{Ω_{ω^2}})
(0,0,0)(1,1,1)(2,2,1)(3,2,0)=C(C(Ω*2,0),0)=ψ(ψ_I(0))
(0,0,0)(1,1,1)(2,2,1)(3,2,0)(2,2,0)(3,3,1)(4,4,1)(5,4,0)=C(C(Ω*2,0),C(C(Ω*2,0),0)=ψ(ψ_I(1))
(0,0,0)(1,1,1)(2,2,1)(3,2,0)(2,2,1)=C(C(C(Ω*2,0)+1,0),0)=ψ(ψ_I(ω))
(0,0,0)(1,1,1)(2,2,1)(3,2,0)(2,2,1)(1,1,1)(2,2,1)(3,2,0)(2,2,1)=C(ひどくごちゃごちゃ)=ψ(ψ_I(ω)+ψ_{Ω_{ψ_I(0)+1}}(ψ_I(ω)))
(0,0,0)(1,1,1)(2,2,1)(3,3,1)=C(C(C(Ω*2,0)+2,0),0)=ψ(ψ_I(ω^2))
(0,0,0)(1,1,1)(2,2,1)(3,3,1)(4,3,0)=C(C(Ω*2,0)*2,0)=ψ(ψ_I(I))
(0,0,0)(1,1,1)(2,2,1)(3,3,1)(4,3,1)=C(C(Ω*ω,0),0)=ψ(ψ_I_ω(0))
(0,0,0)(1,1,1)(2,2,1)(3,3,1)(4,4,1)=C(C(Ω*ω^2,0),0)=ψ(ψ_I_{ω^2}(0))
(0,0,0)(1,1,1)(2,2,1)(3,3,1)(4,4,1)(5,4,0)=C(C(Ω^2,0),0)=ψ(ψ_{χ(1,0)}(0))
(0,0,0)(1,1,1)(2,2,1)(3,3,1)(4,4,1)(5,5,1)(6,6,1)(7,6,0)=C(C(Ω^3,0),0)=ψ(ψ_{χ(2,0)}(0))
(0,0,0)(1,1,1)(2,2,2)=C(C(Ω^ω,0),0)=ψ(ψ_{χ(ω,0)}(0))
(0,0,0)(1,1,1)(2,2,2)(3,3,1)=C(C(Ω^ω^2,0),0)
(0,0,0)(1,1,1)(2,2,2)(3,3,1)(4,4,2)=C(C(Ω^C(Ω^(Ω+ω),0),0),0)
(0,0,0)(1,1,1)(2,2,2)(3,3,2)=C(C(Ω^(Ω*ω),0),0)
(0,0,0)(1,1,1)(2,2,2)(3,3,3)=C(C(Ω^Ω^ω,0),0)
(0,0,0,0)(1,1,1,1)=C(C(ε_{Ω+1},0),0)
87 :
132人目の素数さん
2015/12/12(土) 23:00:54.62 ID:1eb7iBDp
新しく記号を追加するなら記号の種類を行列に対応させればいいよね多分
88 :
2015/12/12(土) 23:04:23.38 ID:1eb7iBDp
>>67 だけどp(x)はテトレーション程度の強さはありそうに思える
89 :
2015/12/13(日) 00:47:45.88 ID:jrmNNddA
2重リストアッカーマンって
n個目の引数がaであることを、[n, a] と表現する
ってあるけどこれって本質的に関数の増大に寄与してるの?
この表現方法を使わないで多変数アッカーマンを2重リスト化した場合と比べたら
どの程度大きさ変わってくるの?
90 :
2015/12/13(日) 16:20:36.10 ID:cRfdOl15
>87
元の定義を崩さずにもういっぱいいっぱいやってきたし、記号を追加するならこのタイミングがいいという判断です。
とにかく記号をすくなくするなら0と1だけで十分だしね
91 :
2015/12/13(日) 16:49:14.72 ID:jEDjnr3M
まあ究極的には点をn個並べるだけでも十分なんだけどね

順序数を実数に拡張したら
有限の数 < 0.00001ω < 0.1ω < ω
になるかな?
92 :
2015/12/13(日) 18:22:01.60 ID:cRfdOl15
有限の範囲ではそれでいいけど無限論理では事情が違ってくることに注意>n個の点
93 :
2015/12/13(日) 19:04:49.82 ID:jEDjnr3M
>>92
91の1行目は情報の表現の仕方について言っただけ
3 = 11 = ...
94 :
2015/12/14(月) 01:47:54.12 ID:0iVyqDUG
>>91
2ω=ωなので、考えるとしたらω*0.5 = ω/2 が存在するかどうか?だけど、
もし α = ω/2 < ω が存在するとして、αが有限ならばα*2も有限となるので矛盾、
αが無限ならωが最小の超限順序数であることに矛盾するから、αは順序数ではなくなる。

順序数の拡張というよりも、順序数ではない独自の何かを定義するしかないかな。
95 :
2015/12/14(月) 22:20:02.96 ID:VCfQiqx3
ωは最小の無限ではなく有限の数との間にアレフ1個の点があるとすれば解決?
96 :
2015/12/14(月) 22:22:46.29 ID:VCfQiqx3
f(0)=0
f(α+1)=f(α)
f(極限順序数 α)=sup{m>f(α[n])|n<ω}
f(α)=α/ω ?
97 :
2015/12/14(月) 22:23:11.11 ID:VCfQiqx3
あ、supじゃなくてminかな
98 :
2015/12/14(月) 22:34:01.55 ID:ocLDTp4g
https://ideone.com/c7WVL0

これ誰か大きさ見積もってくれ〜
前も上げてるけどスルーされてるT△T
ε_0 行く?
99 :
2015/12/14(月) 22:50:05.07 ID:0iVyqDUG
>>95
それでは、ωじゃなくてω_1になってしまう
100 :
2015/12/15(火) 00:05:49.21 ID:+77bKgK/
>>98
面倒なんだよ C#できてかつ読み解く意欲がある人がどれくらいいると
自分で考えるのが一番早いぞ
101 :
2015/12/15(火) 00:14:40.17 ID:CfWkvBdA
考え方がわからんT△T
順序数とプログラムの対応付けとかみんなどうやってんの?
102 :
2015/12/15(火) 03:52:32.95 ID:lUoPcBQZ
自分なりに分かったところまで努力した結果が見えないと、最初から人任せのように見える
103 :
2015/12/15(火) 21:20:03.73 ID:CfWkvBdA
とりあえずBEAF読んでるけど理解が追いつかない。
104 :
2015/12/15(火) 22:19:13.38 ID:CfWkvBdA
アッカーマンとかヒドラゲームのほうが参考になりそう?
105 :
2015/12/15(火) 22:40:58.13 ID:CfWkvBdA
んー>>98はω^ω^ωくらい?
よくわからん。
106 :
2015/12/15(火) 23:27:56.77 ID:CfWkvBdA
2重リストアッカーマン読んでる。
よくわからん。
107 :
2015/12/16(水) 00:12:07.74 ID:BS/afaXq
単純なループ(n*n*n*...) → f_2ぐらい
2重ループ
 例: n=3; n.times{n.times{n*=n}} ⇔ n**=n**=n**=8 , f_3ぐらい
3重ループ
 例: n=3; n.times{n.times{n.times{n*=n}}}, f_4ぐらい
n重ループ
 例: n=3; f=proc{|g|proc{n.times &g}}; g=proc{n*=n}; n.times{g=f[g]}; g[], f_ωぐらい
108 :
2015/12/16(水) 00:14:56.24 ID:7a5lTG6j
>>98はn重ループは超えてる。
多変数アッカーマンも超えてると思う。
109 :
2015/12/17(木) 04:26:42.50 ID:1EAmoXv4
http://pastebin.com/ivQxt5ee

計算してみるとまんま1行のバシク行列だった
ε_0なのかな?
110 :
2015/12/18(金) 19:59:15.12 ID:yht5ga3o
直接適当な情報で関数を定義するよりも、関数から関数への写像をつかさどる関数を定義したほうが効率がいい
→具体的な定義はなんらかのアルゴリズムで画一化し、情報を型の定義にまわすことではるかに強くなる。

バシク行列において、(0)列および(0)列から始まる行列は関数の型に属すると考える
(1)列および(1)列から始まる行列は関数から関数への写像の型に属すると考える
(2),(3),...についても同様に、それぞれm(n)と対応する型に属する


(0)(1)(2)(1)は一番右の(1)が関数(0)(1)(2)をうけとって新しい関数を返す。

ここでは型と構造に注目し、これを発展させていくことで具体的な定義には触れなくても影響はほとんどないようになる。
計算の優先順位や入れ子構造についてはバシク行列の定義を参照

(1,1)は(1)と同じ型に属するが、計算の過程で引数の型を変化させることがある(返す値の型が変化するわけではない)。

以上、考え方にも依るところがあるかもしれないが、型の変化がない。
(0)によって表される関数がそれぞれ固有の型に属するとむりやり考えれば変化はあるが意味を見いだせない。
111 :
2015/12/18(金) 20:00:28.60 ID:yht5ga3o
おもに形式的な定義

(1)をそのはたらきを(0)に依存する型とかんがえる。
(0)(1)で関数m(1)、(0)(0)(1)でm(2)、(0)(0)(0)(1)でm(3)、・・・これでε_0まで
(0)(1)(1)は(1)の配列の次元をつかさどる。


(0)(1)(0)(1)(0)(1)(1)(0)(0)(1)(0)(0)(1)(0)(1)(1)でε_0

(0)(0)(1)(1)は次元の関数、(0)(0)(0)(1)(1)は次元のm(2)、
(0)(1)(1)(1)は(1)(1)の配列の次元

以上、バシク表記法のレギオン配列まで。
(2)は値から型への写像で、(0)(0)...(0)(2)=(0)(1)(1)...(1)とか
(0)(1)(2)→(0)(1)(0)(1)(1)...(0)(1)(1)...(1)とかなるようにしたい

CoCを勉強してきます。
112 :
2015/12/19(土) 22:52:11.32 ID:Zr8uMjcd
C(Ω*(2+n),0)がどうしても再帰的n-到達不可能基数になってTaranovsky氏の計算結果とあわない。
そうだとすると
(0,0,0)(1,1,1)(2,2,2)=C(C(Ω*ω,0),0)
(0,0,0)(1,1,1)(2,2,2)(3,3,3)=C(C(Ω^(Ω*ω),0),0)
(0,0,0)(1,1,1)(2,2,2)(3,3,3)(4,4,4)=C(C(Ω^Ω^(Ω*ω),0),0)
となるけど結果的に
(0,0,0,0)(1,1,1,1)=C(C(ε_{Ω+1},0),0)
となるのはかわらない。

1.こっちの計算が間違えてる
2.Taranovsky氏の計算が間違えてる
3.そもそもこっちの読解がまちがえてる。

できれば3がいいけど、もうね、まかせる(ぶん投げ)
113 :
2015/12/20(日) 21:36:37.14 ID:bLK4OLo+
あれはC1関数だったか、よかった。
ψ関数を基準にして、(=は等しいではなく同じはたらきをするという意味です)
C(Ω*2,0)はα→Ω_αが成り立つ最初の十分大きい順序数、すなわち最初の到達不可能基数(本当は違うけど)
C(C(Ω*2,0),0)はα→Ω_αが成り立つ最初の順序数、すなわちψ_I(0)
んでもってC(C(C(Ω*2,0),0),0)=ψ(ψ_I(0))
C(C(Ω*2,0),C(C(Ω*2,0),0))=ψ_I(1)
C(C(Ω*2,0)+C(Ω*2,0),0)=ψ_I(I)
C(Ω*2,C(Ω*2,0))=I_2
C(Ω*2+1,0)=I_ω
C(Ω*3,0)=χ(1,0)
C(Ω*ω,0)=χ(ω,0)=(2,2,2)
C(Ω*ω+1,0)=χ(ω^2,0)=(2,2,2)(3,3,1)
C(Ω*ω^2,0)=(2,2,2)(3,3,2)
C(Ω^2*ω,0)=(3,3,2)(4,3,2)
C(Ω^ω*ω,0)=(3,3,2)(4,4,2)
C(Ω^ω^2,0)=(2,2,2)(3,3,3)
C(Ω^(Ω*ω),0)=(4,3,0)(3,3,3)
C(Ω^(Ω*ω^2),0)=(3,3,3)(4,4,3)
C(C(Ω^Ω^ω),0),0)=(4,4,4)
C(Ω^Ω^ω^2,0)=(4,4,4)(5,5,5)
C(Ω^Ω^Ω^ω,0)=(6,6,6)

また計算結果を修正しないといけないけど、ψ関数の解析結果はあれで確定ということでいいし、
Cによる評価は上のように引数の最大値だけを解析すればいいな
114 :
2015/12/20(日) 21:39:47.26 ID:bLK4OLo+
訂正
C(Ω^Ω^ω,0)=(4,4,4)
115 :
2015/12/21(月) 14:18:27.39 ID:N35rPqLA
寿司9話来てる
116 :
2015/12/21(月) 23:12:39.25 ID:AVLHqC0z
きたー
117 :
132人目の素数さん
2015/12/22(火) 18:51:37.56 ID:yR59hPA0
にわかだから
寿司9話の s_(i-1)(0,n)=f(n) がもうわかんねえ
118 :
2015/12/22(火) 21:33:17.34 ID:dGKHfwus
>>117
ただの定義だぞ
変数に値を代入したら計算できる

f(n)=n+1, i=2としておいて
g(3) = S_2(0,3) = S_1(3,3) = S_1(2,S_1(3,2)) = ... = 63

f(x)にS変換することをS(f)(x)と書くと、S_0はf(x)、S_1はS(f)(x)、S_2はS(S(f))(x)が計算できる。

JavaScript:
function S(f){
 function B(m,n){ return m==0 ? f(n) : n==0 ? B(m-1,1) : B(m-1,B(m,n-1)) }
 return function(x){ return B(x,x) }
}
119 :
2015/12/22(火) 21:35:38.28 ID:dGKHfwus
間違えた、
f(n)=n+1, i=1としておいて
g(3) = S_1(0,3) = S_0(3,3) = S_0(2,S_0(3,2)) = ... = 61
120 :
2015/12/23(水) 13:38:00.71 ID:xQXnQnvo
ψ関数やCに関することも含めていろいろひどい間違いをしていた。解説をしながら修正

行列 M_0 が極限順序数 に対応するとする。その1行目の値にn足したものを M_n とすると、M_0M_1M_2...=α^α^α^...
M_0 の1列目の1行目と2行目の値に1を足し3行目以降はすべて0にした列を E とすると、M_0E=ε_{α+1}

(1,1,1)はC(Ω+1,0)、ψ関数の中ではΩ_ω、後ろに(2,2,1)をn個くっつけてそれぞれC(Ω+n+1,0)、Ω_{ω^(n+1)}
(1,1,1)(2,2,1)(3,2,0)でα→C(Ω+α,0)、α→Ω_αの最初の不動点、C(Ω+C(Ω*2,0),0)、ψ_I(0)
C(Ω*2,0)、I はそれぞれ最初の十分大きい不動点
(1,1,1)(2,2,1)(3,2,0)(2,2,1)はそのω番目の不動点と対応する。C(Ω+C(Ω*2,0)+1,0)、ψ_I(ω)
さらに上と同様に(1,1,1)(2,2,1)(3,2,0)(2,2,1)(2,2,1)...(2,2,1)でC(Ω+C(Ω*2,0)+n,0)、ψ_I(ω^n)
(3,2,0)で閉じて最初の不動点、C(Ω+C(Ω*2,0)*2,0)、ψ_I(I)
(2,2,1)をうしろにつけてω番目の不動点、C(Ω+C(Ω*2,0)*2+1,0)、ψ_I(I*ω)
これを繰り返して (1,1,1)(2,2,1)(3,2,0)(3,0,0)=C(Ω+C(Ω*2,0)*ω,0) or ψ_I(I^ω)

間違えやすそうなところ

(1,1,1)(2,2,1)(1,1,1)(2,2,0)(3,3,1)(4,4,1)=ψ_{Ω_{ω+1}}(Ω_{ω^2})+ψ_{Ω_{ω+1}}(Ω_{ω^2})
(1,1,1)(2,2,1)(1,1,1)(2,2,0)(3,3,1)(4,4,1)(3,3,1)(4,4,0)=ψ_{Ω_{ω+1}}(Ω_{ω^2})+ψ_{Ω_{ω+1}}(Ω_{ω^2}+ψ_{Ω_{ω*2+1}}(0))
(1,1,1)(2,2,1)(1,1,1)(2,2,1)=ψ_{Ω_{ω+1}}(Ω_{ω^2}*2)
(1,1,1)(2,2,1)(2,2,1)=ψ_{Ω_{ω+1}}(Ω_{ω^3})
(1,1,1)(2,2,1)(3,2,0)(1,1,1)(2,2,1)(3,2,0)=ψ_{Ω_{ω+1}}(ψ_I(0)*2)
(1,1,1)(2,2,1)(3,2,0)(2,2,1)=ψ_{Ω_{ω+1}}(ψ_{Ω_{ψ_I(0)+1}}(ψ_I(ω)))
(1,1,1)(2,2,1)(3,2,0)(2,2,1)(1,1,1)(2,2,1)(3,2,0)(2,2,0)(3,3,1)(4,4,1)(5,4,0)(4,4,1)=ψ_{Ω_{ω+1}}(ψ_{Ω_{ψ_I(0)+1}}(ψ_I(ω))*2)
(1,1,1)(2,2,1)(3,2,0)(2,2,1)(1,1,1)(2,2,1)(3,2,0)(2,2,0)(3,3,1)(4,4,1)(5,4,0)(4,4,1)(3,3,1)(4,4,1)(5,4,0)(4,4,0)=ψ_{Ω_{ω+1}}(ψ_{Ω_{ψ_I(0)+1}}(ψ_I(ω))+ψ_{Ω_{ψ_I(0)+1}}(ψ_I(ω)+ψ_{Ω_{ψ_I(1)+1}}(0)))
(1,1,1)(2,2,1)(3,2,0)(2,2,1)(1,1,1)(2,2,1)(3,2,0)(2,2,1)=ψ_I(ω)*2
121 :
2015/12/23(水) 13:39:47.27 ID:xQXnQnvo
(1,1,1)(2,2,1)(3,2,0)の後ろに(3,2,0)をくっつけてα→ψ_I(I^α)となる最初の不動点、C(Ω+C(Ω*2,0)^2,0)、ψ_I(I^I)
(2,2,1)を後ろにくっつけてω番目の不動点、C(Ω+C(Ω*2,0)^2+1,0)、ψ_I(I^I*ω)
そんなこんなで(1,1,1)(2,2,1)(3,2,0)(4,0,0)=C(Ω+C(Ω*2,0)^2*ω,0)、ψ_I(I^(I*ω))
(X,2,0)とC(Ω*2,0)、Iとのつながりが見えてきたところで、(3,2,0)(4,3,1)(5,4,1)(6,4,0)=C(C(Ω*2,C(Ω*2,0)),0)、ψ_{Ω_{I+1}}(ψ_I_2(0))
(1,1,1)(2,2,1)(3,2,1)=C(Ω*2+1,0) or ψ_{Ω_{I+1}}(ψ_I_ω(0))
行列をなす各列が最初に0列に係るため、巨大基数の効果が(1,1,1)からはじまる行列へと受け継がれます。おっかない


おまけ

ものさしの強さ

1 関数
 1ー1 指数と指数タワー
 1ー2 テトレーションと上矢印表記
 1ー3 チェーン表記
 1ー4 多変数アッカーマンとBEAFの線形配列
 1ー5 多次元配列とs(n)変換
 1ー6 カントール標準系
2 順序数
3 巨大基数
4 算術
5 述語論理

下に行くほど抽象的で次元の違う強さになります。型付きバシク行列で5までぶっこみたい
122 :
2015/12/23(水) 20:17:46.19 ID:xQXnQnvo
(3,2,0)最初の不動点 (2,2,1)(2,2,1)...(2,2,1)n番目の不動点 (3,2,1)ω番目の十分大きい不動点
これらを組み合わせることで、(3,2,1)(2,2,1)(2,2,1)ω^3番目の十分大きい不動点、とか

(1,1,1)(2,2,1)(3,2,1)(2,2,1)=C(Ω*2+2,0)=ψ_I_{ω^2}(0)
(1,1,1)(2,2,1)(3,2,1)(2,2,1)(3,2,0)=C(Ω*2+C(Ω*3,0),0)=ψ_{χ(1,0)}(0)
(1,1,1)(2,2,1)(3,2,1)(2,2,1)(3,2,1)=C(Ω*3+1,0)=χ(1,ω)
(1,1,1)(2,2,1)(3,2,1)(3,2,0)=C(Ω*C(Ω^2,0),0)=ψ_M(0)
(1,1,1)(2,2,1)(3,2,1)(3,2,0)(2,2,1)(3,2,0)=C(Ω*C(Ω^2,0)+C(Ω^2,0),0)=ψ_M(M)
(1,1,1)(2,2,1)(3,2,1)(3,2,0)(2,2,1)(3,2,1)=C(Ω^2+1,0)=M_ω
(1,1,1)(2,2,1)(3,2,1)(3,2,0)(2,2,1)(3,2,1)(2,2,1)(3,2,0)=C(Ω^2+C(Ω^2+Ω,0),0)
(1,1,1)(2,2,1)(3,2,1)(3,2,0)(2,2,1)(3,2,1)(3,2,0)=C(Ω^2+Ω*C(Ω^2*2,0),0)
(1,1,1)(2,2,1)(3,2,1)(3,2,0)(3,2,0)=C(Ω^2*C(Ω^3,0),0)
(1,1,1)(2,2,1)(3,2,1)(3,2,0)(4,2,0)=C(Ω^C(Ω^Ω,0),0)
(1,1,1)(2,2,1)(3,2,1)(3,2,1)=C(Ω^Ω+1,0)
123 :
2015/12/24(木) 02:22:30.11 ID:A9Q9GFzy
ε_0*ω=(0,0)(1,1)(1,0)
ε_0^ω=(0,0)(1,1)(1,0)(2,1)(2,0)
ε_0^ω*ε_0=(0,0)(1,1)(1,0)(2,1)(2,0)(1,0)(2,1)
ε_0^ω^2=(0,0)(1,1)(1,0)(2,1)(2,0)(2,0)

ε_0^ε_0=(0,0)(1,1)(1,0)(2,1)(2,0)(3,1)

どこか間違ってる?
124 :
2015/12/24(木) 13:28:11.69 ID:hPMulqNe
>123 あっていると思います。

(1,1,1)(2,2,1)(1,1,0)(2,2,1)(3,3,1)=ψ_{Ω_{ω+1}}(Ω_{ω^2})+ψ_{Ω_{ω+1}}(Ω_{ω^2})
(1,1,1)(2,2,1)(1,1,1)=Ω_{ω^2}+Ω_ω
でした。最初から入れ子の内側にぶち込んでよかったな。
幸いにも後の計算には響きません。

>122の解析結果も間違えていました。すみません

α→f(α)の最初の十分大きい不動点をΩ、n番目の不動点をf_Ω(n)とする。
(1,1,1)Xがf_Ω(0)に対応するとすると(1,1,1)X(2,2,1)(2,2,1)...(2,2,1)=f_Ω(ω^n)
(3,2,0)でΩを呼び出し最初のα→f_Ω(α)、つまりf_Ω(Ω)は(1,1,1)X(2,2,1)(3,2,0)
Ωに対応してるのは(3,2,0)であり(1,1,1)X(2,2,1)(3,2,0)ではない。
しかし
(1,1,1)X(2,2,1)(3,2,1)はω番目の十分大きいα→f(α)、Ω_ωに対応。
最後の列が直接0列に係るかどうかがミソね

また、(1,1,1)X=C(α,0)として、(1,1,1)X(2,2,1)(3,2,1)=C(α+Ω+1,0)

これらを踏まえたうえで、

(1,1,1)(2,2,1)(3,2,1)(3,2,1)=C(Ω^2,0)
(1,1,1)(2,2,1)(3,2,1)(4,2,1)=C(Ω^Ω,0)
(1,1,1)(2,2,1)(3,3,0)=C(ε_{Ω+1},0)=C(C(C(C(Ω_2,Ω),0),0),0)
(1,1,1)(2,2,1)(3,3,0)(4,4,1)=C(C(C(Ω_2,Ω)+1,0),0)
(1,1,1)(2,2,1)(3,3,0)(4,4,1)(5,5,1)(6,6,0)=C(C(ε_{C(Ω_2,Ω)+1},0),0)
(1,1,1)(2,2,1)(3,3,1)=C(Ω_2+1,0)
(1,1,1)(2,2,1)(3,3,1)(2,2,1)(3,2,1)=C(Ω_2*2+1,0)
(1,1,1)(2,2,1)(3,3,1)(2,2,1)(3,3,0)=C(ε_{Ω_2+1},0)=C(C(C(C(Ω_3,Ω_2),0),0),0)
(1,1,1)(2,2,1)(3,3,1)(2,2,1)(3,3,0)(4,4,1)=C(C(C(Ω_3,Ω_2)+1,0),0),0)
(1,1,1)(2,2,1)(3,3,1)(2,2,1)(3,3,0)(4,4,1)(5,5,1)(6,6,1)=C(C(Ω_3+1,0),0)
(1,1,1)(2,2,1)(3,3,1)(2,2,1)(3,3,1)=「Taranovsky's Cの強さ」

これはよく考えたらCoC、もしくはそのサブシステムくらいの強さがあるのかもしれない。
125 :
2015/12/24(木) 22:44:29.68 ID:hPMulqNe
バシク行列の強さ

列を型とみなし、その後ろに続くその列より小さくない列からなる部分行列をその型の値とみなすことができる
→見やすさと表現力の両立
高階のカインドの実装:型から型への関数、さらにその関数をつかさどる型から・・・といった構造
→高階述語論理の力の獲得

算術とは次元が違うからどうりでCでは敵わないわけだ
ポリモーフィズムと依存型に当たるものは見当たらないからCoCではないし、loader.cには敵わないな
126 :
132人目の素数さん
2015/12/25(金) 00:13:14.29 ID:unfSRvIU
変換と関数から関数への写像U(S,f)=S(f)
変換と関数と数から数への写像V(S,f,x)=g(x) g=S(f)
バージョン1のSS変換
SS(m,f,S)=(V(S^f(m),f,m),U(S^f(m),f),S^f(m))
バージョン2のSS変換はどのような形になるのでしょうか
127 :
2015/12/25(金) 19:27:56.06 ID:BGNWJ9a1
>>126
ver2のSS変換はver1と同じ。
http://pastebin.com/JckviZLp

function U(S,f){return S(f)}

V(S,f,x)=S(f)(x)=U(S,f)(x)
128 :
2015/12/25(金) 19:47:59.35 ID:BGNWJ9a1
>>126 >>127
・・・と思ってたがどうやら違うようだった

R(S) = S^*とする

URLは同じで
129 :
2015/12/26(土) 21:37:33.86 ID:TojQpFYR
近似です

バシク行列数〜(0,1)(1,0)[10]
大バシク行列数〜(0,1,2)(1,1,0)(1,0,0)[10]

部分的に標準形として計算する際、(0,X)は(1)がかかる場合を除いて0列として扱う。
130 :
2015/12/26(土) 22:27:59.67 ID:icCGk/7c
>>129
(0,1)(1,0)[10]
(0,1)(0,1)(0,1)(0,1)(0,1)(0,1)(0,1)(0,1)(0,1)(0,1)[10]

(0,1)が最後に付いたときはどうするんです?そこから計算方法が分からないんですが
131 :
2015/12/26(土) 23:04:04.18 ID:TojQpFYR
既に上に書かれある通りで、右から(0,0,...,0)(1,1,...,1)と展開
大バシク行列は
(0,1,2,0)(1,2,3,1)(1,2,3,1)(1,0,0,0)
のほうが適切かな

ひとつ断っておくと、私はバシク行列の開発者ではなく、個人的に勝手に拡張しただけです。
132 :
2015/12/28(月) 19:19:52.76 ID:G60ZxAXJ
133 :
2015/12/29(火) 20:09:45.61 ID:Z2IJmRTV
今回の拡張では、1列目が違ったら必ず違う順序数になりますか?
(0,1,2)(1,2,3)≠(0,0,0)(1,1,1)など
134 :
2015/12/29(火) 21:28:11.91 ID:n+td/D3S
>132 大バシク行列の近似という意味です。

>133
そうなりますね。
縦方向の展開を優先します。
(0,1,2)(1,2,3)→(0,1,2,0,0,...,0)(1,2,2,...,2)
横方向には、部分的な標準形だけで大きさを比較します。
(0,1,2,0)(1,2,1,1)→(0,1,2)(1,2,1)(2,3,2)...
(0,1)と(1,2)として大きさを比較
3行目をみると0-1=-1となるためここから下の行は比較しない。
(0,1,2)(1,2,2)なら比較不能

書いてて思ったけど、定義をまとめないとあやふやだな。
135 :
2015/12/29(火) 21:33:06.67 ID:n+td/D3S
変な間違いをしていた。

3行目を見ると1-2=-1となるためここから下の行は比較しない。

です。すみません
136 :
2015/12/29(火) 21:53:25.41 ID:n+td/D3S
(0,1,2)と(1,2,2)で比較不能というのはまずいけど、それぞれの列で1行目の値を引いていけばいいのか。
この場合
(0,0,0)と(1,1,0)を比較して(1,1,0)の方が大きい。よって(0,1,2)よりも(1,2,2)の方が大きい、と。
空白のような役割を果たしている0の機能がほかの数に移っていく仕組み
137 :
2015/12/29(火) 22:24:37.17 ID:n+td/D3S
縦方向の展開を優先するから
(1,2,2)→(1,2,1,2,...,1,2)
か、そして問題となる列は
(1,2,2,1)
ボロボロなのでしばらくROM専にもどります。
138 :
2015/12/29(火) 23:17:14.66 ID:Z2IJmRTV
(1)(2)(2)(1)→どうなるか

(0,1,2)(1,2,1)(2,3,0)...でもよくね?
139 :
2015/12/30(水) 01:33:47.86 ID:0VLW5fpO
>>34
バシク行列を独自に拡張したのなら、その拡張した定義をまず書いてもらわないと、
なにをやっているのか全くわからない。
140 :
2015/12/30(水) 01:34:19.92 ID:0VLW5fpO
>>34 じゃなくて >>134
141 :
2015/12/30(水) 21:36:42.63 ID:ADw3kuv4
縦方向の展開を優先する

一番左の列のさらにひとつ左に(-1)(-1)...(-1)という列が隠れているものとする
→最悪(0)(1)まで「より小さい」という条件が満たされなくても、任意の非負整数の列は(-1)列より大きいと見なすことができる

一番右側の列S_nから一番下の0でない値を0に変えたものと最初の悪い列との差分をとり、これをΔとする。
(最初の悪い列が(-1)列となる場合、S_nの一番下の0でない値がある行から下は0にかえて計算する。
 S_nが標準形でない場合、一番下の0でない値が一番上の値と同じならば、パラメータに関数fを適用させてその一番下の0でない値を0にする。)
標準形の列は従来の定義と同じくそれぞれの行に対応するΔの値を足していく。
標準形でない列にはその列の中で、それぞれの行jと1行目との差分δ_jを取った後、1行目のみにΔの1行目の値を足し、
2行目以降は1行目にそれぞれ対応するδ_jを足した値をいれていく。

あとはBEAFのテクニックを応用して、すくなくともレギオン配列まで拡張することができます。


例 (0,1)=αとします。

(0,1)(1,0)→(0,1)(0,1)...(0,1)
(0,1)(1,0)(2,2)→(0,1)(1,0)(2,0)...(n,0)
(0,1)(1,0)(2,3)→(0,1,0,0,...,0)(1,0,0,...,0)(2,2,...,2)(3,3,...,3)
(0,1)(1,1)=ψ_{Ω_{α+1}}(0)→(0,1)(1,0)(2,3)(3,0)...(n,0)(n+1,n+2)
(0,1)(1,1)(2,0)(3,4)(4,1)=ψ_{Ω_{α+1}}(ψ_{Ω_{α+1}}(0))
(0,1)(1,1)(2,1)=ψ_{Ω_{α+1}}(Ω_{α+1})→(0,1)(1,1)(2,0)(3,4)(4,1)(5,0)...(n,0)(n+1,n+2)(n+2,1)
(0,1,0)(1,1,0)(2,2,2)=ψ_{Ω_{α+1}}(Ω_{α+ω})
(0,1,0)(1,1,1)→(0,1,0)(1,1,0)(2,3,2)(3,3,0)...(n,n,0)(n+1,n+2,n+1)
(0,1)(1,2)→(0,1,0,0,...,0)(1,1,...,1)(2,2,...,2)

最初想定していた定義とは違います。
142 :
2015/12/30(水) 21:44:47.65 ID:0VLW5fpO
よくわからないけど、本当に計算終了するの?
バシク行列ですら計算終了することが示されていないのに、
(0,1)(1,0)(2,3)→(0,1,0,0,...,0)(1,0,0,...,0)(2,2,...,2)(3,3,...,3)
のように展開されるものが計算終了するようには見えない。
それから、バシク行列と表記が同じでまぎらわしいので、せめて何か名前をつけて
区別してほしい。
143 :
2015/12/31(木) 18:49:48.83 ID:t4vV6kxx
最初想定していた定義 一番上の値がその列の最大値、もしくは0を除いて最小値となっていることを前提とする。

縦方向の展開を優先する

一番右側の列の一番上の値がその列の最大値となっている(標準形)場合はそのまま左に向かってより小さい列を探す。
標準形の列との比較は標準形のルールに則る
一番上の値がその列の0を除いた最小値となっている場合、その列の0でない値をすべてその最小値におきかえて計算する。

一番右側の列の一番上の値がその列の最小値mとなっている場合、S=((X:m以上の値が続き、s>mでおわる),(M:mの列))とし、
制約上、標準形の列と比較することはない。
一番上の値がその列の0を除いた最小値bとなっている場合、Xの部分と同じ行は標準形のルールに則って比較し、Mの部分と同じ行はすべてその最小値bにおきかえて
あとは標準形と同様に計算する。

標準形の計算が終了するならばこれ以降の拡張された計算も終了することが簡単に証明できます。ただしその分弱体化します。

バシク行列の拡張ならもうこっちでいい気がする。変にひねくれたことしなくてよかった。


例 (0,1)=αとします。

(0,1)(1,0)→(0,1)(0,1)...(0,1)
(0,1)(1,1)→(0,1)(1,0)(2,0)...(n,0)
(0,1,0)(1,1,1)→(0,1)(1,1)(2,2)...(n,n)
(0,1)(1,2)→(0,1,0,0,...,0)(1,1,...,1)
(0,1,0)(1,2,1)=ψ_{Ω_{α+1}}(0)→(0,1)(1,2)...(n,n+1)
(0,1,0,0)(1,2,1,1)=ψ_{Ω_{α+1}}(Ω_{α+ω})→
(0,1,0,1)→(0,1,0,0,...,0)(1,2,1,1,...,1)
(0,1,0,1)(1,2,1,1)→(0,1,0,1)(1,2,1,0)(2,3,2,0)...(n,n+1,n,0)
144 :
2016/01/01(金) 20:16:13.04 ID:751/G8/G
そして三次元へ・・・かな?
145 :
2016/01/02(土) 02:55:27.73 ID:SZO4lphG
細かい定理や証明は端折ってます。行列をなすそれぞれの列は上の行から下の行へむかって値が大きくなることはない(標準形)とします。

定理1

「M,Nは任意の行列
M,Nはそれぞれ計算が終了する⇒MNは計算が終了する⇒M(1)は計算が終了する⇒M(1)(1)は計算が(ry・・・」

計算が終了する行列、Nの1行目の値すべてに1を足したものをN'とする。(2)は必ず(1)に直接かかる。計算の過程で(1)(1)...(1)がでてくる。
MN'について、N'の部分の展開はN'内で閉じている。右端に出てきた(1)は次のように処理する。

MN'_k(1)→MN'_kMN'_k...MN'_k

最終的にMM...Mと展開され、Mの計算が終了するならば、かつそのときのみにMM...Mの計算は終了する。
MN'_k(N'_kはN'をk回展開してできた行列)について、N'_nを最後の展開とし、

MN'_nの計算が終了することはすでに述べたとおり。((1)を処理していくだけ)
MN'_kの計算が終了するならば、MN'_{k-1}の計算は終了する。

数学的帰納法と定理1により、MN'の計算が終了することが証明された。
MN'の1行目の値すべてに1を足したものをM'N''とする。
同様の手口で任意の行列Lの計算が終了するならばLM'N''の計算も終了することが証明される。その後も同様・・・

以上より(0,0)(1,1)まで。

ついでに任意の行列M1,M2,M3,...の計算が終了するならばM1M2'M3''...の計算も終了することが証明された。(定理2)
146 :
2016/01/02(土) 02:56:34.56 ID:SZO4lphG
Nの1行目と2行目の値すべてに1を足したものをN?とする。(2,2)は必ず(1,1)に直接かかる。計算の過程で(1,1)(1,1)...(1,1)がでてくる。
MN?ついて、N?の部分の展開はN?内で閉じている。右端に出てきた(1,1)は次のように処理する。

MN?_k(1,1)→MN?_k(MN?_k)'...(MN?_k)''...'

定理2よりこの計算は終了する。また最終的に一番右端に展開されるMM'...M''...'も計算が終了する。
それと定理1および数学的帰納法により、MN?の計算は終了することが証明された。
同様の手口でLM?N??の計算も終了することが証明される。

この論証を続けていくことで(0,0,...,0)(1,1,...,1)の計算が終了することが証明されます。

定義と証明が固まったところでバシク数とloader.cを解析したい。バシク版レギオン配列、あるいはそのもうちょっと先で結構いけると思う。
勝手が過ぎてきた気もするから、上でご指摘もあるとおり名前変えたがいいかもしれない。どんな名前がいい?
147 :
2016/01/02(土) 14:09:32.09 ID:SZO4lphG
N'の部分の展開が閉じているというのは間違いでした。
1行目のみに1を足すときはいいけど、たとえば(1,1)<(2,0)は成り立っても(2,2)<(3,1)は成り立たない。
新たに定理を導出して2行目の証明を補修することはできたけど3行目以降はわからん
148 :
2016/01/02(土) 21:24:27.14 ID:SZO4lphG
アルゴリズムを複雑にしていって計算可能な順序数と対応させる手もあるけど、システムが強力すぎて現実的でない。

プランB



(0,0,0)(1,1,1)(2,2,2)(3,3,3)S

S=(4,3,3)ならば、集合A={#(2,2,2)(3,3,3),#(4,3,2)(5,4,3),#(6,4,2)(7,5,3),...}のなかで全称量化される。
S=(4,3,0)ならば、集合B={#(2,2,2)(3,3,3),#(4,2,2)(5,3,3),#(6,2,2)(7,3,3)...}のなかで全称量化される。
S=(4,2,0)、C={#(1,1,1)(2,2,2)(3,3,3),#(4,1,1)(5,2,2)(6,3,3),#(7,1,1)(8,2,2)(9,3,3),...}

展開された後、集合は消えその要素が残る。
さらに要素=ひとつ下の階層の集合を展開して、これを最下層、すなわち空集合までくりかえす。

階層が常に下がり続けること、推移律が成り立つこと、無限下降列が存在しないことを証明する(真であるならば)。

階層が常に下がり続けることは自明なので後の二つを証明する。

行列Mの右端の列をSとする。
展開するとM1'M2'...Mn'
行列Mn'の右端の列Sn'はM{n-1}の右端の列S{n-1}'をもって右側にかかる。Sn'がS{n-1}'にかかるならば、繰り返す行列の長さの単位はかわらないが0でない
値からなる列の行の長さは一行分浅くなる。
⇒展開される行列の単位の長さ、もしくはそれを構成するある列の行いずれかが短くなる。これ以上短くできなくなると今まで積み上げてきた
繰り返し行列を消費してゆくことになる。そして次の、階層ひとつ下の部分行列について同じことを繰り返す。
⇒なんやかんやで計算が終了する。

これを高階述語論理と絡めながら厳密に証明していくことになる。わかるづらい
こうしてみると多次元配列をもっと有効に使えるんじゃないかという気もしてくる。
149 :
2016/01/02(土) 22:00:08.65 ID:PQOxFuGk
>>145-146
「M,Nはそれぞれ計算が終了する⇒MNは計算が終了する」
のところですでに引っかかる(ので、その先は追えていない)。
Nが先に消えてからMが計算されるのであれば自明だけど、
Nを消す計算の中でMの要素まで含めてコピーされてしまうときには、
MNの計算が終了することは自明ではない。
たぶん、そのあたりのことが >>147-148 に書いてあるのだと思うけど。

ちなみに、そのことはここに指摘されているようだ。
http://ja.googology.wikia.com/wiki/%E3%83%A6%E3%83%BC%E3%82%B6%E3%83%BC:Kyodaisuu/%E3%83%90%E3%82%B7%E3%82%AF%E8%A1%8C%E5%88%97%E8%A8%88%E7%AE%97%E7%B5%82%E4%BA%86%E3%81%AE%E8%A8%BC%E6%98%8E%E3%81%B8%E5%90%91%E3%81%91%E3%81%A6

> S[n] と T[n]と ST_1[n] (T_1 は、T の一番左の要素) が計算終了することが
> 示されている時に、ST[n] が計算終了する事を一般に示すにはどうすれば
> 良いか?ここが、第一の関門です。
150 :
2016/01/03(日) 00:20:57.98 ID:BMWYtFbk
K L M N O
0=M, 1=N, 2=O, 3=NK, 4=NL, 5=NM, 6=NN, 7=NO, 8=OK, ..., 13=NKK
-1=L, -2=K, -3=LO, -4=LN, ..., -7=LK, -8=KO, ..., -13=LOO
0.2=MLK, 0.04=MLLK, 0.008=MLLLK, 0.0016=MLLLLK, 1/5^5=MMKLK
1/5^10=MMKMKLK, 1/5^20=MMKMKMKLK, 1/5^30=MMKMKMKMKLK, 1/5^40=MMLKLK
1/5^50=MKMLKLK, 1/5^60=MMKMKMLKLK, ..., 1/5^100=MMLKMLKLK
1/5^150=MMLKMLKMLKLK, 1/5^200=MMLKMLKMLKMLKLK, 1/5^400=MMLLKLK
1/5^6400=MMLLLKLK, ..., 1/5^102400=MMLLLKLK, 1/5^(100e+2^15)=MMLLLLKLK
1/5^(100e+2^20)=MMMKLKLK

K, L:K→K, M:K→L, N:(K,L)→M, O:(K,L,M)→N
151 :
2016/01/03(日) 21:32:04.26 ID:I/rKjyAv
メモ

(0)→(0)(1)などの写像を使って展開することでより抽象的な表現、より強力なシステムが可能になると思われる
10で既出のアイディアだがな
152 :
2016/01/06(水) 18:58:19.21 ID:PF6AJWdp
>149 そのM,Nについてはそれぞれ0列から始まるという条件が抜けています。よく精査しないまま投稿したもので、すみません

(n)# #は(n+1)より小さくない列からなる行列
(n)$ $は(n)以上の列からなる行列(注、計算上の大小関係とは別、1行目の値のみに注目する)

(0)は自然数型
(0)=1 (0)(0)=2 (0)(0)(0)=3 ...

型(n+1)は型(n)の集合をあらわす型。(n)#(n+1)で型(n)の値は集合[(n)#(n+1)]={(n)#,(n)#(n)#,(n)#(n)#(n)#,...}のなかで全称量化される。
(n)#(n+1)(n+1)なら{(n)#(n+1),(n)#(n+1)(n)#(n+1),(n)#(n+1)(n)#(n+1)(n)#(n+1),...}のなかで??すでに全称量化された型(n)の自由な値をさらに
全称量化する。つまり全称量化された「全称量化された(n)#」

わかりやすく(?)言えば
ある範囲で任意の値をとりうる自由変数xに依存する変数yを、ある集合の中で任意の値をとるものする。
この場合、ある集合とは{x,x+x,x+x+x,...}
xの具体的な値は、計算途中でほとんどの場合変わることに注意、順序数を展開するうえでのωのようなもの。

順序数に当てはめると、
(1)は次数を表す型、(1)で次数1、(1)(1)で次数2、・・・
次数1は次数0の順序数の集合、次数2は次数1の集合、・・・それぞれの集合は一つ手前の順序数からなる収束列となっている。
(2)は次数の次数、(3)は次数の次数の次数、・・・をあらわす型


(0)(1)(1)(0)(1)=ω^2+ω (0)(1)(1)より大きい最初の次数1の順序数

媒介変数は自然数のため、計算する集合はせいぜい有限個
有限のステップで、有限個の集合のうち任意の集合の、有限番目の要素は計算を終え、計算はひとつ前の要素へ移るため、有限のステップで
1行の行列は計算を終える。
153 :
2016/01/06(水) 19:00:24.00 ID:PF6AJWdp
述語論理とのつながり

とくに断りがない場合、一階部分の量化の範囲はすべての自然数の集合(0を含む)

∀m[m<α⇔α∈A_0]∧∀n[∀m(m∈A_0)∧?n=0∧?m+n=α⇔α∈A_1]∧∀n[∀l∀m(l,m∈A_n)∧l>m∧?l+m=α⇔α∈A_{n+1}]

二階述語論理は集合を量化する、つまり一階の述語を量化することが可能。
算術的内包公理は、算術による述語を項として扱うことができます。
帰納法をつかってA_ωを定義したり、算術による述語を算術による述語で評価したり。
前提とする公理系によって明示されていればかまいませんが、A_αがwell-orderdな集合であることを示しておく必要があります。

(0)#(1)は集合[(0)#(1)]のなかで全称量化された数変数。(1)#(2)は集合[(1)#(2)]の中で全称量化された集合変数

集合論的には
(0)$は数(順序型)、[(0)#(1)]はACA_0で構成可能な数の集合、[(1)#(2)]はACA_0で構成可能な「ACA_0で構成可能な数の集合」の集合、・・・

nは任意の自然数
集合[(n)#(n+1)]={(n)#,(n)#(n)#,(n)#(n)#(n)#,...}はすべての自然数からなる集合と同型(関係を保存する自然数からの全単写が存在する。)
展開することで集合が「分解されて消え」、その要素が残る。比喩をつかった表現だけど、正式な言い回しがわからん
すべての要素を使い果たしたら、一つ上の、一つ手前の集合へと切り替える。一つ手前の集合がなければ見つかるまでさらに一つ上へ、それでも見つからなければ
その部分の計算は終了
どの要素も有限回下降し続けることで0から写される最小値へとたどり着く。そして集合のタワーも無限の高さに達しない。証明はまかせる
つまり、一回のステップでその要素の集合が分解され、有限回のステップでその一つ手前の要素へと移動し、有限回の移動ですべての要素を消費し、一つ上の集合
へと昇華し、有限回の昇華で頂上へとたどり着き、それからあとは有限回の移動で頂上の最小値へと移り、有限の高さから後は一番下まで下降して終わり。計算終了
154 :
2016/01/06(水) 19:05:49.49 ID:PF6AJWdp
2行

(1,1)、2行目の1は一階の型を全称量化する型、型演算、型の型、etc...



[(0,0)(1,1)]={(0),(0)(1),(0)(1)(2),...}

計算過程で型の一階部分が全称量化され、後ろに階段状につなげられる。
集合論の視点からみると、悪い部分がΠ^1_1-CA_0の強さを持った集合の中で全称量化され、その媒介変数t番目の要素までを後ろに階段状につなげていき、
ACA_0の集合タワーができる、もちろんtは自然数なので無限の高さにはならない。

論理式 簡潔に要点だけ
∀C∀B∀A[(A∈C∧(P(A)⇔A∈B)⇒B∈C)⇔B∈A^1] Pは算術による述語
算術による述語を全称量化してΠ^1_1

A^1は二階部分が(1)の型

1行の計算に一階の集合タワーの展開が追加される。有限の高さをもつ一階の集合タワーの計算が終了するということはすでに証明済みなので、ちょっと端折るけど、
2行目が1より大きくならない2行の行列は計算が終了する。
2行目がnより大きくならない2行の行列は計算が終了すると仮定する。2行目がn+1となるとき、2行目がnとなる列、二階部分がnの型をΠ^1_1の強さをもつ述語により
全称量化する。仮定より、これにより構成される行列は計算が終了する。よって2行目がn+1より大きくならない2行の行列も計算が終了する。

3行目以降も論理および型の階層を上げながら同様に証明していく。
n行でn階述語論理のある体系(n階のカインド)を対角化した強さとなる。
155 :
2016/01/06(水) 19:07:26.72 ID:PF6AJWdp
補足

この論証方法では、(0)(1)(1)と(1)(2)は順序関係∈について全順序となっていない。正確には、この論証方法で定義される異なる全順序集合間の順序関係を
定義することができない。よってバシク行列全体の構造は全順序集合とはならない、ということになる。
ついでに言えばA_nの要素がA_{n+1}の要素となっていないので推移律も成り立っていない。
ただこれは証明のために定義した構造と関係∈について言えることで、計算順序については全順序関係が成り立つ。

これ以降の拡張により、二次元目と三次元目の間に展開される標準形は、「カインドの高さを全称量化する型」の高さを全称量化した強さを持ち、
三次元目と四次元目の間では「「カインドの高さを全称量化する型」の高さを全称量化する型」を全称量化した強さを持ち、以降も同様。
(多次元配列を見直そうかという件については、今後の拡張のことを考えてこのままでいいと判断しました。)
レギオン配列で型(n)の値を次元およびその全称量化に関する値として扱うことも可能(ポリモーフィズム)
一般的には、「数の区切り」という大きな型のなかで、(0)は次元をつかさどる値へと決定されれ、(1)は(0)のレギオン空間の区切りとなり、・・・
多相型を全称量化する型、それよりも大きな多相型、・・・L2空間の中で多相型の大きさが全称量化される。

・・・このままBEAFを応用していってもλωのなかでアルゴリズムが組み立てられていくばかりで依存型がそろわない。
よってCoCには届かない。一から新しいのつくしかないね
156 :
2016/01/06(水) 19:16:31.95 ID:xGvP5bGV
巨大数プログラムを書きやすい言語とか作ったら面白いかな。
具体的には2重リストアッカーマンをきれいに実装できる言語がいい。
157 :
2016/01/07(木) 00:24:56.67 ID:th/7jpLN
>>154
「n階述語論理のある体系(n階のカインド)を対角化した強さ」を表記する順序数はある?
1階のカインドがε_0で2階のカインドがϑ(Ω_ω)になるという計算は妥当?
158 :
2016/01/07(木) 18:32:54.56 ID:kIXECDuX
ここのコメント欄で、Deedlit11 さんからのコメントが来た
http://googology.wikia.com/wiki/User_blog:Nayuta_Ito/Introduction_To_Bashicu_Matrix_System

ここにもコメントしていたし、ずっと興味を持ってくれている模様
http://ja.googology.wikia.com/wiki/%E3%83%90%E3%82%B7%E3%82%AF%E8%A1%8C%E5%88%97%E3%82%B7%E3%82%B9%E3%83%86%E3%83%A0
159 :
2016/01/07(木) 22:38:33.52 ID:weRUT6Lc
>157
型とかカインドとかいうのは計算を支援するシステムなのでそれ自体には固有の強さというのはありません。
プログラムのの強さを測る尺度とイメージすればいいかも
これを利用した体系の強さをあらわす順序数はあるにはありますが、このレベルになると順序数で評価しようとする対応があまり意味をなさないと思います。
順序数を表現するための背景にある体系、より抽象的には論理そのものを議論していくことになるので。


証明自体はあってると思うけど言葉の使い方までは保証しかねる。まずい表現があれば誰かが突っ込んでくれるだろう。
3行の強さの評価は順序数と巨大基数の表現力では難しいというか、Z_2の体系では(最初の基礎となっている公理系を
めちゃくちゃ広げたりでもしない限り)推し量れません。

(0,0,0)からの可算順序数に変換しようかと思ったけど、定義上ψ関数みたいに因数の上限だけを記しておくような書き方が
許されてるのか怪しいので、(1,1,1)の強さを評価している部分はそのままにしておきます。
160 :
2016/01/07(木) 22:39:16.80 ID:weRUT6Lc
(0,0,0)(1,1,1)(2,2,1)(3,2,0)(2,2,1)=ψ(ψ_I(ω))
(0,0,0)(1,1,1)(2,2,1)(3,2,0)(2,2,1)(3,2,0)=ψ(ψ_I(I))
(0,0,0)(1,1,1)(2,2,1)(3,2,0)(4,3,1)(5,4,1)(6,4,0)=ψ(ψ_I_2(0))
(0,0,0)(1,1,1)(2,2,1)(3,2,1)=ψ(ψ_I_ω(0))
(0,0,0)(1,1,1)(2,2,1)(3,2,1)(2,2,1)=ψ(ψ_I_{ω^2}(0))
(0,0,0)(1,1,1)(2,2,1)(3,2,1)(2,2,1)(3,2,0)=ψ(ψ_{χ(1,0)}(0))
(0,0,0)(1,1,1)(2,2,1)(3,2,1)(2,2,1)(3,2,1)=ψ(ψ_{χ(1,ω)}(0))

(1,1,1)(2,2,1)(3,2,1)(3,2,1)=C(Ω^2+1,0)
(1,1,1)(2,2,1)(3,2,1)(4,2,1)=C(Ω^Ω+1,0)
(1,1,1)(2,2,1)(3,3,0)=C(ε_{Ω+1},0)=C(C(C(C(Ω_2,Ω),0),0),0)
(1,1,1)(2,2,1)(3,3,0)(4,4,1)=C(C(C(Ω_2,Ω)+1,0),0)
(1,1,1)(2,2,1)(3,3,0)(4,4,1)(5,5,1)(6,6,0)=C(C(ε_{C(Ω_2,Ω)+1},0),0)
(1,1,1)(2,2,1)(3,3,1)=C(Ω_2+1,0)
(1,1,1)(2,2,1)(3,3,1)(2,2,1)(3,2,1)=C(Ω_2*2+1,0)
(1,1,1)(2,2,1)(3,3,1)(2,2,1)(3,3,0)=C(ε_{Ω_2+1},0)=C(C(C(C(Ω_3,Ω_2),0),0),0)
(1,1,1)(2,2,1)(3,3,1)(2,2,1)(3,3,0)(4,4,1)=C(C(C(Ω_3,Ω_2)+1,0),0),0)
(1,1,1)(2,2,1)(3,3,1)(2,2,1)(3,3,0)(4,4,1)(5,5,1)(6,6,1)=C(C(Ω_3+1,0),0)
(1,1,1)(2,2,1)(3,3,1)(2,2,1)(3,3,1)="Limit of Taranovsky's C"
161 :
2016/01/07(木) 22:48:24.25 ID:weRUT6Lc
需要があるみたいなので簡単な大一次数列だけ解説する。
原始数列に(m)(m+n+1)=(m)(m+n)(m+n*2)...というルールを追加する。それだけ
162 :
2016/01/08(金) 00:04:52.52 ID:KJozq+4B
2重リストアッカーマン思ったより凶悪に増加するっぽい。
甘く見てた。
163 :
2016/01/08(金) 00:51:56.21 ID:KJozq+4B
2重リストアッカーマンって計算過程で実質多重リストの入れ子構造になるよね?
164 :
2016/01/08(金) 12:56:05.34 ID:nzufgM6e
>>159
なるほど。完璧に理解したわけではありませんが、大体の雰囲気は分かりました。
とりあえず、バシク行列の計算が終了するということはほぼ間違いなさそうなので、
英語版ページの作成を始めようと思います。
165 :
2016/01/08(金) 18:51:50.01 ID:ozXuxuUK
記事の作成ありがとうございます。
早速すみませんが4行の(0,0,0,0)(1,1,1,1)の解析結果は>>120を投稿したころより誤りだということが判明したため削除をお願いします。
バシク行列の解析のブログ記事も、引用されたころより更新されているのでそれにあわせたほうがいいかと。
おそらくv関数の定義もまだ固まっていないとも思われるので。
166 :
2016/01/08(金) 19:10:33.16 ID:nzufgM6e
Wiki なので、比較的誰でも更新できると思います。適宜、やっちゃってください。
日本語版のページは、とりあえず検証途中のものをどんどん載せていくという方針で
やっていましたが、英語版には確定的な情報を載せます。
167 :
2016/01/08(金) 22:24:35.82 ID:KJozq+4B
http://textuploader.com/5pm0v

2重リストアッカーマンのプログラム書いてみました。
Rubyでegisonというgemを使ってます。

動作が正しいかどうか誰かチェックしてくれませんか〜
168 :
2016/01/09(土) 01:32:50.38 ID:n1twDky8
英語ページできました
http://googology.wikia.com/wiki/Bashicu_matrix_system

英語表現でおかしなところはむこうの人たちが直してくれると思うので、
解析におかしなところがあれば分かる人が直してください。
169 :
132人目の素数さん
2016/01/09(土) 12:19:49.56 ID:M2HjVa1w
>>168乙でした
170 :
2016/01/09(土) 15:16:12.41 ID:G5NkDZy0
色々と編集されているので、しばらく様子を見て、編集が落ち着いたら日本語版を英語版にあわせて書き直します。
171 :
2016/01/09(土) 17:50:35.70 ID:G5NkDZy0
http://googology.wikia.com/wiki/Talk:Bashicu_matrix_system

n階述語論理の「ある体系」って何?という質問が来ています。
とりあえず理解している範囲で答えておきましたが、何とも曖昧なので、できればきちんと
わかっている人が答えてください。
172 :
2016/01/10(日) 23:01:40.68 ID:vJEauuUK
論証体系を順序型から始めて新しく構築する。そのほうが分かりやすい気がする。今回一階部分が自然数の集合という前提は無し。
前回雑になってた部分も丁寧にする方針。

(0)は順序型、(1)は(0)の次数、(2)は(1)の次数、・・・
(n)#(n)#で値の足し算

後者関数の定義
∀A∀x∀y{x,y∈A⇒x<s_A(x)∧x<y→s_A(x)≦y]

加法の定義
∀a∀b[a+0=a∧s(a+b)=a+s(b)]

内包公理による集合の定義 超限帰納法をつかって媒介変数t∈A_0を添える。
∀t∀a∀x[t∈A_0∧a,x∈A_t∧?s_A_t(x)=a⇔a∈A_{t+1}]
∀t∃t'∀a∀x[t,t'∈A_0∧t<t'∧a,x∈A_t∧t<t'?s_A_t(x)=a⇔a∈A_t']
tは集合を抽象化するための媒介変数、証明するうえで次数と一致するとみなしていい⇒tをもつ任意の集合Tと同型の集合を構成することができる。
たとえば、T=A_0として、P_A_0(t)⇔P_A_A_0(A_t) かつ F_A_0(s)=t⇔F_A_A_0(A_s)=A_t Pは任意の述語、Fは任意の関数

集合の集合((2)の機能にあたる)をつくる。
∀t[t∈A_0⇔A_t∈A_A_0]∧
∀t∀a∀x[t∈A_0∧A_a,A_x∈A_A_t∧?s_A_A_t(A_x)=A_a⇔A_a∈A_A_{t+1}]
∀t∃t'∀a∀x[t,t'∈A_0∧t<t'∧A_a,A_x∈A_A_t∧?s_A_A_t(A_x)=a⇔a∈A_A_t']

1行の行列との対応

(0)$ A_0の要素、(1)$ A_A_0の要素、(2)$ A_A_A_0の要素

(0)=1 (0)(0)=2 (0)(0)(0)=3 ...
(1)=A_1 (1)(1)=A_2 (1)(1)(1)=A_3 ...
(0)(1)(0)(1)=ω*2 A_1の2番目の要素となるA_0の要素
(0)(1)(1)(0)(1)(0)(1)=ω^2+ω*2 A_2の最初の要素から数えたA_1の2番目の要素となるA_0の要素
(1)(2)=A_ω A_A_1の最初の要素となるA_A_0の要素
(0)(1)(2)(1)=ω^ω A_A_1の最初の要素から数えたA_A_0の最初の要素となる集合の最初の要素となるA_0の要素

(3),(4),(5),...についても同様。

後は完成してからどこかに纏めてあげてしまおう
173 :
2016/01/11(月) 01:46:40.71 ID:ysbFin6c
> 後は完成してからどこかに纏めてあげてしまおう

できれば英語版もお願いします(ゆっくりで構わないので)
174 :
2016/01/11(月) 01:57:36.86 ID:ysbFin6c
日英両方作るのが面倒だったら、むしろ英語版だけでも
175 :
132人目の素数さん
2016/01/13(水) 19:15:05.74 ID:wwxvM0Kk
☆ 日本の核武装は早急に必須ですわ。☆
総務省の『憲法改正国民投票法』、でググってみてください。
日本国民の皆様方、2016年7月の『第24回 参議院選挙』で、日本人の悲願である
改憲の成就が決まります。皆様方、必ず投票に自ら足を運んでください。お願い致します。
176 :
2016/01/15(金) 23:16:28.59 ID:RgPutaxM
wiki,式の展開の様子が分かるプログラムも載せてくれると嬉しい。
177 :
2016/01/16(土) 23:57:12.90 ID:f2JNdYhG
何を言ってるのかわからないけど、wikiなんだから編集したら?
178 :
2016/01/17(日) 21:20:54.35 ID:w2MBjmhO
勝手に編集していいもんなの?よく知らんけど。
179 :
132人目の素数さん
2016/01/17(日) 22:42:36.11 ID:XfpCffao
ラヨ関数について分析していたら、ラヨ関数は実は通説よりも小さい関数なのではないかという考えに至ったため、
どなたか検証をお願いします。
詳しいことは以下のpdfに記載してあります。
https://drive.google.com/file/d/0B93hxCF3d6JUWlUzeDV4QlBXQUE/view?usp=sharing

要約すると、
・モデルのとり方によって真偽が異なる論理式を扱うことはできない。
・ゲーデルの完全性定理から、ラヨ関数はチューリング次数0''の神託機械で計算可能。
・ふぃっしゅ数ver7は、大きく見積もってもふぃっしゅ数ver4と同程度である。
となります。
180 :
2016/01/18(月) 00:11:59.93 ID:fWHdtP7/
>>178
Wiki というのは、そういうもの。トップページの「手伝う」に方法は書いてある。
181 :
132人目の素数さん
2016/01/18(月) 09:03:40.35 ID:+tJ0vSmq
>>179です。
さっそく間違いを見つけたので訂正します。
p13の「そもそもの目的であった論理式の〜」のところに、
∧ω∈B∧P(ω)∈B
を付け加えてください。
182 :
2016/01/18(月) 17:54:31.26 ID:sU5gmt8i
編集しました。
2重リストアッカーマンのプログラムを載せました。
問題あったら教えてください。
183 :
2016/01/18(月) 22:36:43.74 ID:OfUMG9KM
ラヨ関数の定義に特定の公理系による制限ってあったっけ
184 :
132人目の素数さん
2016/01/24(日) 09:27:19.11 ID:v0G3jWAc
計算不可能な関数で表現される巨大な数

計算可能な関数で表現される巨大な数
の間ってどうなってるの?
185 :
2016/01/24(日) 14:57:24.35 ID:0/wuvwru
「計算可能な関数で表現される巨大な数」の上限がビジービーバー関数Σ(n)。
nをいくつにするかで上限が決まるけど、それは恣意的なものになる。

計算不可能なビジービーバー関数Σ(n)で表されるΣ(1000)という数は、
1000状態-2記号のチューリングマシンで書けるので、そのように書けば計算可能な
関数で表現できたことになる。
のだけれど、それがどういう関数なのかはわからない、ということ。
186 :
2016/01/24(日) 15:04:13.35 ID:0/wuvwru
仮に「計算可能な関数で表現される巨大な数」の上限をΣ(1000)と定めたとして。
「計算可能な関数で表現される巨大な数」を実際に定義すれば、それはΣ(1000)よりも小さいだろう。
実際に定義される数は人間の計算能力や証明能力に依存して大きくなる。

実際に人間が定義する上限は「人間の能力」に依存するものなので、数学的に一意に決まっているものではない。
「最大の素数」はないけれど「最大の知られている素数」はある、というのと同じ関係かな。
187 :
2016/01/24(日) 15:25:41.03 ID:WEowCeNN
数学的に間ってなんですか?

>>179
そのモデルをFOSTで定義して真偽を決定できるんじゃ
別にラヨ関数の定義にZFCは用いられてないというか、そもそもZFCを記述する言語を対角化しているわけで、
どうも抽象の次元を捉え違えている気がする。
こちらの勘違いでしたら失礼
188 :
132人目の素数さん
2016/01/24(日) 20:35:07.33 ID:8h3fFG2S
>>185>>186
ふーむ…。まだ理解が浅いみたい
もう少しいろいろ思考を重ねてみるよ。ありがとう

>>187
それが厳密にわかってたら、そもそもこんなあほな質問もしてないと思うんだ多分
189 :
132人目の素数さん
2016/01/25(月) 16:45:15.79 ID:8Yku2p4B
>>187
FOST自体、First Order Set Theory(一階集合論)の略ですから、
集合論を含むと考えるのが自然だと思われます。そしてZFC抜きには、∈記号の
意味が定義されません。ラヨ関数の定義にあたって∈記号が出てくるのですから、
この記号はZFCでの意味ととらえることを前提としていると考えられます。
したがって、ラヨ関数がZFCを用いずに定義されているとは考えられません。
190 :
2016/01/25(月) 20:05:26.54 ID:TMZu+8vF
∈記号の意味を定義するのにZFCに限らなくてもいいし、
集合論にもZFCに巨大基数の存在を追加して拡張したものもある。まだ勉強不足だから深く突っ込んだことは言えないけど。
ラヨ関数でいうところの集合論ってのはもっと広い意味を持ってるんだと思う。

第一∈記号の意味を定義するとはどういうことか、
・・・自分への宿題としておきます。
191 :
2016/01/25(月) 20:31:11.52 ID:TMZu+8vF
言葉の整理
体系 ZFCとかCoCとかいうやつ。真理値を決定するための前提条件。公理系。強力であればあるほどさまざまな命題の真理値を決定することができる。しかし強すぎると矛盾してしまう。
ゲーデルの不完全定理により無矛盾でありながら任意の命題を決定できる体系は存在しない。無矛盾であれば、これで作られたプログラムは厳密にひとつの数を定義することができる、
ある変数設定がaであると同時にaではない、ということにはならないし、表現力が強ければ強いほどより抽象的な概念を決定することができるようになる。

論理 体系を定義する形而上の材料。具体的には言語。矛盾するものも含めて、文字数が長ければ長いほど、また高階述語論理やOodle theoryのようにより抽象的な論理であればあるほど
強力な体系を構築できるようになる。
192 :
2016/01/31(日) 02:22:08.30 ID:S0mqWxRS
>>184
計算不可能な巨大数でも適当な計算可能な方法で表現することができます。
というかいかなる巨大数も1+1+...+1で表現することができます。ほとんど現実的に不可能というだけで
当たり前のことですが一応確認

C言語でもなんでもかまいませんが、任意の計算可能な関数を定義できる言語を用意します。
n文字のその言語で表現可能な最大の数をBB(n)とすれば、BBはソフトウェア的に解釈した
ビジービーバー関数となります。
BB(x)が計算可能であれば、その言語で有限の最低限の文字数aで定義できてしまいますが、ここでBB(a+4)について考えてみます。
細かい話は抜きにして、「a+4」は「x」よりも2文字多いのでBB(a+4)を
出力するプログラムはa+2だけの文字数を必要とします。
さらに「BB(a+4)+1」はa+4だけの文字数を必要とします。
しかし、これはBB(a+4)がa+4の文字数で表現できる最大の数であることと矛盾しています。
よってBB(x)は計算することができません。
そしてBB(513)だけでローダー数などを軽々と飛び越えてしまったりします。

感覚をつかめましたか?
193 :
2016/01/31(日) 02:51:48.78 ID:S0mqWxRS
>>189
最初から∈記号の意味が定義されているからこそラヨ関数が定義される、というのが誤解のもとだと思います。
最初から∈記号の意味が定義されているなんらかの体系、この場合はZFCを対角化するのではなく、
体系を構築する論理(言語)を対角化するというのがラヨ関数の強さの秘訣でしょう。

「意味が定義される」というのを、「その述語に関するある命題で真理値が決定される」と捉えます。
すると、
∀a[?a∈b]
というbに関する変数設定だけでも、「c∈bは成り立たない」などという意味が与えられます。
194 :
132人目の素数さん
2016/02/01(月) 18:12:35.02 ID:LcI7btvt
>>193
巨大数wikiのラヨ数の記事で、論理式で単一の自然数を指し示す具体的な例がのっています。
例えば、?∃2(2∈1)は1に入るものが空集合しかありえず、空集合を順序数として捉えると
0になるから、この論理式は0を指し示す、というようなことが書いてあります。
なぜ1に代入するとこの論理式が真になるようなものが存在していると断言できるかというと、
空集合の公理∃t∀x(?x∈t)が空集合の存在を保証しているからです。
なぜ1に入るものが空集合しかありえないかというと、
外延性の公理∀x∀y∀t(x=y⇔(t∈x⇔t∈y))が、要素を持たない集合が
いくらあったとしても、それらはすべて=で結べるため区別する必要がないこと
を保証しているからです。
?∃2(2∈1)が0を指し示す論理式であると保証するためにもZFCから公理を借りてくる
必要があります。
ラヨ関数がZFCの対角化かどうか置いておくとしても、ZFCを(その中の外延性の公理や
空集合の公理まで)一切前提としないで定義されてる、とも言えないと思います。

ラヨ関数が何らかの集合論の公理のもとに定義されていると考えるなら、
確かに集合論にもZFC,NBG,ZFC+連続体仮説,色々ありますが、ただ単に集合論と
言ったら、それはZFCのことを指す、と考えるのが自然です。
また、ZFCには分出公理があるため、一階述語論理では「有限の」論理式で
ZFCを記述することはできません。そのため、n文字以下の論理式でZFCは記述できません。
195 :
2016/02/06(土) 20:31:03.49 ID:oIQHTW2U
ひとまず確認
私はラヨ命名可能な数を構築する上での任意の論理式は無条件で真であることを前提としていると
解釈しています。よってZFC上で空集合の存在が保証されていなくても、
「任意のaはbの要素ではない」
とラヨ命名の仮定で言われればこれは証明抜きで真であると受け入れなければならないと考えています。
196 :
2016/02/06(土) 21:20:10.31 ID:oIQHTW2U
?∃2(2∈1)が成り立つモデルにかかわらず、たとえ空集合の区別がついてしまうモデルでも、
形式的に「0」を定義することは可能です。そしてそれから推移的に、そして形式的に、
それが成り立つ具体的なモデルではなく、その抽象的な条件のみを抽出して「1」、「2」・・・を
定義していくことがラヨ命名だと考えています。
集合論をZFC限定してしまうと、フォン・ノイマン宇宙でなくグロタンディーク宇宙を対角化した強さになってしまうと
思うので、やはりそのように考えることには抵抗があります。

分出公理では論理式(一階の述語)の量化がされていないので、一階述語論理で記述できると思います。
恥ずかしながら最近まで論理と算術をごっちゃにして盛大に勘違いしてましたが
197 :
132人目の素数さん
2016/02/07(日) 17:08:15.28 ID:Zepty6fv
>>195
任意の論理式が無条件で真であることを前提としている、とありますが、
∃1(?1=1)や、∃1?∃2(1∈2)というような、普通には真になりえないと考えられるもの
まで真であるようなときはどうなるのでしょうか。
>>196
すみません。「?∃2(2∈1)が成り立つモデルにかかわらず、たとえ空集合の区別がついてしまうモデルでも、
形式的に「0」を定義することは可能です。そしてそれから推移的に、そして形式的に、
それが成り立つ具体的なモデルではなく、その抽象的な条件のみを抽出して「1」、「2」・・・を
定義していく」というのが、具体的にどうすればいいのか分かりません。
>>179の前書きには、ラヨ関数の定義には曖昧さがあるのではないかと考え、
これに具体的な定義を与えようとした結果小さいという結論に至った、という経緯を書いてあります。
何だか違和感がある、というお気持ちは十分に伝わりましたから、ラヨ関数は具体的にどんな関数で、
>>179にある定義にはどこに問題があってどう修正しなければならないかを明示して頂けると幸いです。
そして、分出公理が一階述語論理で記述できないとはどこにも書いていません。
ただ、一階述語論理で記述しようとすれば必ず無限に長い論理式になる、と書いています。
198 :
2016/02/07(日) 19:25:49.04 ID:bgYWKt2T
明らかに矛盾が導かれる論理式はビジービーバー関数で計算が終了しないプログラムを
除くのと同じ要領で、無視すればよいと思います。

分出公理は、一階の述語を量化した二階述語論理と同じ厳密な表現を目指せば
無限に長くなりますが、そこまで厳密さを要求しなければ有限の長さで記述できます。
というかwikipediaなどに書かれてあるあの論理式がそのまま一階の式となっています。

他は長くなると思うのでしばらくお時間をください。

新しくブログ記事を作りましょうか、
いろいろと曖昧だった部分を勉強しなおしている最中で、バシク行列の証明も最近
ほったらかしで申し訳ない状態がつづいてますが
199 :
132人目の素数さん
2016/02/07(日) 22:23:15.16 ID:Zepty6fv
一応、公理の部分も変数化した場合の考察と、高階述語論理やBIGFOOTについての
考察をしたものを>>179の続編として作りましたので、よろしかったらご覧になってください。
https://drive.google.com/file/d/0B93hxCF3d6JUbEdMeTlNNGFWMjQ/view?usp=sharing
200 :
132人目の素数さん
2016/02/07(日) 22:54:53.99 ID:Zepty6fv
>>198
>明らかに矛盾が導かれる式は
>>197の∃1(?1=1)は∀x(x=x)という前提があるからこそ否定できるもので、
∀x(x=x)という公理か、あるいは外延性の公理かその他を導入しないと別に
矛盾は導かれません。
∃1?∃2(1∈2)という式にしたって、これを否定できるのは対の公理があるからで、
ZFCを前提としない立場では別に矛盾は導かれません。
分出公理は、wikipediaの書き方にそって言うなら、任意の論理式Ψ(x)について〜
という書き方になっているので、すべての論理式というのは無限個ある訳ですから、
分出公理を正確に書こうとすれば無限個の論理式になるか、∧でつないで単一の論理式
にするなら無限に長いものになります。一階述語論理は論理式の量化をしない以上、
∀Ψ(x)〜という書き方はできないのですから、分出公理は有限には書けない、とすべきだと思います。
あと、言い忘れていましたが、>>196の「?∃2(2∈1)が成り立つモデルにかかわらず」
のところについて、この論理式の1は量化されてない自由変数ですから、この論理式は
閉論理式ではありません。自由変数を含んでいる式の真偽を考えても仕方がないので、
∀1?∃2(2∈1)か∃1?∃2(2∈1)としないと意味が通りません。
201 :
132人目の素数さん
2016/02/14(日) 12:04:31.42 ID:AEncE0am
ローダーのD関数がH[Ψ(Ψ_{α→I_α}(0))]より大きいとされた理由はどこのページで見られますか。
202 :
2016/02/15(月) 16:51:38.35 ID:zNexzu1h
>>200
返信遅くなってすみません。

https://ja.wikipedia.org/wiki/%E5%85%AC%E7%90%86%E7%9A%84%E9%9B%86%E5%90%88%E8%AB%96
私の知ってるwikipediaの記事では
>この公理は、論理式 ψ をパラメータとする公理図式である。
と書かれてあるだけで「任意の」とまでは書かれていませんが・・・

∃1(?1=1)だけでは明らかに矛盾を導き出せなくても∃1(?1=1)∧∀x(x=x)
とすれば矛盾します。
といいますか、ラヨ関数は前提を記述する言語を対角化しているので、その前提から矛盾が導出されるのであれば
論理式に組み込んで矛盾する論理式の例とすればよいと思います。

?∃2(2∈1)の1は媒介変数くらいに捉えてください。
具体的に「ある公理系で定義されたある構造における定数」と捉えてもらってもかまいません。
203 :
2016/02/16(火) 21:03:10.30 ID:pjy9cf+4
∃1(?1=1)は(普遍的な前提条件とする)論理の公理に反するのでその時点で矛盾とみなしてもいいですね。
もしかしたら著者によってどこまでを大前提とするかが違うのかもしれませんが。

>∃1?∃2(1∈2)という式にしたって、これを否定できるのは対の公理があるからで、
>ZFCを前提としない立場では別に矛盾は導かれません。

これは最低限?(∃1?∃2(1∈2))を証明できる公理系であれば矛盾するので、
ZFCを前提としない〜というのは言いすぎだと思います。

そして>>179の論文について
同じ変数記号を2回以上束縛することはできないとありますが、その理由を教えていただけないでしょうか?
カット除去の定理などにより、その最大の順序数ならともかく、束縛する回数を数えること自体にあまり意味が
あるとは思えません。
定数記号も(意味があるのかどうかはおいといて)別に束縛してもいいと思いますし、
「x+y=2ならば1+1=2」
というような、x、yの値に関係なく真となる恒真命題もつくれます。
(定数を束縛するという表現自体なにか間違ってるような気がしますが)
このように変数の量化に関係なく真偽を議論できる場合もあるでしょう。
完全性定理により任意の一階の恒真な論理式は一階述語論理で導出できることが保証されていますし。

完全性定理に関することも含めていろいろありますが、後はまとめて後日・・・
204 :
2016/02/16(火) 21:59:20.13 ID:pjy9cf+4
205 :
132人目の素数さん
2016/02/17(水) 21:11:21.05 ID:CwRTlhiw
>>202
>「任意の」とまでは
任意の集合 X と論理式 ψ(x) に対して、X の要素 x で ψ(x) をみたすような x 全体の集合が存在する:
「任意の集合Xと論理式Ψ(x)」の「任意の」という修飾語が「集合」にだけかかっていて「論理式」には
かかっていない、と解釈するか両方にかかっていると解釈するかは日本語の問題ですが、
後者と考えるのが自然かと考えます。
英語版のAxiom schema of specificationには、
Note that there is one axiom for every such predicate φとあります。参考までに。

>∃1(?1=1)だけでは明らかに矛盾を導き出せなくても∃1(?1=1)∧∀x(x=x)
とすれば矛盾します。
少なくとも>>198の時点では、∀x(x=x)という公理があるとは聞いてませんでした。

>?∃2(2∈1)の1は媒介変数くらいに捉えてください。
私の知る限り、一階述語論理に自由変数や束縛変数という言葉はあっても、媒介変数という言葉が出てきたことはありません。
一階述語論理ではなく数?などで出てくることは知っています。
206 :
132人目の素数さん
2016/02/17(水) 21:43:50.85 ID:CwRTlhiw
>>203
>これは最低限?(∃1?∃2(1∈2))を証明できる公理系であれば矛盾するので、
ZFCを前提としない〜というのは言いすぎだと思います。
これは確かにその通りですが、>>198の時点では、ZFCではない何か別の公理が示されていなかったので、
「明らかに矛盾が導かれる論理式」ではないはずだ、と反論したつもりです。

>同じ変数記号を2回以上束縛することはできない
これは例えば、∀x?∃y∃x(x + y = 2)のようにxが複数回束縛されている論理式について
考えることに意味が無いからです。∀x∃y(x + y > 1)∧∃x∀y(x + y = y)というように、
同じ名前の変数記号を使っていても別に紛らわしくならないところについては普通に変数記号を
使いまわせます。説明が足りなかったかもしれません。

>定数記号も(意味があるのかどうかはおいといて)別に束縛してもいい
単純に、定数記号を束縛することに意味が無いから禁止しています。
定数が束縛されているという、謎な論理式の作成を許容しても扱いに困るだけです。

>「x+y=2ならば1+1=2」というような、x、yの値に関係なく真となる恒真命題もつくれます。
x,yの値に関係なく成り立つなら、x+y=2→1+1=2ではなくて、∀x∀y(x+y=2→1+1=2)と書けばよいでしょう。
量化記号を省略する意味も別にありません。
207 :
132人目の素数さん
2016/02/17(水) 22:24:57.69 ID:CwRTlhiw
おそらく一番の問題は、主張されている「一階述語論理の対角化としてのラヨ関数」というのが
どんなものなのか、そこでは?2(2∈1)が0を指すと言えるのかどうかさえも私には分からず、
したがってそれを受け入れることもそれに反論することもできないというところではないでしょうか。

計算可能関数なら計算手順を示せば誤解の余地なく定義できる一方で、
計算不能関数を曖昧さが残らないように定義するのは容易ではありません。
少なくとも、ここで議論が起きるくらいラヨ関数の定義には不確かさがあるのですから、
主張されている「>>179とは違う正しいラヨ関数の定義」というのが明確にならないと
議論は平行線を辿るばかりです。

それに対して>>179にあるラヨ関数の定義はかなり具体的です。
何せ、(結果論的に)0''の神託機械で計算する手順まで示せたのですから。
208 :
132人目の素数さん
2016/02/17(水) 22:32:25.61 ID:CwRTlhiw
まだ「>>179とは違う正しいラヨ関数の定義」は人に明確に説明できるようになるまで
完成していないのではないでしょうか。それなら、ラヨ関数の解釈について、
>>179の対立候補となれるまで作り上げて、その定義に曖昧さが無さそうだと言えるようにして、
そして可能ならそれがビジービーバー関数より大きいという証明を付けて、
(0'の神託機械で計算できない、というだけでは不十分です。増加度の遅い
計算不能関数の可能性もあります。また、>>179ではビジービーバー関数との大きさの
比較は行ってません。)
それがどこかに公開されるまで議論は止めにしましょう。
209 :
132人目の素数さん
2016/02/17(水) 22:36:48.36 ID:CwRTlhiw
訂正
>>207
?2(2∈1) → ?∃2(2∈1)
210 :
2016/02/19(金) 19:25:26.23 ID:2+0y1c4N
総括すると、具体化したために本来の定義よりも弱くなってしまっているとおもうのです。
扱いに困るとかいう理由で安易に制限をもうける考え方はまずいと思います。
思わぬところに落とし穴があるかもしれませんし、今は意味がなくても今後
の拡張や止揚などで重要な意味を帯びてくるかもしれませんし。
制限をもうけなくても都合の悪いものは計算不可能レベルの特権で無視することもできますし。
曖昧というか、抽象ゆえの強さを殺してしまってる気がします。
ラヨ関数の定義は、関数としての必要十分条件を満たしており、あれですでに明確に完成されていると
私は考えています。
不確かさを受け入れなければ本当の強さが見えてこないのではないでしょうか・・・

>>198では特に>>197の例に答えたわけではありません。紛らわしくてすみません。

every というのは、ここでは「すべて」というよりは「〜ごとに」と言う意味で、
数学的に「すべての、任意の」を意味するのであればallが使われると思います。

媒介変数という言葉はこの場合パラメータという英語で使われることのほうが多いかもしれません。

全称量化記号は暗黙のうちに省略することもあります
たぶん充足可能性の議論なんかでは特に
211 :
2016/02/19(金) 20:18:10.78 ID:2+0y1c4N
媒介変数と言ってしまうと数に限定されてしまうのでこの場合パラメータと言うべきでしたね。
こちらの言葉遣いに問題がありました。すまない

比較的簡単な0の定義について

0が必ず空集合を意味しなければならないわけでもありませんが、ここでは簡単のため
0 := {}, suc(a) := a ∪ {a}
とします。

すこし語弊があるかもしれませんが、率直に言って、ラヨ関数の論理式は前提条件から
真偽を図るものではなく、前提条件そのものです。
?∃2(2∈1)はZFCから「1」が空集合であることが保証されると言われましたが、ではなぜ
ZFCの空集合の公理が正しいのかと言われると、それは公理だからとしか言いようがありません。
なぜ空集合が0を意味するのかということについても、集合論とは別の問題ですが、同じことです。
理論ではなく論理の前提条件としてそれを0と呼んでいるだけです。
(論理の前提条件とは普遍的な大前提のことです)
212 :
2016/02/19(金) 21:14:22.04 ID:2+0y1c4N
無矛盾なZFCを前提条件とすれば任意の論理式は真か、偽か、決定不能のいづれかに分類されます。
しかしZFCを前提条件としなければ、

ZFC∧「任意の論理式」

というかたちで議論を保持することもできますが、ほかの前提条件、たとえ矛盾するような
前提条件でも議論の基準とすることが可能となります。
かかれた式はすべて真となります。なぜ真になるのか? ではなくそう決めたから真なのです。

書かれた式を前提条件とし、「「1」は100である」という命題が真となり矛盾しなければ「1」
は100を意味することになるます。(等号に関する公理は大前提とします)
0については、ここでは空集合の唯一性についてまでは触れませんが、
?∃2(2∈1) ・・・式A
から「1」が{}であり、0という数を意味することがわかります。
ここでなぜ式Aが{}を意味するかを構成的に確認するとなると、
フォン・ノイマン宇宙の要素すべてを{}のなかに入れてみることになります。
しかしフォン・ノイマン宇宙は構成可能な集合ではないので、構成的に
まとめる事が出来ない=神託機械でも計算できない、ということになります。
構成という言葉の使い方に自信がありませんが。
213 :
2016/02/19(金) 21:27:37.54 ID:2+0y1c4N
http://googology.wikia.com/wiki/User_blog:Vel!/BB(n)_in_FOST

Vel!さんが作ったFOSTによるビジービーバー関数のプログラムだそうです。
FOSTで定義可能という時点でラヨ関数はより強い関数ということになります。
私は把握できていないのでこのプログラムの正しさを保証することはできませんが、
彼(彼女?)ならおそらく信頼できるでしょう。
214 :
132人目の素数さん
2016/02/20(土) 15:37:16.42 ID:XR/jrIxI
曖昧さを残している関数に対して、私は
「f(n) = (n文字以内の日本語で定義できない最小の自然数)
ただしベリーのパラドックスやその他の矛盾は一切自動的に排除される」
という関数と区別を付ける理由をもちません。
あなたは「いや、それとは違う」とあなたなりの言葉を並べて否定するかもしれません。
しかし、どこまでの曖昧さが許されてどこからが許されないかなんてもはや哲学の問題です。

(少なくとも議論が起きる程度には不確かな)関数に明確な定義を与えよう、という立場と
すでに十分定義として完成しており、具体化しようとするほうがむしろ間違い、
不確かさを受け入れるべき、という立場では
話が合わないのも当然です。

この2つの立場のどちらが妥当かというのはもはや哲学の問題で、
具体的な定義を与えられないと数学の議論にはならず、
またそちらは具体的な定義を与えるつもりも無いようですので、
議論は降りさせていただきます。
215 :
2016/02/22(月) 19:33:42.87 ID:BR6aQj3k
すべての矛盾を排除する必要はないと思いますが、その話はこの際おいといて、

「ラヨ関数の今の定義は関数として成り立つには十分具体的である」
というのが私の立場です。
数学的な議論はできなくても、超数学的な考察を展開してみます。
どこまで曖昧さが許されるのか、というのは裏を返せばどれ程の具体さが必要とされるのか、
という問題になります。
ここでは「具体的な定義」について考えてみます。
私が「具体的な定義」と感じるものが貴方に具体的に見えないというのが意見の食い違う元です。
「具体的な定義」を数学的に議論するために具体的な定義が必要となりますが、
これが定義に関する議論であれば循環論法になってしまい数学のなかで扱えなくなってしまいます。
ここで形式主義から述語論理に頼ったり論理主義から型理論に頼ったりするのでしょうが、
まだまだ学が浅いものですぐボロがでてしまうのでこのくらいにしておきます。

これまででも十分勉強になりました。ありがとうございます。
また折りがあれば触れたいと思います。
216 :
2016/02/22(月) 19:56:07.41 ID:BR6aQj3k
ちょっと思い付いたので補足してみる。

フォン・ノイマン宇宙Vをして、FOST上では{}と{V}は区別がつきません。
ラヨ命名とは文字通り、「もの」ではなく「名前」を定義するため、
具体的にはどのような「もの」であろうと、「FOST上での」0の定義を満たしていれば
{V}も0と名付けられます。

いかに具体的な定義でも論理が抽象的に深くなれば、
曖昧な部分を掘り起こすことができるようになります。
ZFC上でも同じことが言え、{}と{V}の区別がつきません。
217 :
132人目の素数さん
2016/02/24(水) 19:08:46.94 ID:8s+x0HCh
可融数の強さって算術と対応させることができるからでしたかね?
218 :
2016/03/03(木) 11:59:09.91 ID:iHhoMPoZ
寿司きてる!
219 :
132人目の素数さん
2016/03/03(木) 11:59:43.35 ID:iHhoMPoZ
記念age
220 :
2016/03/06(日) 21:00:22.60 ID:3xxZorGC
#defineが多いコードは
$ gcc -E Loader.c
のようにすれば読みやすくなるんですよ
221 :
2016/03/07(月) 21:41:14.06 ID:MwdG/6Hv
寿司がいつかハリウッド映画化して日本のガラパゴス化した巨大数にうんたらかんたら
アニメ化でもいい、ってかこっちのほうがよっぽど現実的
222 :
2016/03/09(水) 12:11:32.56 ID:3Hlsk1pY
>>221
???「巨大数のアイドル、BOX_M?だよー」
223 :
2016/03/09(水) 19:42:59.92 ID:tu05kg83
>>222
おっぱいでかくてもぼくは内面気にする人なんです
224 :
2016/03/10(木) 17:39:48.02 ID:/UvFysI6
貧数よりは巨数の方がいいが爆数までいくとちょっとひく
まぁ美数にかなうものはないんですが
225 :
132人目の素数さん
2016/03/12(土) 10:08:55.00 ID:a/Cx6aW6
巨大数の式って展開していくところが自己相似のフラクタルっぽいけれど
フラクタル側から巨大数にアプローチできないもんかな
226 :
2016/03/12(土) 17:53:41.90 ID:EA+ZN8v2
>>225
コッホ曲線を元にしてできた関数
C(a,b,c)=C(a,C(C(a,b-1,c),C(a,b-1,c),C(a,b-1,c)),c)
C(0,a,b)=a^b
C(a,0,c)=C(a-1,C(a-1,a,c),c)
C(a,b,0)=C(a-1,b,C(a,b-1,b))
明らかに弱いしω^ωより絶対に小さい
227 :
2016/03/14(月) 16:58:08.18 ID:sT/u/Cgs
構造の計算ってのがさっぱりわからん
こういうのがあったけど
http://www.is.ocha.ac.jp/?asai/jpapers/ppl/suzuki06.pdf
228 :
2016/03/18(金) 19:42:56.25 ID:9ZQehogR
息抜き原始数列

Number.prototype.times = function(f){for(var n=+this,i=0;i<this;i++)f()};
Function.prototype.repeat = function(n,x){var f=this; n.times(function(){ x=f(x) }); return x};
Array.prototype.last = function(){return this[this.length-1]};

f = function(m){
 if(m==1)return function(n)n+1;
 if(m>=2){
  var g = function(a,x){
   a = a.slice();
   a.push(x);
   if(a.length >= m){
    var h = a.shift().repeat(a.last(), a.shift());
    return a.reduce(function(f,x)f(x), h); //h(a[0])(a[1])...のようにする
   }else
    return g.bind(null, a);
  };
  return g.bind(null, []);
 }
}
229 :
2016/03/19(土) 20:28:36.72 ID:pY9n5UjB
巨大数を生み出す表記はだいたい数字をいろんな記号で区切ったり入れ子構造にしたり配列空間を
複雑にしたりする。しかし見やすさを重視しなければすべて数字に置き換えることが可能、
そのかわり数字の解釈が複雑になる。

基本的にその表記の極限へ向かう列はだんだん大きくなっていく。
⇒無限に大きくなる表記の極限を小さい表記にすることで空間を効率よく活用できる。

バシク行列はその上に解釈される数字の役割を列や行列に当てることで解釈や表記をを簡単にしている
形式の面ではもうサラダ数的なテクニックしか残ってないと思う。
230 :
132人目の素数さん
2016/03/20(日) 09:48:58.25 ID:IrUBdJit
巨大数種類の記号を式で定義できればまだ伸びるってこと?
231 :
2016/03/20(日) 09:49:25.34 ID:IrUBdJit
ごめんsage損ねた
232 :
2016/03/20(日) 14:00:03.80 ID:qWvhRznF
或いは解釈を複雑にしていくとか
表記そのものより表記を読み解くプログラムの開発、もしくは論理的な掘り下げ、
ふたつの方針がある
233 :
2016/03/26(土) 15:25:24.93 ID:QvshSWoH
数字じゃなく図形で巨大数を表記できないかな
234 :
2016/03/26(土) 16:06:48.41 ID:gyqb5B6b
すべての曲線の集合はℵ2らしいから大きくなりそう?

でも、意味あるのかな?
235 :
132人目の素数さん
2016/03/26(土) 16:52:27.51 ID:ZYwB9lcF
2ってアレフのこと?
236 :
2016/03/29(火) 02:23:48.35 ID:tBmqRa4S
あ、機種依存文字読めなかった?

すべての曲線の集合はAleph_2らしいから大きくなりそう?

でも、意味あるのかな?
237 :
2016/03/29(火) 12:07:39.76 ID:xGYEbYdU
Alephというより、Bethだと思う
238 :
2016/03/29(火) 20:31:50.89 ID:GxZ3+c2s
そういや順序数って離散的なのに全体の集合は実数濃度より大きいんだっけ?
なんか自分で言っててよくわからんくなってきたが。
239 :
2016/03/30(水) 11:35:04.71 ID:PcDYXwuk
>>238 つΩ
240 :
2016/03/30(水) 16:11:59.09 ID:fYOdLkWo
順序数全体は集合でなく真のクラス
241 :
2016/03/31(木) 10:45:59.60 ID:U9iDzcmk
ナゴヤどうなったの
242 :
2016/04/02(土) 19:34:44.42 ID:mr9Lf7Py
論理的な掘り下げというのは逆だったな

配列を複雑にしたり、記号を増やしたりする形式的(論理的)な強化は比較的簡単。
定理を駆使して形式はそのままにシステムそのものを見直すのが難しい。

文字数を増やしてプログラムを複雑にするのが前者、
文字数はそのままに、あるいは減らして、最初からより強力なシステムを作っていくのが後者
後者は計算が終了することを証明しなきゃいけない、というのが
ビジービーバー関数の値を求めるくらいに難しい
243 :
2016/04/05(火) 02:13:29.04 ID:NR45aGYT
自分はバッハマン・ハワードまでしかまともに理解できてないんですけども収束列って便利ですよね。全部の表記を収束列の定義という形でどうにかできませんかね。
244 :
2016/04/05(火) 18:47:43.87 ID:NR45aGYT
ヴェブレン関数をn次元に拡張しました。
多分バッハマン・ハワード行くと思います。

α=α_0 ⇔ i=0
α>β⇔ i>j ∨ α_j>β_j
where
 α=(α_i, …, α_0)
 β=(β_j, …, β_0)
 i =max{0,k | α_k≠0}
 j =max{0,k | β_k≠0}

C_0(α)={0}∪α
C_{n+1}(α)={γ+δ,φ(η) | γ,δ∈C_n(α); η<α∧¬(∃β∈η∧β∉C_n(α))}
C(α)=∪[n<ω] C_n(α)
φ(α)=min{β | β∉C(α)}
245 :
132人目の素数さん
2016/04/06(水) 02:04:18.18 ID:cOqSesfo
φ(1,,0)=θ(Ω^ω)
φ(1,,1)=θ(Ω^ω×Ω)
φ(1,,ω)=θ(Ω^ω2)
φ(1,,φ(1,,0))=θ(Ω^θ(Ω^ω))
φ(1,,1,0)=θ(Ω^Ω)
φ(1,,1,1)=2nd α↦θ(Ω^θ(α))
246 :
2016/04/06(水) 20:20:42.56 ID:L6ywnPTc
φ(.. )メモメモ
\(^ω^\)( /^ω^)/
(θωθ)
247 :
132人目の素数さん
2016/04/06(水) 23:11:52.78 ID:B5VJ8hO+
あのー、幾つか質問なんですけど、

?wikiの方でヴェブレン関数をω使ったチェーン表記に直してる奴あったんですけど
途中までしかなくてよく分からなかったんだけどもあれは変数増えるにつれて
チェーンが一個伸びるって事で良いんですかね?
だとしたら小ヴェブレンは三重帰納関数にω突っ込んだ大きさ?

??が合ってるとしたら大ヴェブレンはωを、θ(Ω)=Γ_0の大きさの関数に
突っ込んだ大きさ?

?バッハマン・ハワードはΩ↑↑ω?だとすると?の操作を行う回数を
数え上げればOK?いや、Ω↑↑ωみたいな書き方が許されるのかも
分からないんですが・・・

色々すみません数学そんなに得意じゃないもので・・・
248 :
132人目の素数さん
2016/04/06(水) 23:18:20.05 ID:B5VJ8hO+
あー、でもよく見てみたら大ヴェブレンの基本列の途中で?の奴超えてますね・・・
じゃあ?のバッハマンの解釈がそのまま大ヴェブレンの大きさって事なんですかね?
249 :
2016/04/07(木) 00:04:38.89 ID:UzN5hUED
小ヴェブレン順序数はω^ω
大ヴェブレン順序数はω^(ω+1)
バッハマン・ハワード順序数はε0
と、勝手に思っている
250 :
2016/04/07(木) 11:46:47.45 ID:6iLYfBA+
順序数のチェーン表記のやつは多分あてにしないほうがいい。
251 :
132人目の素数さん
2016/04/07(木) 23:28:57.11 ID:xhOGAPDq
うーん、でもそうなると増加速度を順序数で表すのが出来なくて・・・
φ(ω,0)までは良いんだけどそこから先が・・・
って言ってもそこまでの計算が合ってるのかも微妙ですが。
252 :
2016/04/08(金) 08:01:31.32 ID:nkGHWTok
...φ(ω0)
253 :
2016/04/08(金) 13:32:47.06 ID:PHQd/G4T
何に悩んでいるのかわからんが、自然数の関数に超限順序数をそのまま放り込むのはよくない。
順序数の計算は左結合じゃないと(あまり)意味ないから
254 :
2016/04/08(金) 13:47:59.89 ID:PHQd/G4T
根本的に順序数を勘違いしてる? どうして増加速度を順序数で表せない
という結論になるのか分からない。
255 :
132人目の素数さん
2016/04/08(金) 18:10:38.71 ID:WS1MZ0CX
いや、皆さんがどうやって計算してるのかが分からなくて・・・
とりあえず自己流で色々考えてみたんだけどやっぱり駄目っぽいですね。
頑張って調べてみます・・・
ちなみにその辺(計算法)ここの前の方のどこかで出てたりしました?覚えてたらで良いんで教えて頂けると有り難いです。
256 :
2016/04/09(土) 01:56:47.58 ID:dnN4ZpXH
俺専用のφ関数計算表がこれ
α,βは順序数、0..は0個以上の0、#は0個以上の何か

φ(α)=ω^α
φ(0..,#)=φ(#)
φ(#,β+1)[0]=φ(#,β)+1
φ(#,α+1,0..,0)[0]=φ(#,α,0..,0)
φ(#,α+1,0..,β)[n+1]=φ(#,α,φ(#,α+1,0..,β)[n],0..)
257 :
2016/04/09(土) 02:05:43.21 ID:dnN4ZpXH
θ(α)はα=Ω^β*γのときφのβ番目がγと見做して計算できた希ガス

θ(Ω)=ε_0
θ(Ω^2)=Г_0
θ(Ω^ω)=アレ
θ(Ω^Ω)=もっとアレ
258 :
2016/04/09(土) 20:37:25.60 ID:2GPjqpXt
ω*(ω+α+1)=(ω*(ω+n))+ω
ω↑(ω+α+1)=(ω↑(ω+n))*ω
ω↑↑(ω+α+1)=(ω↑↑(ω+α))↑ω
展開すると左結合の入れ子構造ができるように定義すると、指数タワーができないというのが問題
後はまかせた
259 :
2016/04/10(日) 14:06:58.24 ID:iAsZKM/s
C_n(α,0,β) = β∪{β}
C_(n+1)(α,x,β) = {γ,hyper(γ,u,δ)|γ,δ,u∈C_n(α,x,β); 0<u<x}
C(α,x,β) = ⋃[n<ω] C_n(α,x,β)
hyper(α,x,β) = min{γ|γ∉C(α,x,β)}

C_0(α,0,ω) = {0,1,2,...,ω}
C_1(α,0,ω) = {0,1,2,...,ω}
...
C(α,0,ω) = {0,1,2,...,ω}
hyper(α,0,ω) = min{ω+1, ω+2, ...} = ω+1

ω^ω = hyper(ω,3,ω):
 C_0(ω,3,ω) = {0,1,2,...,ω}
 C_1(ω,3,ω) = {0,1,2,...,ω, ω+1,...,ω+ω, ω*2,...,ω*ω}
 C_2(ω,3,ω) = {0,1,2,...,ω,...,ω*4, ω*ω,...,ω*ω+ω*2, いろいろ, ω*ω*ω*ω}
 hyper(ω,3,ω) = {ω^ω, ...} = ω^ω

おk?
なんか前n^ω=ω+1とか言ってた人がいたから合ってるか心配なんだが
260 :
2016/04/16(土) 01:54:25.96 ID:Yms7D0Qn
生きてますか
261 :
132人目の素数さん
2016/04/18(月) 23:58:01.82 ID:P4xYVoLE
皆さん色々有難うございます、とりあえず小ヴェブレン〜大ヴェブレンについては
基本列と比較しながら計算してたら多分分かりました。
ただ、あの、前に書いた通り数学あんまり得意じゃないのでこの手のやつは
何と言うか苦手だったり・・・
まあ、具体的なの聞かないとあんまり理解できそうに無いんで自分で出来そうな所から
少しずつ頑張っていきます!
ちなみに計算法知りたいのは自分の作ってる数の大きさ調べる為、かなり
ゴチャゴチャしてて出来良くないからここで聞かずに自分で計算するつもり。
262 :
2016/04/24(日) 22:15:54.77 ID:7zEqEfX7
ふぃっしゅ数バージョン4では関数を神託にしていますが、
神託はテープを処理しなければならないので、
神託を具体的に構成する必要があると思います。
263 :
2016/04/24(日) 22:36:05.70 ID:v1Iv1yam
神託を具体的に構成ってどういうこっちゃ。
神託の意味わかっとるんか。
264 :
2016/04/25(月) 16:26:44.86 ID:3rsp5cSn
smapのリーダーだろ
265 :
2016/04/25(月) 17:25:41.17 ID:Y/42B474
test
266 :
2016/04/27(水) 02:39:46.39 ID:2L1s7Mem
辞書式配列とかカントール標準形とか、これが強さの秘訣だと
決めつけ過ぎるのも考えを浅くしてしまうなと思いました。
過学習みたいなことが人間にも言えそう。
267 :
2016/04/28(木) 09:07:26.56 ID:5FqmvL1r
取っ掛かりをつくると初心者にも分かりやすく説明できるが、あんまり楽な
やりかたばかりに頼っていると本質を蔑ろにしてしまうおそれがある。
言語から直感、直感から霊感への昇華が要求される。脱構造主義的な努力が重要だ
268 :
2016/05/05(木) 20:52:52.00 ID:uScstc6X
やっぱ繰り返しとその中を直線的にいくことをどれだけ簡潔に表せるかでしょ
269 :
2016/05/05(木) 23:28:09.08 ID:8Eb/ZGVE
#(A, B) = hyperA(B, hyperA(B, hyperA(...[B回繰り返し]...hyperA(B, B)

#(A, B, C) = hyperA(hyperA(B, hyperA(B, hyperA(...[B回繰り返し]...hyperA(B, B) , hyperA(,
270 :
2016/05/05(木) 23:32:44.23 ID:8Eb/ZGVE
途中カキコミス

#(A, B) = hyperA(B, hyperA(B, hyperA(...[B回繰り返し]...hyperA(B, B)

#(A, B, C) = hyperA(hyperA(B, hyperA(B, hyperA(...[B回繰り返し]...hyperA(B, B), hyperA(hyperA(B, hyperA(B, hyperA(...[B回繰り返し]...hyperA(B, B), hyperA(hyperA(B, hyperA(B, hyperA(...[B回繰り返し]...hyperA(B, B),
hyperA(...[C回繰り返し]...
hyperA(hyperA(B, hyperA(B, hyperA(...[B回繰り返し]...hyperA(B, B)
271 :
2016/05/06(金) 00:02:44.31 ID:SMtyXma4
巨大関数を作るアルゴリズムは計算不可能なのかNP困難なのか
272 :
2016/05/06(金) 20:42:10.74 ID:K2vLIneF
ビジービーバーまで行けば計算不能だが。
巨大関数って結局メモリいっぱい使うからNPまでだとあんまり大したことなさそう。
273 :
2016/05/06(金) 20:51:11.99 ID:SMtyXma4
原始数列システムが画期的すぎてあれに勝てるのを思いつけない雑魚←
274 :
2016/05/06(金) 23:14:03.30 ID:a+o7OkhU
n番目の奇数の完全数とか、3つ組の社交数とか、コッラッツ予想の反例とか。存在すればだけど。
275 :
2016/05/06(金) 23:55:16.03 ID:Dmd6RGaE
>>270
括弧あけっぱ
276 :
2016/05/07(土) 02:27:35.22 ID:W2BwXITc
↑無限

計算不能
──────
↑無限

計算可能
277 :
2016/05/07(土) 03:04:14.09 ID:W2BwXITc
メモ

α↦f(α) = sup{f(α), f(f(α)), f(f(f(α))), ...}

φ(α)=ω^α
φ(1,α)=ψ(α)=ε_α
φ(2,α)=ψ(Ω×α)    φ(2,0)=ψ(Ω)=α↦ψ(α)
φ(3,α)=ψ(Ω^2×α)    φ(2,φ(2,φ(3,α)))=ψ(Ω^2×α+Ω×2)
φ(ω,α)=ψ(Ω^ω×α)
φ(ε_α,β)=ψ(Ω^ψ(α)×β)     φ(φ(ε_0,0),0)=ψ(Ω^ψ(Ω^ε_0))
φ(1,0,α)=ψ(Ω^Ω×α)     φ(1,0,0)=ψ(Ω^Ω)=α↦ψ(Ω^α)=Г_0
278 :
2016/05/07(土) 03:21:18.18 ID:W2BwXITc
うわっ間違えた
×αじゃなくて×(1+α)だ
279 :
2016/05/07(土) 03:21:47.72 ID:W2BwXITc
それに^αじゃなくて^(1+α)だ
280 :
2016/05/07(土) 16:22:23.38 ID:W2BwXITc
(0,0)(1,1)=ε_0
(0,0)(1,1)(1,0)(2,1)=ε_0^2
(0,0)(1,1)(1,0)(2,1)(2,0)=ε_0^ω
(0,0)(1,1)(1,0)(2,1)(2,0)(3,1)=ε_0^ε_0
281 :
2016/05/07(土) 19:59:45.49 ID:W2BwXITc
バシク行列をどんどん上がっていき対応する表現を知ることで
バシク行列だけでなく強い順序数の表現もわかりそうな
282 :
2016/05/10(火) 20:41:25.31 ID:U2eUzQF3
Taranovsky's C では Second order なんとかとそれより前では Ω の扱いが違っていて、
二階算術の力を持っている方の定義では、
Ω は十分大きい順序数、Ω_2 は十分大きい順序数を定義するためのより十分大きい順序数、
Ω_3 はより十分大きい順序数を定義するための更により十分大きい順序数、・・・
となっている。もやもやしていた原因が判ってすっきりしたわ
283 :
2016/05/15(日) 02:58:29.10 ID:3ipWRuVt
θ(α,β)>θ(γ,δ)⇔(α=γ∧β>δ)∨(β,δ<Ω∧α>γ)
θ(α,β)>β⇔β<Ω

C_0(α,β)={0,β}
C_x(α,β)={γ+δ,γδ,γ^δ,θ(η,δ),θ(α,ε)|γ,δ,η,ε∈C_y(α,β); y<x; η<α; ε<β}
θ(α,β)=min{γ|γ∉C_ω(α,β)}

θ(0,0)=ω, θ(0,1)=ε_0
284 :
2016/05/22(日) 00:17:44.18 ID:tG2h3cTG
巨大数の表記をゲーデル数化みたいなテクニックを使って、計算を数論的な操作
に置き換える、というのは、証明が高度に抽象的な定理を比較的簡単な形式的証明で
(一部)表現できるというロマンがある。

グーゴロジーってマイナーな分野でまだあまり知られていないし、
一般的な分野への応用ってのはけっこう開拓の余地がありそう。
285 :
2016/05/28(土) 15:40:01.61 ID:MyjvHFel
P[0](n)=1

P[b](n)の定義(bは正の整数)
ビジービーバー関数(Σ(n))の
「テープ上の現在位置の数値が0」を「テープ上の現在位置の数値をaとしてP[b-1](a+1)が偶数もしくは0」に変えて
「テープ上の現在位置の数値が1」を「テープ上の現在位置の数値をaとしてP[b-1](a+1)が奇数」に変えて
「現在位置の数値を1に書き換える」を「テープ上の現在位置の数値をaとして現在位置の数値にP[b-1](a+1)を加算する」に変えて
「テープに書かれた1の数」を「テープに書かれた数値の合計」に変えたもの

P[ω+1](n)の定義
P[b](x)の「P[b-1](a+1)」を「P[a](a)」に変えたもの

定義からP[1](n)=Σ(n)です
P[2](n)以後の大きさはわかりません
P[ω+1](n)の大きさは全く分かりません
ここから更に拡張できるけどその大きさは更に分かりません
そもそも関数として成立するかも分かりません
286 :
2016/05/28(土) 15:45:45.80 ID:MyjvHFel
ミスしました
「テープ上の現在位置の数値をaとして現在位置の数値にP[b-1](a+1)を加算する」は「テープ上の現在位置の数値をaとして現在位置の数値にP[b-1](a+1)に書き換える」です
287 :
2016/05/29(日) 03:08:24.58 ID:4syGsLFf
ふぃっしゅ数バージョン4と同じアイディアだべ
ふぃっしゅ数はそこまで具体的に踏み込んでなかったけど

計算不可能レベルは論理の力を借りないと固くてぜんぜん掘り下げられないのだ
神託機械で掘れるところはすでに掘られてるし
288 :
2016/05/29(日) 12:27:26.60 ID:isTysxwN
>>287
P[ω+1](n)は「ビジービーバー関数から相対的に計算不可能な関数から相対的に計算不可能な関数から……と有限回数繰り返した関数全てから相対的に計算不可能な関数」
にしたつもりだったけど、この様子だと既にあったんだろうなぁ
289 :
2016/05/29(日) 16:35:44.80 ID:V9erIGG9
現在位置の数値に1を加算するのとP[b-1](a+1)を加算するのってどれぐらい違うんでしょうか
290 :
2016/05/30(月) 06:25:22.71 ID:zQcbv53d
神託を使うか使わないかの違い
291 :
2016/05/31(火) 20:24:48.97 ID:wwMW4Ikb
理解
292 :
2016/06/01(水) 23:00:29.91 ID:QIx67vEi
クサイ関数はひとつの神託コンビネータでω^CK_ωまでいけるのがなかなかおいしい
293 :
2016/06/06(月) 10:55:17.08 ID:UGytk50x
loader.cのZ関数ってZ(0)の計算が終了しないんじゃないの
294 :
2016/06/06(月) 12:50:53.61 ID:UGytk50x
Z(0)=0として計算してみるとD(2)=15*2^(7*2^128)になる。
英語版のtalkページによると65565になったり
ならなかったりする模様。
295 :
2016/06/06(月) 12:53:52.11 ID:UGytk50x
訂正 65565→65536
296 :
2016/06/06(月) 22:39:18.06 ID:yv8DOjRT
関数f(x)とg(x)があるとき、ある自然数aに対してa≦bとなるb全てにおいてf(b)<g(b)となるとき「g(x)はf(x)より強い」とする
また「f(x)はg(x)より弱い」とも言う

強比関数
a>bであるとき[a]C(x)は[b]C(x)より強い
a>0であるとき[a]C(x)は[a]C([b]C(x))より強い
これを前提にして色々弄って構築する

……

[1]C^{a+1}(x)=[1]C([1]C^{a}(x))としたとき[1]C^{a}(x)はa次関数相当
[1]C([1]C^{x}(a))でx↑↑(x+y)程度
[1]C^{x}([1]C^{x}(a))でx↑↑(x^y)程度

[1]C^{a}^{b+1}(x)=[1]C([1]C^{a}^{b}(x))としたとき
[1]C([1]C^{x}^{x}([1]C^{x}(a)))でx↑↑↑(x+y)程度

みたいなの作ってみたけど、なんだか急増加関数に似てる
297 :
2016/06/06(月) 22:55:11.29 ID:yv8DOjRT
[1]C^{a+1}(x)=[1]C([x]C^{a}(x))としたとき[1]C^{a}(x)はa次関数相当
[1]C([x]C^{x}(a))でx↑↑(x+y)程度
[1]C^{x}([x]C^{x}(a))でx↑↑(x^y)程度

[1]C^{a}^{b+1}(x)=[x]C([1]C^{a}^{b}(x))としたとき
[1]C([x]C^{x}^{x}(x))でx↑↑↑(x+y)程度

またミスった、ちくせう
298 :
2016/06/10(金) 02:25:28.95 ID:935AerH0
http://pastebin.com/esC6EzF2
Loader.cをRubyに直そうとして挫折しました
299 :
132人目の素数さん
2016/06/10(金) 12:26:16.73 ID:3h2dAEId
>>50
それは、CG関数じゃないですか
CG(n)=n→n→n→n→・・・n組のn
では俺が今難航してる関数を見せてあげる
a(b)=a→a→a→・・・ b組のa
a(b)c=a(b)→a(b)→・・・ c組のa(b)
どっちが大きいかな?
a(b(c)d)、[a(b)c]d
前者がa→a→・・・d組のb(c)組のa
後者がa(b)c→a(b)c→a(b)c d組のa(b)c
a(b(c)d)e=a(b(c)d)→・・・ e組のa(b(c)d)
[[a(b)c]d]e=[a(b)c]d→・・・e組の[a(b)c]d
もうこれで解るよね、二つに別れてしまうんだ
最終的にはこんな事に
a(b(c(d(e)f)g)h
[[[[[a(b)c]d]e]f]g]h
複雑でしょ、どっちが大きいか順位付けしなきゃならない
でもこれはステージ1だからまだ序の口
300 :
2016/06/10(金) 12:56:36.67 ID:E8Vst9Nv
Z関数の真理値の解釈逆だったかな。whileの条件式もなんかはっきりとわからない。
誰かがifに直してくれたものを参照してみるけど。
313 :
四次元表記に挑む者
2016/06/11(土) 09:42:55.04 ID:gV9Z5/0x
ではステージ2に行こうか、ステージ2はステージ1に関数の入れ子が入るんだ
a^2(b)=a(a(b))=a→a→・・・a(b)組のa
a^n(b)=a(a(a(・・(b))・・)=a→a→・・・ a(a(a(・・(b))・・)組のa
a^2(b)c=a(a(b))→・・・c組のa(b)組のa
後はステージ1と同じく二つに別れてしまう
こいつの強さはBEAFの線形配列位かな?
324 :
2016/06/11(土) 11:11:40.14 ID:bOxhyo7t
sageたほうが良い。
f_{ω_1}を仮想関数としてなにかできそうなものだけど、順序数崩壊関数とかぶる。
335 :
四次元表記に挑む者
2016/06/11(土) 12:56:14.89 ID:gV9Z5/0x
それでは、こっから先はmixiで悪戦苦闘してるので良かったら来てね
350 :
2016/06/12(日) 01:48:06.33 ID:p+bNzhXN
1文字レスじゃp53効かないな・・・
¥だけの書き込みNGに指定されないかな・・・
352 :
2016/06/12(日) 16:23:39.28 ID:hlYfk4FC
>>350
コテハンをNGnameにすればいい
353 :
2016/06/12(日) 23:07:08.86 ID:p+bNzhXN
>>352
ありがつお

巨大数のアルゴリズムって圧縮に応用できそう・・・できそうじゃない?
354 :
2016/06/12(日) 23:14:31.78 ID:30OWvOMg
むりじゃねw
355 :
2016/06/13(月) 01:12:32.19 ID:a2wQDq8z
むりかー


あとこれ
http://b it.ly/1Uu0RSJ
F_α(a,0)=a
F_0(a,b)=b+1
F_α+1(a,b+1)=F_α(a,F_α+1(a,b))
F_g(α,X)(a,b)=F_g(α,b)(a,b)

これだけ使ったら極限はどれだけいけるんだろう

F_n(a,b)=a→b→n-2
F_2X(a,b)=F_{F_1(X,X)}(a,b)=F_(X+3)(a,b)・・・H[ω^2]ぐらい?

まずφ(ω,0)とГ_0はいけそうな気がする
356 :
2016/06/13(月) 01:13:59.91 ID:a2wQDq8z
間違えた
F_0(a,b)=a+1ね
357 :
2016/06/13(月) 20:41:12.47 ID:s3hLQ9h0
マグニチュード

計算式

log_{10}E=4.8+1.5M

E は(普通は地震の)エネルギー、M はマグニチュード。

7.0から大地震、8.0から巨大地震、9.0から超巨大地震。

9.5よりも大きい地震は現在知られていない。

大雑把な計算でプレートテクニクスで10.0までの地震が起こりうるとか。

恐竜が絶滅した原因となった直径10-15kmの隕石の衝突で
解放されたエネルギーが11以上と見積もられている。

地球が太陽から受け取る総エネルギー一日分が11.5

直径400kmクラスの隕石が衝突した際に解放されたエネルギーが14.5?
例の動画のやつ。

ジャイアントインパクト説に基づく、火星大の原始惑星(直径6000kmほど?)
が衝突した際に解放されたエネルギーが18.0? 真正面から衝突していたら
地球は崩壊してたとか。

マグネターSGR 1806-20の星震の総エネルギーが23.1?

最近観測された重力波の震源地で解放されたエネルギーが28.6ほど。
太陽3個分の質量をすべてエネルギーに変換したくらい。
物理的防御すべてを貫通するので耐震制震免震が一切効かず、
近くで起こったらなかなかやっかい。

観測可能な宇宙の総エネルギーが40台だったか?
358 :
2016/06/13(月) 21:23:00.96 ID:a2wQDq8z
>>299
それって関数というか表記法じゃ・・・

n(n)≒f[ω^2+1](n)
n(n)n≦f[ω^2+2](n)
n(n(n)n)≒f[ω^2+2](n)
でおk?

理想は繰り返しの構造を簡潔に表してそこを置き換えさらに複雑になれるようなやつだけど・・・
それってバシク行列じゃん!
359 :
2016/06/13(月) 21:40:09.13 ID:3JJBaIJ9
計算可能な関数はプログラムつけてほしいよなぁ。
計算過程もわかるような感じで。
360 :
2016/06/13(月) 22:28:51.48 ID:a2wQDq8z
>>359

ふぃっしゅ氏作のバシク行列計算機
http://gyafun.jp/ln/basmat.cgi
361 :
2016/06/13(月) 22:38:21.85 ID:a2wQDq8z
f_0(β) = β+1
{f_α+1(β)}[n] = f_α^n(β)
{f_α(β)}[n] = f_α[n](β)

以上のルールで導入テスト どれぐらい強くなるのか・ならないのか

f_1(ω)[2]=f_0(f_0(ω))=ω+2
f_2(ω)[2]=f_1(f_1(ω))
f_1(f_1(ω))[2]=f_1(ω)+2

f_{f_2(ω)}(2) = f_{f_1(ω)+2}(2) = f_{f_1(ω)+1}(f_{f_1(ω)+1}(2))
= f_{f_1(ω)+1}(f_{f_1(ω)}(f_{f_1(ω)}(2))) = f_{f_1(ω)+1}(f_{f_1(ω)}(f_{f_1(ω)}(2)))
= f_{f_1(ω)+1}(f_{f_1(ω)}(f_ω+2(2))) = f_{f_1(ω)+1}(f_{f_1(ω)}(f_ω+1(f_ω+1(2))))
= f_{f_1(ω)+1}(f_{f_1(ω)}(f_ω+1(f_ω(f_ω(2)))))
= f_{f_1(ω)+1}(f_{f_1(ω)}(f_ω+1(f_8(8))))

順序数を少し大きくするのと同程度には大きくなった(気がする)

解析のやり方は知らないんでちょっと下手だけども
f_1(ω)≒ω・2
f_2(ω)≒ω・2^ω
のような感じだろうと思う
362 :
2016/06/13(月) 22:45:28.97 ID:3JJBaIJ9
>>360
ほほう。
こういうのはありがたいね。
さすがふぃっしゅ氏。
363 :
2016/06/13(月) 23:15:16.34 ID:3JJBaIJ9
よくわからんがバシク行列ってのはかなりいい線行ってる関数なの?
364 :
2016/06/13(月) 23:35:29.15 ID:a2wQDq8z
>>363
巨大数探索スレで考案された中ではおそらく一番、巨大数の表記でも一番、強さではおそらく二番
一番強いのがLoader.c
365 :
2016/06/13(月) 23:37:53.63 ID:a2wQDq8z
あ、計算可能ではね、と思ったけどwiki見てみたらもっと大きいのがあるらしいな・・・
366 :
2016/06/14(火) 00:14:25.92 ID:K+MR2/0t
レイバーのテーブルは強いのか弱いのかよくわかっていない状況で、
一番強いのはフリードマンの無限ゲームとかいうやつか?

ドラゴンボール見てる気分だ
367 :
2016/06/14(火) 14:47:13.51 ID:nlk32OiV
寿司きてる
368 :
2016/06/14(火) 15:18:05.38 ID:nlk32OiV
f_n(α)=α+ω^n
だめじゃないのこれ

f_0(β)=β+1
f_α(β)=R(f_α,β,β)
R(f,0,x)=x
R(f,α+1,x)=f(R(f,α,x))
R(f,α,x)[n]=R(f,α[n],x)

H[f_1(ω2)](3)=H[R(f_0,ω+3,ω2)](3)=H[R(f_0,ω,ω2)](3)+3](3)=H[R(f_0,ω,ω2)](3)](6)
=H[R(f_0,6,ω2)](3)](6)=H[R(f_0,0,ω2)+6](6)=H[ω2](12)=H[ω+12](12)=H[ω](24)
=H[24](24)=48
369 :
2016/06/14(火) 16:38:41.54 ID:K+MR2/0t
有限ゲームだった。このへんまだはっきりと強さが分かってないっぽいな
370 :
2016/06/14(火) 20:46:17.89 ID:mUIjFDF1
バシク行列ってヒドラの拡張とも考えられるの?
371 :
2016/06/14(火) 21:52:56.21 ID:K+MR2/0t
「それは君の捉え方次第さ」
スナフキンは言った。
372 :
2016/06/14(火) 22:11:15.83 ID:nTD0hdn1
個人的には考えている。拡張の1つとして
373 :
2016/06/15(水) 14:06:34.22 ID:xiIy5eza
カントール標準形を超えていれば何でもヒドラでとらえ直すことができるわけで、
システムが十分強ければヒドラをもとにしているかどうかは次元の低い問題となる
だろう。
374 :
2016/06/15(水) 19:44:43.40 ID:BsAi80Ns
捉えている、の方が通じただろう
375 :
2016/06/15(水) 20:49:52.76 ID:2A1aJ2ok
多重リストをつかったプログラムはたいていε_0になるの?
それともうまくやらないとε_0にならない?
376 :
132人目の素数さん
2016/06/15(水) 23:18:13.58 ID:CtUppN7y
>>358
うん、そうだよ表記法の一つなんだけど
nだけだと表記法としては致命的なんで
a(b)に書き直した
例えばn(n)だけだとn=3だったら3(3)しか書けないよね
3(9)ってかけないからね、
つまり自由度を高めたんだよ
チェーンの強みは簡単に指数タワーを生成できる事だよね

 
377 :
2016/06/15(水) 23:24:15.05 ID:xiIy5eza
ageたら荒らされそうなんで特に必要でない限りsageてほしいです。
388 :
2016/06/16(木) 11:37:21.52 ID:gI2C11nO
表現を複雑にして自由度を高めたという要領なんだろうけど、
どうか読み解くほうを複雑にして自由度を高める方法もあることを知ってほしい。
それに弱いシステムは何でも強いシステムよりも自由度が低いと言ってしまえるので、
これを理由にするのはどこか的外れのような気もする。
389 :
132人目の素数さん
2016/06/16(木) 20:37:20.67 ID:sfrcYgk7
>>388
確かにチェーンじゃBEAFにはかなわない、実は俺の狙いはフラクタルチェーン表記を定義しようとしてるんだ
フラクタルを可視化しようとしてるんだよ
もちろん次元化もする
それを・(ドット)変換して
四次元超立方体を生成して
四次元表記を定義しようとしてるんだよ
BEAFの実態をちょっとは知ってるからね
配列次元演算子&の威力は凄いからね
あれはめっちゃ強いよ、3^3&3って書くだけで3次元立方体が出来るんだからね
でも3次元の域を抜けないんだよ
何処までいっても3次元の巨大数なんだ
390 :
2016/06/16(木) 21:40:16.12 ID:gI2C11nO
チェーンがBEAFに敵わないとかいう話はしてないんですが、
どこから出てきたんですか?

配列次元演算子は何も指数だけとしか組み合わせちゃいけないようなもの
でもありません。
391 :
2016/06/16(木) 22:30:42.11 ID:15n8sAIo
多次元表記と多重リストって結局同じじゃないの?
402 :
四次元表記に挑む者
2016/06/17(金) 16:26:03.58 ID:6bfV9qgU
まっ、誰もBEAFを超えようとしてないからね
403 :
2016/06/17(金) 18:15:37.37 ID:OI1lZl7T
Taranovsky's Cが本気を出すまでもなくBEAFを超えてるし、その全力を出した
Cをバシク行列が、一般的な定義は未完成だけど、計算を想定している範囲だけで
余裕で超えてる。
404 :
2016/06/17(金) 18:18:36.46 ID:OI1lZl7T
同じなのはテトレーション配列と多重表記じゃなかった?
405 :
2016/06/17(金) 20:17:27.48 ID:B8r/nhFl
soko機能を試してみるか・・・

うpだてした順序数のFGH
_は^と併用すると関数の反復か単なる冪乗かわかりにくいので収束列をα_n、定数をf[]の書き方で
f[0](β) = β+1
f[α+1](β) = f[α]^β(β)
f[α]^0(β) = β
f[α]^γ+1(β) = f[α](f[α]^γ(β))
f[α]^γ(β)_n = f[α]^γ_n(β)
f[α](β)_n = f[α_n](β)

概ねうまくいったような
406 :
2016/06/17(金) 20:19:20.39 ID:B8r/nhFl
sokoできてなかったからsage sokoじゃなくてsokoだけで試してみる
407 :
2016/06/17(金) 20:19:54.52 ID:B8r/nhFl
意味なかったし上がってる
この板sokoできないのかな
408 :
2016/06/18(土) 12:33:19.95 ID:XKU76a4O
C関数わかりません><
ボスケテ
409 :
132人目の素数さん
2016/06/19(日) 06:28:35.93 ID:zLdqGbSU
>>391
多次元表記と多重リストは違うよ、多次元表記はn次元演算子を使って立方体を作るんだけど、多重リストは同じ関数が同時に存在するんだよ
だからねまさに次元が違うんだ強さも違うし
BEAFの構造は立方格子構造なんだ
でっ今まで定義されてきた巨大数は二次元フラクタル構造なんだよ
410 :
132人目の素数さん
2016/06/19(日) 07:02:42.10 ID:zLdqGbSU
>>390
ああ、的外れな事を言ってゴメンね
それはねこのスレの頭にstl関数が出てきてたからね
だから出てきた
411 :
132人目の素数さん
2016/06/19(日) 07:25:35.09 ID:zLdqGbSU
>>403
ならそれも超えようではないか、因みに四次元超立方体一個の威力は関数を8重リスト化してしてしまうんだよ、BEAFははっきり言って失敗だね、次元空間を有限にしなかったから
空間は無限大だからね、だから空間に最果てを作らないと無限大に無限大を入れ子したって、計算のしようがないからね
412 :
2016/06/19(日) 11:33:53.13 ID:TtlfQr5R
>>409
荒らしが来るからsageてね。
sageるってのはE-mailの欄に「sage」を入れるって意味だよ。
413 :
2016/06/19(日) 12:31:53.47 ID:BdVKliKM
いっちゃ悪いけど、なんかBEAFを立方云々という辺りちゃんと理解できていない
みたいだし、4次元フラクタルとか言うやつもε_0もなさそうだし、
4次元超立方体、というか多次元配列の威力が2重リストくらいだったはず、
BEAFなら。
はっきり言うと理解も考えも浅いし、言葉に酔ってる。
BEAFの超次元配列を説明してもらえますか?
414 :
2016/06/19(日) 16:22:38.12 ID:sOeYqFwR
フラクタルの人は前スレから居るフラクタルフラクタル言ってるだけの人でしょ。
相手する価値は無い。
415 :
2016/06/19(日) 21:06:14.11 ID:TtlfQr5R
チェーンの数字をすべてそのチェーン自体に置き換えるぐらいならチェーンの最後に→3を付け加える方がよっぽど大きくなるんですね、実は。
(a→b→c→d)→(a→b→c→d)→(a→b→c→d)→(a→b→c→d)よりも
a→b→c→d→3のが大きいってことです
416 :
2016/06/19(日) 22:31:26.14 ID:TtlfQr5R
多変数アッカーマン関数もBEAFの線形配列も方や多重リスト、方や多次元を使ってω^ωからε_0(またはω↑↑ω)に辿り着いているんですがね
417 :
132人目の素数さん
2016/06/19(日) 22:34:40.34 ID:TtlfQr5R
間違えた 方や→片や
とりあえず多重リストの定義を見せて頂きたい
418 :
2016/06/19(日) 22:55:11.04 ID:9fofoY7/
>>415
a,b>2の場合に限る
419 :
2016/06/19(日) 22:56:20.62 ID:BdVKliKM
多次元じゃなくてテトレーション配列じゃないの?
あと細かい話だけど、多変数や線形配列の一種として多重リストや多次元
という言葉があるわけではなく、それぞれ独立した配列方法や空間の名前で
あって、線形配列が多次元を使って??とかいう表現はまずいかと。
420 :
2016/06/20(月) 04:13:39.68 ID:D80DjIqF
確かにそうでした。
線形は1次元なので多次元ではないですね。
それと「辿り着く」は不適切でした。正確には・・・うーん・・・
421 :
2016/06/20(月) 23:06:26.50 ID:BGOvliGM
「に相当する」とか「の強さがある」とか?
422 :
2016/06/20(月) 23:54:16.37 ID:D80DjIqF
>>421
ああ、そうです!それです!
423 :
132人目の素数さん
2016/06/21(火) 17:55:04.26 ID:EXkPl6sF
>>413
いいでしょう、超次元を説明するためには、どのように立方体が出来るかそのメカニズム説明しなければならない
まずバウワーズ氏はこのようにして立方体を作っているんだ
まず配列次元演算子をこのように定義してる
424 :
2016/06/21(火) 18:19:32.94 ID:EXkPl6sF
b&a={b,b,b,・・} a変数のb
つまりa&bではなくb&aなんだよ
でもこれは違うよ
3^3&3と27&3
でっ、3^3&3をこのようにして立方体を作る
(1)は改行、(2)は次の平面、(3)は3次元へ(4)は四次元へ、(n)でn次元へ
それでこのようにして立方体が出来る
{3
425 :
132人目の素数さん
2016/06/21(火) 20:49:04.07 ID:EXkPl6sF
b&a={b,b,b,・・} a変数のb
つまりa&bではなくb&aなんだよ
でもこれは違うよ
3^3&3と27&3
でっ、3^3&3をこのようにして立方体を作る
(1)は改行、(2)は次の平面、(3)は3次元へ(4)は四次元へ、(n)でn次元へ
それでこのようにして立方体が出来る
{3,3,3(1)3,3,3(1)3,3,3(2)3,3,3(1)3,3,3(1)3,3,3(2)3,3,3(1)3,3,3(1)3,3,3}
これで3次元の立方体が出来る
壁壁壁壁壁壁壁
壁3,3,3壁
壁3,3,3壁
壁3,3,3壁
壁壁壁壁壁壁壁
↑この奥に同じようなものが2層広がってる
426 :
2016/06/21(火) 20:58:34.14 ID:shTmSKB6
(n)でn次元ならなぜ>>389で3次元の域を抜けてないってことになるんだ?
基準がちがうのか?よくわからん。
427 :
132人目の素数さん
2016/06/21(火) 22:16:03.25 ID:EXkPl6sF
>>426
それはね、縦横奥にしか広がって無いからなんだよ
バウワーズ氏は3次元立方体をルービックキューブ状に拡大して巨大数を作り出しているんだ
428 :
2016/06/21(火) 22:22:03.68 ID:shTmSKB6
ルービックキューブは図示するための方便じゃないの?
実質多次元なんでしょ?
429 :
2016/06/21(火) 22:58:47.44 ID:f0LFv6F0
レギオン配列だって書くだけなら1行で書いてしまって「はい線形配列」と
言ってしまえるわけで、言葉の意味が本質的に理解できていないんじゃないか。
430 :
2016/06/21(火) 23:08:22.44 ID:f0LFv6F0
そもそも次元だの空間だの、そしてフラクタルだのというのはあるレベルに
達すると薄っぺらい表層になってしまう。バシク行列はおろか、Cにすら到底
及ばないよ。
431 :
2016/06/22(水) 06:20:44.71 ID:t+PrEYZ4
ところで、バシク行列は(1,?)のつぎは(2,?)が来るから多相型の性質を持ってて、型も値も同じく行列の一部だから型演算の性質も持っている
→バシク行列の強さはλω
これでいいんだっけ?
あと、C関数(二階算術)<バシク行列(λω)ということは二階算術<λω?

システムの証明論的強さ(よくわからない)が強いほど大きな順序数を作るのに使えるということで合ってる?
432 :
2016/06/22(水) 14:17:30.20 ID:+YXDptHE
3次元をルービックキューブ状に拡大と言うんなら、
1次元の時点でフラクタル状に拡大と言ってしまってもいいんだぜ。
フラクタルと次元を組み合わせればより強力にって考えを抱いてるのかもしれんが、
無駄に複雑さを増すだけだ。

>431
算術と述語論理じゃ述語論理のほうが表現力豊かだからそれだけ
強力なシステムを作りやすいってことじゃないかな?
433 :
2016/06/23(木) 21:42:23.51 ID:tqdQH9fb
ヒドラの動きがいまいち理解できない。
入力が小さいときは多重ループと同じに見える。
入力が大きいと分けわからんくなるし、
どうやって理解すればいいんだ?
434 :
2016/06/25(土) 20:32:02.08 ID:3hJAxVQr
ライフゲームで、n個の生きているセルから始まり
滅亡またはループする時の生きているセルの数。
435 :
2016/06/25(土) 20:44:20.40 ID:3ZXEn0f1
一つのセルから全力で広がったとしても
n世代目で(2n+1)^2以内に収まるから大したこと無いんじゃないか?
436 :
2016/06/25(土) 21:09:22.75 ID:XT41Vpfb
ライフゲームはチューリング完全だろ
437 :
2016/06/26(日) 00:21:30.86 ID:tGJcm4mp
ライフゲームを高次元化して面白いの作れないかな
438 :
132人目の素数さん
2016/06/26(日) 00:21:54.47 ID:xtKqPYJs
>>429
そうだよね、レギオン空間がL空間になるんだよ
L1空間はレギオン、バウワーズ氏は厳密な定義を与えていないから、解析が不能なんだよ
439 :
132人目の素数さん
2016/06/26(日) 00:26:59.69 ID:xtKqPYJs
>>437
3dライフゲームでググってみな、面白いものが見れるよ
440 :
2016/06/26(日) 00:34:33.03 ID:tGJcm4mp
>>439
アレは見てもわかんねぇ

あれのn次元版とかならそこそこ強そうな数吐き出してくれないかな
441 :
2016/06/26(日) 01:33:06.18 ID:uAsbwqgv
もしかして>>429>>428への返信だと思ってる?
442 :
2016/06/26(日) 04:09:12.37 ID:EDzSnEef
>>434に触発されて
En = 無限に増えないn個のセルの配置の全ての世代の集合
count(w) = wの生きているセルの数
L(n) = max{count(w)|w∈En}
L(0) = 0
L(1) = 1
L(2) = 2
L(3) = 4
L(4) = 20
L(5) = 319
L(7) >= 1088
L(8) >= 1283
L(9) >= 2117
L(12) >= 4100
L(18) >= 4678
L(31) >= 6410
443 :
2016/06/26(日) 17:03:14.12 ID:GVeuf1dH
終了する := 以前の世代の配置が次の世代で現れる
そういえば、334個のセルから始まってグラハム数ステップ後に終了しても10ステップ後に終了してもどちらもL(334)なんだよね。
だから何ステップかかるかはあまり関係ない

#ライフゲームでプログラムするのはどうやればいいんだろうね
444 :
2016/06/26(日) 18:09:20.62 ID:uAsbwqgv
まず自然言語を(できるだけ)ライフゲームの形式的な表現に翻訳します。1とか2とか
あとはビッグバンが起こるのを待ちます。

計算可能な関数でもn文字のペアノ算術で定義可能な、とかしたらε_0
の関数ができます。たぶん
445 :
2016/06/26(日) 18:15:37.06 ID:uAsbwqgv
1:=■□ 2:=■■□
こんな具合に
446 :
2016/06/26(日) 19:04:07.58 ID:VtqYV0Sm
>>436
計算可能関数になるよね。
447 :
2016/06/26(日) 19:04:34.73 ID:VtqYV0Sm
ミスった恥ずかしい。

× 計算可能
○ 計算不可能
448 :
2016/06/26(日) 23:46:15.41 ID:GVeuf1dH
ライフゲームの中で使いやすいように定義するのが一番いいと思うな
たぶんそうすれば数が大きくなるに従って使うセルの数も増えていくだろうし

■ ■ ■
■ ■ ■
■ ■ ■
が3とかかな?
449 :
2016/06/26(日) 23:52:24.45 ID:GVeuf1dH
こりゃだめだな
■■ ■■ ■■
■■ ■■ ■■
というようにしよう
まあ後はおいおい
450 :
2016/06/27(月) 02:54:08.83 ID:lyCTYJez
□■□□□
□□■□□
■■■■■
□□□■■ (グライダーの先端が2*2四角に横から接している)
は、3ステップで消える。また、2*2四角を構成するマスの1つの上下左右いずれかに1つの生きたマスが接触しているときも消える
451 :
2016/06/27(月) 18:23:41.80 ID:OTxUOlAs
グライダーガンみたいな無限増殖パターンはどうなるんだろう
マックスとかも計算しだいでは強そう
452 :
2016/06/28(火) 05:58:40.47 ID:3CuR2eGK
永遠にランダムな増殖をするパターンってないのかな
ランダムは円周率の桁から取ってくるみたいな感じで
453 :
2016/06/28(火) 17:58:08.28 ID:9H8LnsIM
チューリング完全だから無理数の計算をさせればいいんじゃない?
454 :
2016/06/28(火) 21:00:02.05 ID:pww1LGSL
ランダムの定義をいってみろや
455 :
2016/06/28(火) 21:16:52.50 ID:3CuR2eGK
ランダムはランダムだよ
たとえば区域[0,14]の中から等確率でランダムとかだよ
円周率なんかの桁は0から9の数字が完全にでたらめでほぼ等確率だから外部の入力なしで完全な乱数を実現できるかなと思って。
でも√2なんかでもいいだろうけどね
456 :
2016/06/28(火) 21:41:06.27 ID:/dGkYX0Y
チャイティンがランダムについて考えてたっけ。そんな単純な話じゃないよ
457 :
2016/06/28(火) 21:46:29.50 ID:3CuR2eGK
>>456
wikipedia見てみた。各桁が計算不可能とかなにそれヤバイ
まあどんな規則にも従わないっていうと計算するアルゴリズムが存在しないってことだし計算不可能になるのかな
458 :
2016/06/28(火) 22:08:27.13 ID:3CuR2eGK
F[a,f](b) ∈ S[n] かつ F[a,f] ∈ S[n+1] ここで a∈S[n] かつ f∈S[n+1]
0∈S[0], suc∈S[1] (sucは後者関数)
a>b⇔F[a]>F[b], a=b⇔F[a]=F[b]
459 :
2016/06/28(火) 22:19:19.88 ID:3CuR2eGK
ちょっと間違えた
F[a,f](b) ∈ S[n] かつ F[a,f] ∈ S[n+1] ここで a∈S[0] かつ f∈S[n+1]  (微修正)
0∈S[0], suc∈S[1] (sucは後者関数)
a>b⇔F[a,f]>F[b,g], a=b∧f>g⇔F[a,f]>F[b,g], a=b∧f=g⇔F[a,f]=F[b,g]  (修正・追加)
(a>0∨f≠suc)∧f∈S[1] ⇒ F[a,f] > suc

F[a,f](b)はa∈S[0]かつf∈S[n+1]のときにS[n]からS[n]への写像として定義される
fがS[0]のときには定義されない

F[0,suc](n)=ω・(1+n)
F[0,suc](F[0,suc](n))=ω^2・(1+n)
460 :
2016/06/28(火) 22:20:04.43 ID:3CuR2eGK
あ、suc(a)∈S[0]で
461 :
2016/06/28(火) 22:33:01.43 ID:3CuR2eGK
F[1,suc](α)=ω^ω・(1+α)
F[1,suc](F[1,suc](α))=ω^(ω2)・(1+α)
F[ω,suc](α)=ω^ω^ω・(1+α)
F[0,F[0,suc]](α)=ε_0・(1+α)
F[0,F[1,suc]](α)=ε_0^ω・(1+α)
462 :
2016/06/29(水) 08:17:14.65 ID:kScr29LW
>>443
ライフゲームとは別のWireWorldというセルオートマトンだが素数計算機を作った人がいる
http://www.quinapalus.com/wi-index.html

Gollyってソフトにこれも含めて色々おもしろいルールのサンプルが入ってる
このスレ的に使えそうなものがあるかはわからないが
463 :
2016/06/30(木) 06:18:50.92 ID:2GhS42O3
464 :
2016/06/30(木) 20:06:36.72 ID:cwYII7fV
42億x42億マスってどうやって情報もってるんだ?
相当効率よくやらないとだめだろ。
465 :
2016/06/30(木) 21:30:24.20 ID:G3wNdFpR
適宜情報を持ってくる必要がある範囲内だけで計算をして、その上限が42億
ってことじゃないの
466 :
2016/06/30(木) 21:46:17.75 ID:7UEYnl3F
疎(sparse)行列だな
いろいろと有用だから処理方法は研究されている
467 :
2016/06/30(木) 23:39:30.59 ID:2GhS42O3
WireWorld
□blank(空) ずっとblankのまま
■copper(銅、銅線?) electron headと接触すると次のステップでelectron headになる
●electron head(電子?) 次のステップでelectron tailになる
◆electron tail(電子の尾?) 次のステップでcopperになる

copperの導線をelectron headが流れる

●■■■ → ◆●■■ → ■◆●■ → ■■◆● → ■■■◆ → ■■■■
468 :
2016/07/01(金) 18:19:10.98 ID:iH+JT5lo
おそらくn文字以内のプログラムで出力される数の大きさを競うようなゲームは
コンピュータが人間より苦手とする分野じゃなかろうか?
ディープラーニングも有効じゃなさそうだし、コンピュータはロジックが苦手と言うし。
loader.cを超えるプログラムを作れる人工知能をだれか開発してみてほしい(他力本願寺派)
469 :
2016/07/01(金) 19:11:07.82 ID:MLN8KfMq
ロジックが苦手なのは土台がないからでは?
どうにかしてロジックを得意にするための土台をつくれば・・・
470 :
2016/07/01(金) 20:20:45.95 ID:iH+JT5lo
ロジックが苦手というのはponanzaの確か作者が言っていたことで、コンピュータは
かえって大局感のほうが優れていて、細かい詰みや囲碁で言う寄せが絡むような
局面は苦手なんだと、いやそこまでは言ってなかったっけ

巨大数対決も、たとえば関数→算術→論理っていうふうに土台そのものをどんどん根本から
作っていくゲームと捉える事ができるからコンピュータはだいぶ苦手だろう、
と勝手に考えてる。
471 :
2016/07/01(金) 20:27:47.15 ID:6wd9HSTR
アルゴリズムの自動生成の課題としては面白いかもね巨大数
ある順序数を超える巨大関数を定義出来たらPvsNPが解けるとかあればなお面白いのに
472 :
2016/07/01(金) 20:33:07.06 ID:iH+JT5lo
究極最初から、人間がloader.cを超えるプログラムを作って、あとはそれを人工知能に
出力させる、という風にすればそりゃ超えたことになるけど、問題はそこじゃなくて、
命題論理から述語論理、さらにOodle Theoryへと抽象的な方法論を自力で開発できるのか、
開発するということはどういうことか、コンピュータがそれを理解するとはどういうことか、
我々人類が「コンピュータが理解した」と解釈するということはどういうことか・・・

だんだん人工知能は意思を持ちうるのかとか自由とは何かとかいう問題になってきます。
473 :
2016/07/01(金) 22:08:16.47 ID:MLN8KfMq
それを開発するとか理解するとかは関係なくて
(そもそも理解という言葉を厳密に定義するのはきっとすごく難しいだろう)
それを使っているかで判定すればいいのでは
とはいえぱっと見じゃあloader.cだってCalculus of Constructionsを使っている風には見えないし
そこはどうやって判定すればいいんでしょうかね
474 :
2016/07/01(金) 22:09:39.18 ID:MLN8KfMq
使う・・・そのプログラムの構造が出力する数の大きさに貢献していて、その構造にはあるアルゴリズムを使っている
475 :
2016/07/01(金) 22:16:58.35 ID:iH+JT5lo
1+1=2だってむりやりもすごいシステムの一部分だと解釈することも可能といや可能なわけで、
最終的には他我の存在を受け入れるように直観的に無条件で認めることになります。

ただ今まで人間の子供に勉強を教えて、教えたことを使っているかとかで理解できたか
どうかを判断してきたことを、人工知能に対してもにも同じように解釈して
いいのか、という疑問がわきます。

そろそろスレ違いか
476 :
2016/07/01(金) 22:48:39.09 ID:iH+JT5lo
googologyとしては、ねらってものすごい概念を対角化するか、適当にプログラム
書いてたまたまものすごい概念の対角化にぶち当たるのをねらうか、の違いがある。
たまたまのほうは場合の数と、そしてなにより計算の複雑さの関係でどんなに人工知能が発達しようが
現実的でないと思うけど。

プログラムの評価方法をディープラーニングでもなんでもつかって向上させるとしても、
その証明自体が巨大数を生成する計算の上に成り立つもんだし。
477 :
2016/07/01(金) 23:58:55.20 ID:aAHCsSfV
巨大数 VS 無限の猿定理
478 :
2016/07/02(土) 02:24:40.36 ID:mMvgynM3
Bignum Bakeoffのソースコードは空白除けばだいたい256^513通りしかないんだよね
1コードテストするのに1マイクロ秒としても10^1222年ぐらいかかるけど
479 :
2016/07/02(土) 02:42:09.28 ID:SllYu80r
プログラムで巨大数の比較って可能なのか?
480 :
2016/07/02(土) 02:44:13.07 ID:mMvgynM3
あ、確かに無理だな
計算不可能だった
481 :
2016/07/02(土) 02:45:22.97 ID:Z7Nczarp
停止性問題は根本的に解決できないし、
停止したとしてもメモリが十分ないとおそらく比較不可能
482 :
2016/07/02(土) 02:56:19.51 ID:mMvgynM3
停止性問題をロジック+総当たりで解ける人工知能は・・・
483 :
2016/07/02(土) 02:57:05.07 ID:mMvgynM3
論理で解の空間をバッサバッサ切り落としていけばいつかは総当たりしても現実的に解けるようになりますよね
484 :
2016/07/02(土) 03:05:59.44 ID:SllYu80r
意味の無いプログラムや計算可能な範囲を除外したら何割くらい残るんだろう
485 :
2016/07/02(土) 15:54:03.15 ID:G6kX9+1l
5-状態のビジービーバーさえ求まっていない状況で、解の空間を切り落とすなんて相当難しいことだ。
486 :
2016/07/02(土) 19:11:26.04 ID:TaC/KPE+
遺伝的アルゴリズムならいけると思ったが停止判定ができないな
487 :
2016/07/02(土) 21:44:32.07 ID:zZOphvey
>>482
「停止性問題を解ける」と仮定すると簡単に矛盾が導けるのに、なんなのこの楽観脳。
488 :
2016/07/02(土) 22:31:04.04 ID:G6kX9+1l
一つ下のレベルの停止性問題を解けるって言いたいんじゃないの。
だとすると自分と同じレベルのプログラムは作れないということになって、
原理的に自分を開発した人間を超えられないということになるけど、
神がかった奇跡でも起きない限り。
ディープラーニングより本質的に人間の思考を掘り下げないとだめっぽいな。
何気にこれは時代を先取りした課題なのかも知れない。
489 :
2016/07/02(土) 22:45:37.64 ID:G6kX9+1l
どんなに複雑な停止性問題でも、証明を機械的な作業、つまり計算に置き換えることができれば、
偶然その証明、あるいは反証ができてしまう可能性は常にある。

ただ証明の文字列をメモリに出力するだけじゃダメで、それがある停止性問題の証明であることを
理解して、その停止することが証明された複雑で強力なプログラムを出力して停止しなければならない。

本当に理解したかどうかは他我問題になって数学的に扱えないけど、停止性の証明を
要求されれば先ほどの証明の文字列を出力して、あとはそれを人間が吟味して
本当に証明になっているのかどうかを確かめればいい。

結局証明の扱い方の時点で神がかった奇跡に頼ることになるか
490 :
2016/07/02(土) 23:20:15.01 ID:Z7Nczarp
5次方程式の解の公式がないように
5状態ビジービーバーの停止性の証明も無いとかってことはないのかな
491 :
2016/07/02(土) 23:44:39.32 ID:G6kX9+1l
証明の存在の有無は論証体系の強さの問題でしょう。
そして最大の順序数が存在しないように最強の体系も存在しない。
不完全性定理からもそのことは証明できる。

解の公式の存在も、正確にはある体系で解けないというのであって、
方法の幅を広げれば公式を作る体系とか表現する言語とかに依存することがわかる。
492 :
2016/07/04(月) 03:11:12.46 ID:0ZmLctzH
バシク行列が未だによくわからないので作った
X: 0個以上の自然数, Y: 0個以上の0, n: 自然数
1. (Y) :: On
2. (X,n+1,Y) :: (X,n,Y) -> (X,n,Y)

(0)=0
(1)(0)=1

(1)(α) = α+1
(2)(1)(α) = α+ω
(2)(2)(1)(α) = α+ω^2
(3)(2)(1)(α) = α+ω^ω
(1,0)(α) = α+ε_0
(2)(1,0)(α) = α+ε_0*ω
(2)(2)(1,0)(α) = α+ε_0*ω^2

バシク行列と同じ強さになればいいな・・・
493 :
2016/07/04(月) 20:17:20.39 ID:e5A153Zr
前も書いたと思うけど、巨大数をはかるものさしを弱い方から

0.桁
1.演算
2.関数
3.順序数
4.算術
5.論理

計算不可能レベルでも対角化の範囲に計算不可能だったり矛盾していたり
するものを許容することで応用がききそう。
論理以降は哲学の領域で数学で整理しようとするのはナンセンスかな
494 :
2016/07/04(月) 21:53:25.75 ID:ipgn7uj8
算術って何が該当するの?
495 :
2016/07/04(月) 21:55:57.65 ID:lwP0CPv1
ペアノ算術とか?
496 :
2016/07/05(火) 00:39:00.11 ID:CdkayJmN
4.と5.について読んだらいい本はありますか?
497 :
2016/07/05(火) 06:08:46.07 ID:CdkayJmN
検索したところこれを無料で読めるようなので読んでみようと思いました
http://projecteuclid.org/euclid.pl/1235415519#info

算術的階層, 算術的強制法, 超算術的階層, 帰納作用素, 解析的階層, 記述集合論, 有限型の計算論, 順序数上の計算論がわかるそうです
498 :
2016/07/06(水) 19:15:46.49 ID:VBifEcDi
RCA_0 1〜5までロビンソン算術, 6はΣ^0_1-帰納法, 7はΔ^0_1-内包公理
1. x+1≠0
2. x+1=y+1→x=y
3. y=0∨∃x[x+1=y]
4. x+0=x, x+(y+1)=(x+y)+1
5. x・0=0, x・(y+1)=x・y+x
6. 任意のΣ^0_1-論理式φ(n)に対し, φ(0)∧∀n[φ(n)→φ(n+1)]→∀n[φ(n)]
7. Xを自由変数にもたない任意のΣ^0_1-論理式φ(n),ψ(n)に対し, ∀n[φ(n)↔ψ(n)]→∃X∀n[n∈X↔φ(n)]
499 :
2016/07/06(水) 19:25:41.78 ID:VBifEcDi
3.は0以外の自然数には前者があるという意味
4.は自然数の和の定義
5.は自然数の積の定義
6.は0がある性質を満たし、あるのnがその性質を満たすときn+1もその性質を満たすなら、すべての自然数にその性質があてはまると言っている
7.はすべての自然数が性質φと性質ψを同時に満たすとき、性質φを満たす自然数すべての集合Xが存在するという

Σ^0_1-論理式は∃x∀y<z[原子式]という形の式
500 :
2016/07/06(水) 19:56:24.44 ID:p1odMzWs
Σ^0_0論理式が有界量化子しかもたない論理式で、Σ^0_1はΣ^0_0に外側から
存在量化子を加えた論理式。若干違う。

7はφを満たさなければ必ずψを満たす、つまり有限の時間で任意の項がある集合に
含まれているかどうかを確かめることができる、ってことじゃなかった?
501 :
2016/07/06(水) 22:21:20.73 ID:VBifEcDi
>>500
そうなのか、ありがとう。
でもΣ^0_1はΠ^0_0に存在量化子を加えた式では?
すみません、7については少しよくわかりません
502 :
2016/07/06(水) 22:59:36.80 ID:VBifEcDi
ところでΔ^0は自然数の量化、Δ^1は集合の量化ですがΔ^2は何になるんですか?
503 :
2016/07/06(水) 23:00:46.05 ID:p1odMzWs
Π^0_0もΣ^0_0も同じです。有界量化子は有限の範囲でしか量化しないので、
なくても命題論理で表現できる・・・という理由だと思うけど。
504 :
2016/07/06(水) 23:19:34.39 ID:VBifEcDi
同じだったんですか!
確かに∀x<2[x∈X]は(0∈X∧1∈X∧2∈X)で表現できますし
∃x<2[x∈X]は(0∈X∨1∈X∨2∈X)と表現できますね
505 :
2016/07/06(水) 23:20:09.31 ID:VBifEcDi
あ、2∈Xは間違えました
506 :
2016/07/07(木) 22:00:58.01 ID:KqsRmXC/
>>490
完全性定理よりある言語で表現可能で真ならば証明可能であることが保証されている。

φ(a,n) でnに0,1,2,...を代入していって真となるものがあれば集合Aに含まれる。
ψ(a,n) でnに0,1,2,...を代入していって真となるものがあれば集合Aに含まれない。
∃nφと∃nψが同時に真になることはない。こういうこと。
507 :
2016/07/08(金) 00:37:19.76 ID:dmxLqYcV
すべての自然数についてφ(n)とψ(n)が同値でないなら、{n:ψ(n)}は自然数全体ではないということか

・・・∀nφ(n)かつ?(φ(n)⇔ψ(n))かつ∀nψ(n)であるようなψ(n)は存在しない
・・・∀nφ(n)になるような論理式はすべて同値

これで合ってる?
508 :
2016/07/08(金) 23:08:29.24 ID:Z0ddIe7W
{n:ψ(n)}はψによって自然数全体にもなりえます。
言葉足らずだったけど、含まれる含まれない言ってるのはaのことです。
509 :
2016/07/08(金) 23:26:34.06 ID:dmxLqYcV
ああ、そうでした
{n:φ(n)}が存在しない場合というのは?
510 :
2016/07/08(金) 23:30:28.71 ID:dmxLqYcV
あれ、もしかして∃X∀x[x∈X↔φ(x)]だけでよかったんですかね?

φ(n)↔ψ(n)のところでなぜいきなりψが出てくるのかわからなかったんですが
511 :
2016/07/09(土) 03:09:31.86 ID:omnVS07d
∀n[φ(n)↔ψ(n)]→∃X∀n[n∈X↔φ(n)]
これが、
∀n[φ(n)↔?ψ(n)]→∃X∀n[n∈X↔φ(n)]
の間違いで、論理的な推論だけで
∃X∀n[(?n∈X)↔ψ(n)]
が導出される。
φ(n,0),ψ(n,0),φ(n,1),ψ(n,1),...
と論理式を並べることで必ず有限の時間で真の論理式にたどり着き、
φが真なのかψが真なのかでnが集合に含まれるかどうかを判別できる。
512 :
2016/07/09(土) 03:30:38.63 ID:wx9DQMsE
そのψ(n)の意味ないだろって言ってるんでしょ
結論とは無関係なのに仮定にだけ現れるψ(n)
513 :
2016/07/09(土) 11:08:35.47 ID:omnVS07d
ψがなかったら有限時間でnが含まれないという判断ができない
514 :
2016/07/10(日) 15:32:48.29 ID:cihgiFNu
ああ、∃X∀x[x∈X↔φ(x)]はACA0だけどこれを有限時間以内で判定できるもののみに制限したのがRCA0なのか
515 :
2016/07/11(月) 20:57:22.75 ID:0XqATsgc
CoqやってみたらCoCがちょっとわかったような・・・
516 :
2016/07/14(木) 21:52:19.50 ID:38lO2u3R
ιコンビネータを利用してチューリング完全な原始数列を作れるけど、あまり面白くないからやめた。

自然数をなんらかの方法でバシク行列とその媒介変数に変換B(n)

B(n)=nとなればバシク行列版の完全数
B(m)=n,B(n)=mで友愛数
B(l)=m,B(m)=n,B(n)=lで3つ組の社交数

バシク行列じゃなくてもいろいろ応用が効くな
517 :
2016/07/18(月) 14:26:52.67 ID:VFOkPJlR
f(α) = ψ(α)
f(α,β) = φ(α,Ω_β)みたいな
...

ψ(α)=ε_α iff α<Ω、γ+δ,γδ,γ^δ
f(0)=ε_0
f(0,1)=Ω
f(Ω)=φ(2,0)
f(Ω+1)=ε_(φ(2,0)+1)
f(Ω2)=φ(2,1)
f(Ω^2)=φ(3,0)
f(Ω^ω)=φ(ω,0)
f(Ω^Ω)=Г_0
f(f(1,1))=f(ε_(Ω+1))=ψ(ε_(Ω+1))
518 :
2016/07/19(火) 23:54:55.09 ID:3NM3FTWj
メモ 定義はDeedlit11/Ordinal Notations IIIより
・θ関数
C_0(α,β)=β∪{Ω_ν}, ν≤ω
C_n+1(α,β)={γ+δ,φ(γ,δ),θ(η,γ)|γ,δ,η∈C_n(α,β); η<α}
C(α,β)=∪[n=1][∞] C_n(α,β)
θ(α,β)={γ|γ≠θ(α,δ); δ<β}
・ψ関数
C_0(ν,α)=Ω_ν∪{0}・・・Ω_nはψで到達できないn番目の順序数未満の順序数の集合?
C_n+1(ν,α)={β+γ,φ(β,γ),ψ_μ(δ)|β,γ,δ∈C_n(ν,α); δ<α; ν≤μ≤ω}
C(ν,α)=∪[n=1][∞] C_n(ν,α)
ψ_ν(α)=min{β|β∉C(ν,α)}

θ(α,β)=φ(1,α,β)
θ(Ω+α,β)=φ(2,α,β)
θ(Ωω,0)=φ(ω,0,0)
θ(Ω^2,0)=φ(1,0,0,0)
θ(Ω^3,0)=φ(1,0,0,0,0)
θ(Ω^ω,0)=φ(1,0,...,0)
θ(Ω^Ω,0)=大ヴェブレン順序数
θ(ε_(Ω+1),0)=バッハマン・ハワード順序数
519 :
2016/07/25(月) 15:12:41.44 ID:B2ok11C+
上のラヨ関数の議論について

ある文字列が0を命名している、という主張
0を命名するためには0の存在が保証されていなければならない、という反論
命名する論理式のモデルで考える。
そんなモデルの存在が保証されていなければならない、という反論。
そんなモデルの存在を許す超モデルで考える。
そんな超モデルの存在が保証されていなければならない、という反論。
そんな超モデルの存在を許す超超モデルで考える。
以下、いたちごっこ
520 :
2016/07/25(月) 15:35:09.85 ID:B2ok11C+
具体を議論対象とする主語論理的な考え方がいつまでたっても交わらない並行線をつくっている。
ラヨ命名は抽象的な概念に名前をつけるので、その主語となって真の論理式
をつくる具体的な対象の存在を問わない。
問題があるとすれば、「0である」と「1である」が同時に真となってしまうような場合だが、
述語論理自体は無矛盾であるはずなので、体系の問題となる。
521 :
2016/07/25(月) 18:23:04.03 ID:B2ok11C+
主語論理とか述語論理とかいうのはあまり関係なかったな。
「○○は0である」というのが主語論理的な定義で、
「0は○○である」というのが述語論理的な定義。
522 :
132人目の素数さん
2016/07/25(月) 23:14:25.86 ID:K4kT85Qj
じゃあ0の定義って何だろう
後者関数f(x)と自然数の集合があったときにf(n)=0となる自然数nが存在しないのが0?
527 :
132人目の素数さん
2016/07/25(月) 23:17:09.37 ID:K4kT85Qj
算術の体系にもよるかな?
でも必要な体系ってどんなの?ペアノ算術?ロビンソン算術?
538 :
2016/07/27(水) 23:00:33.89 ID:ioTLIUEw
俺予想
ビジービーバー関数Σ(n)がある数k以下であることを証明するための文字列長はほぼΣ(n)と等しくなる。
539 :
2016/07/27(水) 23:36:01.33 ID:Ntp21Ncc
繰り返しになるけど、具体的な存在でなく抽象的な性質で0を定義する
のがラヨ命名であり、等号に関する公理より0であれば0以外の何物でもないことは自明となる。
矛盾する論理式は除くというルールが受け入れられるか否かがあれだったんだな。

ビジービーバー関数
Σ(4)を求めるプログラムとかどうだろう。
540 :
2016/07/28(木) 00:20:10.08 ID:guseiPTx
>>538
RCA_0上で証明するのならそうなるかもね
いや、log_a(Σ(n))かも?大して変わらないけど
541 :
2016/07/28(木) 07:32:50.53 ID:ikldiUfA
Σ(n)=mを証明するのとだいたい同じだな。
542 :
132人目の素数さん
2016/07/29(金) 19:23:58.64 ID:5IAyzyJa
・0
S(0)=nになるようなnは存在しない

・ωやその他の極限順序数
S(ω)=nになるようなnは存在しない
最も小さい極限順序数

・後続順序数
すべての順序数は0,極限順序数,後続順序数のいずれか
553 :
2016/07/30(土) 14:21:31.98 ID:VcIGl5QJ
非可算無限個の0のリストを考える
f(L,α)はLのα番目の数字を1にする

f([],n) = 2^n
f([],ω+n) = ω*2^n
f([],ω2) = ω^ω
554 :
2016/07/30(土) 14:41:06.12 ID:VcIGl5QJ
f([],α,n)はLのα番目の数字をnにする と変えると
f([],0,n) = n
f([],1,n) = ω*n
f([],2,n) = ω^2*n
f([],ω,n) = ω^ω*n
f([],Ω,n) = ε_0*n
555 :
132人目の素数さん
2016/07/30(土) 14:45:03.86 ID:VcIGl5QJ
自然数に限らず"対応する記法が定義されている"順序数すべてにすると
はじめに0,1を使えるとして
f([],0,x)<ω
f([],1,x)<ω^ω
f([],2,x)<ε_0
f([],3,x)<η_0
・・・
f([],ω,0)=Г_0
568 :
2016/08/05(金) 11:56:20.14 ID:7765ZinM
順序数崩壊関数が無矛盾であることの証明はどこまで得られているんだろう。
ψ(ε_{Ω_ω+1})以降の定義もwell-definedなものがない気がする。
569 :
2016/08/08(月) 14:16:19.53 ID:pABgURtU
570 :
2016/08/09(火) 16:42:31.97 ID:hhMxjvVD
順序数崩壊関数はそれぞれの順序だけ書いて、一般的な厳密な定義は各自挑戦してもらうしかないかな。
Π^1_1-TR_0は無矛盾性の証明が得られていて、その証明論的順序数はψ(ψ_I(0))でおk?
571 :
2016/08/10(水) 17:35:19.90 ID:1eALUFcx
順序数に関する良い入門テキストないですか?
ちゃんと勉強したい。
572 :
2016/08/10(水) 19:04:09.71 ID:hdie6XP5
算術か集合論か証明論か基礎論か、そのへんのテキストを片っ端から読みあさっていけばいいのでは?
順序数は目的というより手段で、それ単体を勉強するものではないかもしれない。
573 :
2016/08/10(水) 21:59:35.52 ID:nivHVKS2
公理的集合論のテキストの最序盤で、順序数として自然数を構成したら、その流れで超限数まで行って一通りやってないかな。
574 :
132人目の素数さん
2016/08/11(木) 07:29:53.28 ID:/QGemoR4
http://www.math.tsukuba.ac.jp/?tsuboi/und/14logic3.pdf
あったよ
587 :
2016/08/11(木) 10:48:15.69 ID:xJrzC3S3
>>574
ありがと〜読んでみるよ
600 :
2016/08/11(木) 21:22:53.59 ID:jEzp6YLu
くわしいことは分かってないネラーの戯言

高階述語論理に相当する表現力をもった言語ではじめて記述することができる理論
を使っている時点でloader.cはフリードマンのいくつかの関数を超えてる気がする。
601 :
2016/08/11(木) 21:34:08.49 ID:jEzp6YLu
TaranovskyがCはZFCとさらにその向こう側を超えてるかもしれないと記してるし、
バシク行列とloader.cは欲張りクリークの関数に匹敵するか、あるいは超えてるかもしれない。
602 :
2016/08/13(土) 17:07:03.72 ID:pCPQudc/
強いのはわかったけどいまいちわかりにくいんだよなー
対応するわかりやすい仕組みとかないの?
603 :
2016/08/13(土) 22:54:56.11 ID:WRd6IG5V
順序数で追っていくのが厳しくなってくると理論で強さをはかるのがよろしいかと。
上では算術と書いたけど理論のほうが適切かな。
フリードマンの分類ではZFC+巨大基数公理という方法をとっている。
604 :
2016/08/14(日) 00:51:52.88 ID:Y32S+jAy
>>603
?クス
605 :
2016/08/14(日) 15:29:14.26 ID:TORfRBug
強配列表記って、hyp cos氏のtalkページ見る限りCを超えてないようだけど、どうなの?
606 :
2016/08/15(月) 01:52:30.28 ID:UVLUMjul
と、hyp cos が考えている、という以上のことは誰にもわからん
607 :
2016/08/15(月) 16:54:00.46 ID:ikxD5NBF
強配列表記がCを超えてると『巨大数論』に書かれてるけど、ソースはどこだろう?
608 :
2016/08/15(月) 16:54:16.51 ID:uYUzU44H
https://en.wikipedia.org/wiki/List_of_large_cardinal_properties
これか。Worldly cardinalってなんぞ
609 :
2016/08/15(月) 18:10:31.22 ID:ikxD5NBF
階層V_kがZFのモデルとなるような基数と書かれている
610 :
2016/08/15(月) 22:36:18.11 ID:uYUzU44H
Cantor Atticのをすこし訳してみた

基数κがworldlyであるとはV_kがZFCのモデルであることをいう。つまりκは強極限であり、ベス不動点かつそれらの列挙の不動点、さらに・・・である。
・到達不能基数はworldlyである。
・にもかかわらず、最小のworldly基数は特異であり到達不能ではない。
・最小のworldly基数は共終数がωである。
・Indeed, the next worldly cardinal above any ordinal, if any exist, has cofinality ω.
・Any worldly cardinal κ of uncountable cofinality is a limit of κ many worldly cardinals.

Degrees of worldliness
基数κは、それ自身がworldlyかつworldly基数の極限であれが1-worldlyである。さらに一般的に、κがworldlyかつ、すべてのβ<αについてβ-worldly基数がunbounded in κであればα-worldlyである。
基数κがhyper-worldlyであるとはそれ自身がκ-worldlyであることをいう。そしてα-hyper-worldlyやα-hyper-β-worldlyの概念もhyper-到達不能基数との類推によって定義できる。
すべての到達不能基数κはκ-worldlyであり、なおかつsuch kinds of cardinalsの極限である。
611 :
2016/08/15(月) 23:20:22.38 ID:ikxD5NBF
選択公理は含むようだ
612 :
2016/08/16(火) 01:52:42.86 ID:i4+GeMjF
>>607
どうやら勘違いだったようなので修正しておきました(ツイッター参照)
613 :
2016/08/17(水) 01:15:44.16 ID:Q3X4vrK5
俺の描写不可能基数を見てくれ、こいつをどう思う?
614 :
2016/08/17(水) 02:09:44.79 ID:eDMqXMuF
モザイクがかかっててよく見えません。
615 :
2016/08/17(水) 12:55:54.14 ID:1L7l83W0
朝起きたらおれの弱コンパクト基数が超コンパクト基数になってた
616 :
2016/08/17(水) 21:03:06.24 ID:MGSmEOlq
ω+1=suc(ω)={ω,{ω}}
ω+2=suc(ω+1)={ω,{ω},{ω,{ω}}}
なんでしょうか?

そうするとω+ωはωを{0,{0},{0,{0}},...}で表した時の
0をωに置換したものになりますか?
617 :
2016/08/17(水) 23:06:53.24 ID:Q3X4vrK5
>>616
suc(ω)=ω∪{ω}={0,1,2,...,ω}でしょ
618 :
2016/08/17(水) 23:12:11.92 ID:Q3X4vrK5
indescribableの訳って「描写不可能」以外にも「名状しがたい」ってのもあるんだな
619 :
2016/08/17(水) 23:20:37.62 ID:MGSmEOlq
>>617
うーんそうですか。
じゃあ

ω+2=
suc(ω+1)=
(ω+1)∪{ω+1}=
{0,1,2,...,ω,ω+1}=
{0,1,2,...,ω,{0,1,2,...,ω}}=
{0,1,2,...,{0,1,2,...},{0,1,2,...,{0,1,2,...}}}

てことですかね?
620 :
2016/08/17(水) 23:35:39.06 ID:MGSmEOlq
{0,1,2,...,ω}

この書き方ってなんかごまかしてない?
一見、綺麗にかけてるようでいて、よくみるとごまかされてるような。
621 :
2016/08/17(水) 23:43:38.17 ID:MGSmEOlq
1={0}
2=1∪{1}={0,{0}}
3=2∪{2}={0,{0},{0,{0}}}
...
ω=0∪1∪2∪...
={0,{0},{0,{0}},{0,{0},{0,{0}},{0,{0},{0,{0}},{0,{0},{0,{0}}...},

だとするならωとω+1の違いがよく分からん。
なんかごまかされてる。

ω∈ω+1 だけど NOT(ω∈ω)てことか?
わからん過ぎる。
622 :
2016/08/17(水) 23:46:45.36 ID:MGSmEOlq
ω∈ω 

たぶんここがわからん胆やな
623 :
2016/08/18(木) 00:07:14.73 ID:Srk2+Gyt
>>619
そう。

>>620
ωは{x|xは自然数}と同じ。
ω+1は{x|xは自然数またはω}と同じ

>>621-622
suc(α)=α∪{α}={α,β|β∈α}
順序数は推移的集合だから次の順序数は前までの順序数すべてを含む。
624 :
2016/08/18(木) 00:07:51.73 ID:+vfijLn+
0∈1∈2∈3∈...
ω=自然数全体
なら
ω∈ωにならなきゃおかしくない?

でもそうすると
ω+1=ωになっちゃわない?
625 :
2016/08/18(木) 00:10:18.92 ID:+vfijLn+
うおリロードしてなかったすいません。
626 :
2016/08/18(木) 00:10:48.52 ID:Ue6QqkEn
>>624
A∈B と A⊂B を混同してるんじゃないの
627 :
2016/08/18(木) 00:21:07.68 ID:+vfijLn+
>>626
推移的集合だと∈と⊂の差ってなくなっちゃわない?
そうでもないのかな…
628 :
2016/08/18(木) 00:31:55.94 ID:Srk2+Gyt
いや、ωは"自然数全体"ではなくて自然数全体の集合
だからω∈ωではない
629 :
2016/08/18(木) 00:33:12.67 ID:Srk2+Gyt
推移的集合ならa∈b⇔(a⊂b∧a≠b)

a=bならa∈bにならない
630 :
2016/08/18(木) 21:48:29.59 ID:+vfijLn+
ωの正確な定義ってなんでしたっけ?
wiki見てみたけど書いてない。

>>623の説明されるとなんとなくわかったような気になるんですが、
ちゃんとした定義が知りたいです。
631 :
2016/08/19(金) 00:54:22.13 ID:ZAqFYDlg
ωは{x|xは自然数}つまり「自然数全体の集合」がちゃんとした定義
632 :
2016/08/19(金) 01:00:18.24 ID:sbcazfsc
α=1+αとなる最初の順序数
1+α≠α+1である点に注意

1+αはαの元をすべて+1して({0,1}→{1,2})、最初に0を加える操作

ωはすべての自然数の集合だし、1+ωの元もやはりすべての自然数の集合になるからω=1+ω
633 :
2016/08/19(金) 01:04:23.10 ID:sbcazfsc
ちょい修正
ωはすべての自然数の集合だし、1+ωの元もやはりすべての自然数の集合になるからω=1+ω


1+ω={0,x+1|x<ω}={x|x<ω}
ω+1={x,ω|x<ω}
634 :
2016/08/19(金) 01:04:23.34 ID:sbcazfsc
ちょい修正
ωはすべての自然数の集合だし、1+ωの元もやはりすべての自然数の集合になるからω=1+ω


1+ω={0,x+1|x<ω}={x|x<ω}
ω+1={x,ω|x<ω}
635 :
2016/08/19(金) 01:14:09.53 ID:sbcazfsc
連投しちゃった

http://ufcpp.net/study/math/set/cardinality/#sum

α+β=ord(α⊗{0}∪β⊗{1}) (⊗は直積)
(x,m)≦(y,n)⇔[m<nまたは(m=n=0かつx≦y)または(m=n=1かつx≦y)] と順序関係を定義する

ω={0,1,...}, 1={0}
ω+1=ord(ω⊗{0}∪1⊗{0})
ω⊗{0}={(0,0),(1,0),...}
1⊗{0}={(0,0)}
ord(ω⊗{0}∪1⊗{0})=ord({(0,0),(1,0),...})=ω
636 :
2016/08/19(金) 01:14:30.85 ID:sbcazfsc
あれ?なんか間違えたか
637 :
2016/08/19(金) 01:16:23.78 ID:sbcazfsc
ω={0,1,...}, 1={0}
ω+1=ord(ω⊗{0}∪1⊗{0})
ω⊗{0}={(0,0),(1,0),...}
1⊗{1}={(0,1)}
ord(ω⊗{0}∪1⊗{0})=ord({(0,0),(1,0),...,(1,0)})=ω+1
こうだった
638 :
2016/08/19(金) 01:17:17.39 ID:sbcazfsc
まだ間違いがあった 連投スマソ
正しくはこう ω+1=ord(ω⊗{0}∪1⊗{1})
639 :
2016/08/19(金) 07:11:47.29 ID:sbcazfsc
本当に申し訳ないが正しくはこう
ω+1=ord(ω⊗{0}∪1⊗{1})=ord({(0,0),(1,0),...,(0,1)})=ω+1
1+ω=ord(1⊗{0}∪ω⊗{1})=ord({(0,0),(0,1),(1,1),...})=ω

{0}⊗{0}∪ω⊗{1}={(0,0)}∪{(0,1),(1,1),(2,1),...}
640 :
2016/08/19(金) 21:33:23.61 ID:Gq80UcMj
ωの存在って無限公理があって初めて保証される微妙なものなんですか?
ひっかかっていたのは其処かも…
641 :
2016/08/19(金) 23:48:53.60 ID:ZAqFYDlg
微妙もなにも、ZFの中に無限公理があるんだから当然仮定するでしょ
642 :
2016/08/20(土) 08:51:29.10 ID:ihyMou0J
有限の範囲しかみてないからωとω+1の違いが見えなくなってたんじゃ? この違いは理解できたでおk?

0の存在も空集合の公理でそれぞれのモデルに存在することが保証されるということで、
じゃあ微妙でない存在って何? って話にもなる。
643 :
2016/08/20(土) 18:35:47.79 ID:gpKK+otz
>>642
なんかω∈ωが成り立たないと0.999999999...≠1っていわれているような納得のいかなさがありますが
まあωの要素は有限集合じゃなきゃいけないということで一応納得しておきます。
644 :
2016/08/20(土) 18:43:35.45 ID:ihyMou0J
たぶん
1,2,3,...という有限の自然数の数列が無限に発散することと、
1,ω,ω^2,...と数列が無限を含むことは別だということを忘れているのが原因では
645 :
2016/08/20(土) 21:11:04.70 ID:kftlEf7C
>>643 「ωの要素は有限集合」
有限集合である自然数が無限にあるのだから、自然数全部の集合は無限集合になる、
だからωは無限集合になる、という説明なら分かる?
646 :
2016/08/20(土) 23:43:42.31 ID:gpKK+otz
>>645
問題なのは有限とは何か、無限とは何かという根本的なところだと思います。
「有限」「無限」に対するあいまいさのない厳密な循環論法にならない定義、書けます?

有限⇒無限じゃない
無限⇒有限じゃない

みたいな循環論法にならないやつ。
647 :
2016/08/21(日) 00:18:27.32 ID:YtMloLAA
無限公理で無限が定義されるでいいだらう。

1,2,3,...
この数列は任意の有限の自然数nに対してこれより大きい有限の自然数
をもっている。しかしすべて有限である。ここまでは異論ない?

1は有限、2は有限、3は有限、・・・
こうしてすべての自然数を表す記号を並べて有限であると定義していけばいい。
648 :
2016/08/21(日) 23:22:28.86 ID:lGN2qd/m
推移的集合にフラクタル的な部分と全体が等しいみたいなイメージを持ったのが迷走の始まりだったのかも?
とりあえずωにωが含まれないのは納得できました。
649 :
2016/08/24(水) 21:39:33.26 ID:UOnxVjH0
空集合の公理、巾の公理、外延公理で0の存在と後者関数の全域性、
すなわち自然数を定義することができる。
ここまででどれほどの関数を定義できるか?
650 :
2016/08/24(水) 21:41:43.13 ID:UOnxVjH0
定義というか全域性の証明か
651 :
2016/08/28(日) 16:58:37.76 ID:Vkp6O5fy
とりあえず後者関数をどんどん合成していっただけの関数は証明可能
652 :
2016/08/30(火) 22:55:56.73 ID:jRa3zmTv
ω×2やω^2の弱い関数ってどんなのがある?
653 :
2016/08/31(水) 06:26:17.05 ID:vFViqi4U
FGHでの強さって意味ならwiki見るといいよ
http://ja.googology.wikia.com/wiki/%E9%96%A2%E6%95%B0%E3%81%AE%E4%B8%80%E8%A6%A7
ω×2ならf(n)=3→3→n→n (チェーン表記)とか
ω^2ならf(n)=A(n,0,n) (多変数アッカーマン関数)とか
654 :
2016/08/31(水) 21:01:23.99 ID:cE/+b5oT
証明論的順序数だとω^2の強さで掛け算を定義できるくらい。
証明論的順序数がなんなのかは知らん
655 :
2016/09/01(木) 19:41:45.30 ID:xn25HAby
超次元BEAFってε0ないのか
多重リストならなんでもε0かと思ってたわ
なにがヒドラとか多重リストアッカーマンとそんなに違うんだろう。
656 :
2016/09/01(木) 21:03:42.91 ID:xM3NSrlg
超次元配列は二重リストでしょう。多重リストはテトレーション配列
657 :
2016/09/01(木) 21:14:22.65 ID:xn25HAby
えっそうなん?
ちゃんと定義読むか…
658 :
2016/09/01(木) 21:28:35.07 ID:xn25HAby
wikiの説明じゃよくわからんwwww
超次元多次元は配列の仕切りを(n)で表すことで実質2重になるのか?
659 :
2016/09/01(木) 21:30:21.09 ID:xn25HAby
超次元多次元ってなんだおれw
超次元配列ね
660 :
2016/09/01(木) 23:52:36.69 ID:xn25HAby
よくわからんが、超次元BEAFは配列の仕切りの種類が1種類しかなく
配列の仕切りの種類が可変個になるとε0ということだろうか?
661 :
2016/09/02(金) 00:19:56.66 ID:gKmZEahP
配列の区切りをツリーで表すとε0?
だめだ完全に混乱してきた。
662 :
2016/09/02(金) 07:02:01.58 ID:8Jwx1Gkm
構造を順序数で表してみる(この順序数はBEAFの強さとは関係ない)
1変数→ω
2変数→ω^2
線形配列(ω変数)→ω^ω
2次元→ω^ω^2
多次元→ω^ω^ω
663 :
2016/09/02(金) 19:52:04.41 ID:gKmZEahP
>>662
その構造的にいくとε0とはどのようなものになるのでしょうか?
664 :
2016/09/02(金) 19:59:41.36 ID:8Jwx1Gkm
>>663
ω^ω次元がω^ω^ω^ωとなって、ω^ω^ω次元がω^ω^ω^ω^ω、・・・
ε0次元がε0になるかな
665 :
2016/09/02(金) 20:03:29.78 ID:gKmZEahP
>>664
もぅマヂ無理。
666 :
2016/09/02(金) 20:05:40.43 ID:8Jwx1Gkm
>>663
1変数のときに入れ子にするとω^2になったりするので
もしかすると線形配列を入れ子にするとε0になるかもしれない
667 :
2016/09/02(金) 20:08:33.60 ID:gKmZEahP
???
1変数を入れ子にするって???
線形配列を入れ子にしたら多次元では???
668 :
2016/09/02(金) 20:13:25.44 ID:8Jwx1Gkm
>>667
入れ子というか多重リストだった
(n)に対して((n))や(((n)))を許したり
(m,n)に対して((m,n),o)や(((m,n),o),p)や(m,(n,o))を許したりする
669 :
2016/09/09(金) 21:42:55.78 ID:LNl6/LV/
loader.cのwhileの中にある(1)とか||の分岐を囲ってる()とか、
なんの意味があるのかわからない。無くてもいいんじゃないの
670 :
2016/09/09(金) 22:19:40.27 ID:ldcXxrQT
そういう無駄に見える部分は、マクロを使って簡略化したことの代償として存在してる。
671 :
2016/09/09(金) 22:32:38.10 ID:LNl6/LV/
そんなに気にしなくていいってことか? ありがとう
ちまちま解析していく。
D(0)=D(1)=0
D(2)=7*2^30
D(3)=9*2^(7*2^(2^7))
D(4)=D(5)=7*2^(7*2^(15*2^2))
D(6)=3*2^(9+7*2^7+9*2^(7*2^2^7))
672 :
2016/09/12(月) 22:47:35.99 ID:kjelKf7V
ヒドラゲームって多重ループとそんなに変わらないのかと思ってたけど、
根元からごっそり増殖するんだな。
予想よりはるかに凶悪。
673 :
2016/09/15(木) 22:36:57.87 ID:PY1CuUUQ
ふぃっしゅ数バージョン 1, 2, 3 をRubyで書いてみたよ
バージョン1
http://ideone.com/YfaGgh
バージョン2
http://ideone.com/eTyuGf
バージョン3
http://ideone.com/xO3oHd
674 :
2016/09/15(木) 23:22:15.34 ID:1hPVzgzD
ラムダ式読めね〜w
でも多分元の定義に忠実にコーディングされてるんだろうね。
675 :
2016/09/18(日) 12:28:46.56 ID:lhoRvQE/
ヒドラ系の回数を数える関数をフィッシュ数みたいに1回ごとに強化していったらどのくらい強さ変わるかな?
676 :
2016/09/18(日) 21:52:39.11 ID:I/NcLp1g
ゲーデル数化と対角化の応用

X=自称「X」

X=「X」という考えを押し付けるな。

X=「X」という名のY

ゲーデル数化のように、「」で囲われることでその言葉の意味は問われず、形式的な扱いのみが問題となる。
普段意識しないだけで巨大数をつくるための基本テクニックとなっている。
677 :
2016/09/18(日) 22:59:57.52 ID:JJI76yJc
対角化って対角線論法だから対角化なんだっけ?
巨大数界隈だと具体的にどういうテクニック?
685 :
2016/09/21(水) 20:19:21.13 ID:dSZ6IMI+
>>677
なんでもいいので関数を帰納的にf_0,f_1,f_2,...と定義してf_x(x)という関数を定義するとか、
その関数も具体的に自然数から自然数への写像に限らずとも、
自然数から関数への写像とか、型から型への写像とかでもよい。

もう少しつっこむと、
f[α](x)
αはxの型を定義域に含む関数の型の値、もしくはf[]は順序数と順序同型な型から
関数の型への写像と捉えることができる、と解釈してよかろうか?

型理論は言葉の使い方や解釈が専門家によって区々でヤヤコシイ
700 :
2016/09/23(金) 01:21:12.92 ID:gZ0Vsytt
後者関数から見た乗法は対角化になる?
701 :
2016/09/23(金) 10:40:34.76 ID:cnj0dUoV
後者を対角化した加算を対角化した乗算を対角化した冪乗を対角化したテトレーション、とするのがハイパー演算子
702 :
2016/09/23(金) 13:42:57.79 ID:uZ4Qtt+h
f_0,f_1,f_2,...の_nがゲーデル数にあたって、
xで呼び出される関数でxを評価するのが対角化
703 :
2016/09/23(金) 20:00:02.20 ID:aKCPV7dC
f_0,f_1,f_2,...を作るやり方によって何を対角化とするのかは変わるでしょ
後者関数をn回繰り返すのがf_n(x)だとすればそれを対角化した関数はf(n)=f_n(n)=2nになる
ハイパー演算子の繰り返しとして対角化するのなら加算になる
704 :
2016/09/23(金) 20:19:03.05 ID:6JzuohMM
任意の関数に対して対角化した関数が存在する?
705 :
2016/09/23(金) 21:42:39.90 ID:uZ4Qtt+h
別の対角化を表すのであればfの部分をgなりhなりに変えて
区別がつくようにすればよいです。
あるいは3変数にしてf(n,α,x)と表してしまうとか。
SKIコンビネータのSがたぶんこれになる。
706 :
2016/09/23(金) 21:48:39.24 ID:uZ4Qtt+h
と思ったけどよく考えたらあまり関係無かった。
Sxyz→xz(yz)
x:関数であることを表す。具体的な意味はもたない。
y:変数であることを表す。具体的な意味はもたない。
z:関数と変数に具体的な意味を与える。
707 :
2016/09/24(土) 00:21:02.38 ID:Ebh8bblA
f(x) に対して f^x(x) を与えることを対角化とすれば、FGHでは+1に相当する。
ただ、その対角化を繰り返してもωには到達しない。
極限順序数に対して基本列を対角化するFGHの定義があって、
順序数の概念でどんどん強めることができる。
708 :
2016/09/24(土) 11:17:07.14 ID:d73uFysG
バシク行列は対角化がわかりやすいかも
, (0), (0)(0), (0)(0)(0), ... → (0)(1)
(0), (0)(1), (0)(1)(1), ... → (0)(1)(2)
(0), (0)(1), (0)(1)(2), ... → (0,0)(1,1)
(0,0)(1,1), (0,0)(1,1)(1,1), ... → (0,0)(1,1)(2,0)
709 :
2016/09/25(日) 00:37:40.97 ID:qM+LggGx
loader.cのwhileの解釈があれであっていたらD(99)は4の倍数の巨大数
になってD(D(99))以降は0になるはずだのに意味不明
710 :
2016/09/25(日) 14:39:10.53 ID:aVxuwJrq
http://pastebin.com/SQKqN1FY
あってるかわかんないけどpython (Loader.py)
711 :
2016/09/25(日) 15:59:48.43 ID:qM+LggGx
その解釈でローダー数が0になってしまうんだ。
aycabta氏がifに書き換えたやつでは反対にx/=2した後のx%2がbreakの条件になっているが、
そうすると今度は計算が終了しなくなってしまう。
712 :
2016/10/01(土) 12:39:00.75 ID:9FblePXi
言語L_0で表されるプログラムをゲーデル数化して扱う言語L_1、
L_1で・・・L_2、L_2でL_3、・・・
713 :
2016/10/02(日) 18:54:36.92 ID:+rcjIZ76
CoCとかいうのは、あまり自信がないけど、既に定義された関数fを順序数を使って強化するFCHみたいな技術であって、
ZFC+巨大基数公理の理論と一緒に巨大数の大きさを計る物差しとして使うのはちょっと違う気がする。
714 :
2016/10/03(月) 00:02:12.22 ID:9ljqe4Kn
とうとう巨大数の本が出ましたね

  鈴木真治『巨大数』、岩波科学ライブラリー 253 (2016年9月刊)

ただ一般向けだからこのスレ的にはほとんど常識的な内容かもしれないですが
715 :
2016/10/03(月) 00:10:27.17 ID:xCu5OH47
まじでw
即買うぜw
順序数のことも書いてあるといいな。
716 :
2016/10/03(月) 22:13:56.38 ID:xCu5OH47
買ってきたよ〜
100ページちょいの軽い読み物って感じ?
ラストのほうには順序数とかハーディ階層とかちょっと出てくるみたい。

まあこれから読んでみる。
717 :
2016/10/04(火) 23:20:24.64 ID:3r1KZwKH
順序数に関してはε0までみたい。
718 :
2016/10/06(木) 21:53:40.21 ID:FIx7qYPB
対角化の説明がいまいち納得いかない
f(x)に対してf(2x-1)を返すのが対角化なのか?
俺が勘違いしてるだけなのか…
719 :
2016/10/07(金) 03:51:18.64 ID:RLAJ4Bjw
>>718
f(x,y)に対してg(x)=f(x,x)じゃないかな
720 :
2016/10/07(金) 20:55:00.76 ID:nhc+qd20
>>719
なるほど、そっちのほうがしっくりきますね!
721 :
2016/10/08(土) 18:16:53.65 ID:PFrLVIOz
言語が離散的なものという前提を捨て去ると何が見えてくるだろうか
連続的言語、n文字という区切りのない言語で巨大数を追究すると
単に無限が出てくるつまらない結果に終わるのか、それとも
722 :
2016/10/08(土) 22:38:13.52 ID:PqiEfR6G
文字がn種類あるとき
1文字が持つ情報量はlog(n)
2文字が持つ情報量はlog(n^2)

4文字ありlogの底が2の場合1文字が2bit、2文字が4bit
2.5文字なら3bitということになるがどうすればいいのか

4つ文字を使ってちょうど8種類の表現ができる場合とは
723 :
2016/10/08(土) 22:39:01.97 ID:PqiEfR6G
有理数、無理数に拡張するのは可能なのか
2.5文字を表現できたなら有理数は可能だと思うが
724 :
2016/10/08(土) 22:41:23.60 ID:PqiEfR6G
文字がω個ある言語ならできそうだけど
725 :
2016/10/08(土) 22:42:14.39 ID:JdgYhFGm
すべての曲線の集合がアレフ3だっけ
そんな感じになるのかな
726 :
2016/10/08(土) 22:44:02.80 ID:PqiEfR6G
4進言語の2.5文字は2進言語の3文字、または、8進言語の1文字に相当するから文字種の量を変えることで
p文字を表現できることに気づいた
727 :
2016/10/08(土) 22:46:08.11 ID:gVrXZ7mI
可算無限ビットあれば自然数から自然数への任意の関数が定義可能
728 :
2016/10/08(土) 22:46:09.11 ID:PqiEfR6G
アレフ1以上は多分無理じゃないかと思う
文字がω個ある言語の方だと今のところ多分原始数列が最強で上限がε_0
729 :
2016/10/08(土) 22:49:30.45 ID:PqiEfR6G
文字数を実数に拡張する試みってなんか急増加関数をω_1以上に拡張する試みと似てる気がする
730 :
2016/10/08(土) 22:52:10.54 ID:JdgYhFGm
ω進数なら
1、ω、ω^2、ω^3・・・・の集合だから
ω↑↑2程度じゃないの?
731 :
2016/10/08(土) 22:55:19.53 ID:PqiEfR6G
ω進数つまり自然数のリストから何か巨大数とか公理系とか順序数とかを生み出す関数のことを言いたかったんです
で、自然数のリストから順序数や巨大数をつくるものとしては原始数列が多分最強なんじゃないかと
732 :
2016/10/09(日) 15:07:25.61 ID:KctRUAZ1
先ほどから具体性に欠く。
n文字で定義される巨大数や関数について考えるとしてその具体的なルールは?
原始数列が最強といっても、行列も自然数のリストで表すことが可能なんだし、
任意の記述可能な定義は自然数のリストで表せる。
さらに有限の文字数で考えるのであれば、ω進数の1ケタの表現力とnケタの
表現力は変わらない。
無限の文字数で考えるのであればω進数も有限進数も表現力は変わらない。
733 :
132人目の素数さん
2016/10/22(土) 15:07:04.16 ID:bcgsyc0x
ちんげとか金玉みたいな記号の意味ってどこに載ってるの?
744 :
2016/10/22(土) 17:04:24.93 ID:8m/JlmHf
あ、sageてね

金玉の方は http://ja.googology.wikia.com/wiki/%CE%A9
745 :
2016/10/22(土) 18:01:45.83 ID:HTnSFuVC
ちん毛みたいな奴が強いのよ!
757 :
2016/11/01(火) 21:55:12.76 ID:9S+BNdAh
冪集合演算を巨大数関数生成に生かすような方法はないの?
冪集合ってすごい使えそうなイメージあるんだけど実際はあんまり巨大数にでてこないよね?
758 :
2016/11/02(水) 00:58:45.01 ID:Du73ds0A
直接使って何かするのは見たことないので、なにかできそうなら考えてみては
759 :
2016/11/03(木) 22:13:57.64 ID:LH4r/ohe
RubyのFiberが面白そうなんだがFiberのネストは無理なのかな?
ネスト出来たらなんか強くなりそうな予感。
760 :
2016/11/03(木) 22:39:46.90 ID:LH4r/ohe
まだぼんやりとしたイメージしかないけど。

具体的には以下のfをFiber化したい

fはいまは整数を引数に取って繰り返しする関数だけど
xに代入するのは-=1と==0とbigとdupが定義されるクラスならなんでもいい
例えばxを配列に拡張することもできるだろう

そこでfそのものを-=1と==0とbigとdupが定義されるクラスに落とし込むことを考える。
そしてその後f(f(整数))などとネストすることで強い関数が生まれるようにしたい。

$a=3

class Integer
def big()
return $a
end
end

def f(x)
x-=1
$a+=1
return if(x==0)

y=x.big()
until(y==0)
y-=1
f(x.dup)
end
end

f($a)

p $a
761 :
2016/11/03(木) 23:15:08.48 ID:LH4r/ohe
ちなみにfに整数を入れたときはfはツリー構造になる。
このツリー構造をさらにfに再び突っ込んでツリーのネストのようなものを実現したいというか。
762 :
2016/11/07(月) 22:05:01.93 ID:5M5B6Lbm
Fiberだめっぽいorz
763 :
2016/11/08(火) 01:28:49.98 ID:gT4gswKO
>>757
> 冪集合演算を巨大数関数生成に生かすような方法はないの?

冪集合はcardinalityに関しては2の冪をとる演算になるから巨大数の構成法としてはかなり低い(弱い)手段じゃない?
764 :
2016/11/08(火) 01:39:40.35 ID:Q7ZcqKqy
誰かハイパー演算子 hyper(a,n,b)=a↑^{n-2}b の n を自然に実数に拡張してくれ。
きっと新しい知見が開ける。
765 :
2016/11/08(火) 21:16:08.04 ID:BlQ8khyS
>>764
出来るんだったらすでに誰かがやってるだろうから出来ないんじゃない?
出来ないという証明があるのかどうかは知らないけど。
766 :
2016/11/08(火) 23:23:27.43 ID:bfH7ZMpx
>>765あらゆる学問全否定だな
767 :
2016/11/08(火) 23:59:01.65 ID:btCRwz4K
人に任せる前に自分である程度実績出してくれって話ではある。
言っちゃ悪いけど誰でも思いつくようなことだし。
ふぃっしゅ氏がなんかやってなかったっけ?
768 :
2016/11/09(水) 00:23:27.54 ID:r4BT5fVF
要するに適当でもいいから自然数の場合の定義と矛盾しなければいいわけだ。出来ないということはない。
でも掛け算や指数、階乗ほど一意に定まるものでははないだろう。
769 :
2016/11/09(水) 01:04:12.47 ID:Fw4x4jnA
a=2 b=2が原点になる何か?
グラフ描こうにも縦軸に巨大関数が必要だなぁ
770 :
2016/11/09(水) 19:25:32.59 ID:5VvIfjS1
ハイパー演算子 hyper(a,n,b) って、先の演算 hyper(a,n-1) の適用を b 回繰り返すって意味だから、
n を実数とするなら、自然な後者関数と足し算の中間、足し算とかけ算の中間、かけ算と指数の中間、といったものを考えなければならない。
771 :
2016/11/09(水) 20:56:43.60 ID:Fw4x4jnA
整数へ拡張だけでも0が後者関数なのはあまりよくないな
1を後者関数 -1を前者関数にする必要があるのか
772 :
2016/11/10(木) 19:50:31.42 ID:IHDvuGTT
微分積分なら、実数階微分・積分簡単に出来るんだけどね。
773 :
2016/11/12(土) 23:01:45.72 ID:Zn1PFb8k
「出来ないということはない」なんていったところで、実際にやってみせなければ説得力ゼロ
774 :
2016/11/12(土) 23:15:22.67 ID:rXgW/cuL
そのくらい自分で調べなよ
775 :
2016/11/13(日) 00:14:26.99 ID:sp61i2k5
たとえばテトレーションの実数への拡張では1≦n<2のとき3↑↑n=3としてしまえば
後は任意の実数についての値は自然数のときのルールを適用することで自動的に決まる。
この定義じゃかっくんかっくんするけど
776 :
2016/11/13(日) 00:17:51.28 ID:sp61i2k5
なめらかな定義はいろんな人が試しているようだが自然な流れで一意に定義されるものではないもよう
777 :
2016/11/13(日) 12:36:25.56 ID:0UMQ5wO/
「滑らかな定義」というのは
「任意の定数a,bについてnの関数hyper(a,n,b)が無限回微分可能」
でいいですよね?
778 :
2016/11/13(日) 17:34:13.92 ID:NTtWZM7T
3↑↑n=3 のnを実数にするのはすでにやられているし、誰もそんな話ししてないだろ
よく読めよ
779 :
2016/11/13(日) 17:36:48.71 ID:NTtWZM7T
いきなり無限回微分可能はハードルが高いので、とりあえずはhyper(a,n,b)が連続であるだけでも十分では
780 :
2016/11/13(日) 17:42:25.16 ID:NTtWZM7T
矢印表記の実数への拡張は「巨大数論」にすでに書かれている
hyper(a,n,b)のbを実数にできるのはいいとして(無限回微分可能はちょっと面倒だけど)
今問題にしているのはnを実数に拡張できるかという話だよね
それについては、誰も具体的な提案をしていないので「やればできる」なんていう話には
説得力がまったくない
781 :
2016/11/13(日) 18:25:40.09 ID:sp61i2k5
いろいろ雑すぎたので反省。
任意の自然数bとaについてf(b*a)をf(a)とf(b)で表すみたいな一般的なある論理式が、
ある実数への拡張においてもどこまで任意の実数の間でも成り立つようにするか? とか、
を考える必要があるし、場合によっては0で割ってはいけないというような例外を設ける必要がある(かもしれない)。

とりあえず具体的にどういう拡張を望むかという方針がなければ、なんでもアリになってしまう、
くらいの表現に抑えるべきでした。
782 :
2016/11/13(日) 18:54:51.25 ID:sp61i2k5
言い過ぎたのは悪かったです。テトレーションのたとえは、多くを望まなければ、ペンテーションでも上矢印表記でも
m≦n<m+1 の値は m と同じように扱って自然数のときの値をそのまま実数のときの値と
してしまう強引なやり方があるくらいのことが言いたかっただけです。

3^^-3のような値を対数をとる回数から定義しようとすると定義不可能であることが示されてしまう、
ということもあり、実数への拡張が必ずしもすべての実数について値が存在することが前提とならないこともあります。
783 :
2016/11/14(月) 00:31:50.34 ID:qevCRA2U
具体的に何を望むかとなると、とりあえずは連続性があればいいかな
たとえば指数とテトレーションの中間の演算を考えるというだけでもけっこう大変
784 :
2016/11/14(月) 00:35:08.88 ID:qevCRA2U
hyper(a,n,b) が満たすべき性質は何だろうか
785 :
132人目の素数さん
2016/11/14(月) 01:05:08.89 ID:qevCRA2U
連続性を満たすだけならばnについて線形補間でもスプライン補間でもなんでもすればいいわけで、
問題はnが実数の時にhyper(a,n,b) が満たすべき性質って何?というところかな
797 :
2016/11/15(火) 01:07:39.40 ID:XEkX0J70
むかし海外勢がBEAFを実数域へ拡張しようとしていた気がするが、気のせいか?
マイノリティーだったり専門性が高かったりすると、ネットで検索してもヒットしないということがある。
証明論なんかは英語で検索するとヒットする分救いがあるわ。
798 :
2016/11/15(火) 01:30:27.75 ID:s5BU+7mi
対角化を一つの函数であらわすってコト?
799 :
2016/11/15(火) 02:52:24.37 ID:+cattmlh
たとえばハイパー3.5演算子って何?ってこと。
ハイパー3演算子とハイパー4演算子の算術平均だよ(線形補間)とすれば、定義はできる。
他にもいくらでも定義のしようはある。
だから定義ができるかどうかが問題ではなくてハイパー3.5演算子が満たすべき本質的な
性質は何か、ということが問題。線形補間で「定義できた」といっても、面白くもなんとも
ないでしょ。
800 :
2016/11/15(火) 18:15:47.71 ID:SIRoLOhh
>>799
実数への拡張じゃつまらないね
君が指摘してるとおり恣意的な拡張はいくらでも可能だから
複素数上に解析関数として拡張できるならば大いに数学的な意味があると思うが
801 :
2016/11/15(火) 18:24:15.24 ID:LXeg2zc0
巨大数の観点からすると、解析関数の増大度には極めて強い制限があるので無意味、というか不可能
802 :
2016/11/15(火) 19:37:37.60 ID:H7W0lMDi
>>801
やっぱりか。
803 :
2016/11/15(火) 20:45:10.65 ID:XEkX0J70
疑似テイラー展開みたいなことをすれば自然数を定義域に持つ任意の(?)関数を実数域へ拡張することができるし、
指数の計算なんかとも一致する

って言おうとしたらそれがスプライン補間なのか。ルンゲ現象とかいううまくいかないパターンもあるとか
804 :
2016/11/16(水) 02:36:09.86 ID:PSs7mXQ0
hyper(a,3.5,b)をhyper(a,3,b)とhyper(a,4,b)の平均であると定義したとして、
hyper(a,3,b) << hyper(a,4,b) だから、結局それは hyper(a,4,b) / 2 ということになり、
テトレーションレベルだとほとんど hyper(a,4,b) と近似できてしまうな。
スプラインだろうがなんだろうが、結局 hyper(a,4,b) の強さに引っ張られるだろう。
805 :
2016/11/16(水) 23:29:06.96 ID:Yr37kDmW
もしかしてhyperを実数に拡張するのは連続体仮説が絡んできたりしてw
806 :
2016/11/17(木) 20:58:57.71 ID:E0EGZIsx
例えば指数関数を多項式にテイラー展開すれば∞次元となるのだろ?
指数と乗算の中間があるか?というのは有限と可算無限の中間があるか?みたいな話じゃないのか?
807 :
2016/11/17(木) 21:18:57.29 ID:FtNYrMVb
そういう話に持って行きたければそういうことにすればいいって話だよ。
上で言ってるどういう方針を持つかという話。

有限と可算無限の間については超自然数でもぐぐれば?
808 :
2016/11/17(木) 21:31:29.31 ID:jMvVkRAe
>>806
意味も分からず数学用語を弄るのはやめーや
809 :
2016/11/18(金) 22:42:34.94 ID:lCpZgFkF
1重ループを加えるのが+1だとするなら^ωに相当するのはどんな操作?
810 :
2016/11/21(月) 01:15:14.70 ID:8FRsTMbB
一言で言うのは無理だろ
あえていうなら、ノードの上にヒドラを乗っける操作
811 :
2016/11/21(月) 21:00:48.45 ID:wlJrU+Yq
ノードの上にヒドラ乗っけたら+ε0になりそうな…
812 :
2016/11/24(木) 22:20:20.77 ID:YJBpav56
アッカーマン関数がω
多変数アッカーマンがω^ω
2重リストアッカーマンがω^ω^ω
超次元BEAFがω^ω^ω^ω

これから推測するに^ωに対応する操作は…
ん〜わからんw
813 :
2016/11/25(金) 01:45:16.48 ID:AVeUH+c+
ε_0の力を持ったシステムじゃないと表現不可能じゃないかな
ドル関数なら[]に入れる操作
814 :
2016/11/30(水) 13:39:32.88 ID:OkNBdN0X
順序数を[]で表現してみた

0
1
2
n
0[]0 = ω
0[]1 = ω+1
0[]2 = ω+2
1[]0 = ω*2
1[]1 = ω*2+1
2[]0 = ω*3
n[]m = ω*(n+1)+m
815 :
2016/11/30(水) 13:41:49.78 ID:OkNBdN0X
0[]0[]0 = ω^2
k[]n[]m = ω^2*(k+1)+ω*n+m
0[]0[]0[]0 = ω^3
0[]0[]0...{0[]がn個}....0[]0 = ω^n
0[][]0 = ω↑↑2 = ω^ω
0[][]0[][]0 = ω^(ω*2)
0[][]0[][]0...{0[][]がn個}....0[][]0 = ω^(ω*n)
0[][][]0 = ω^ω^2
0[][][]0[][][]0 = ω^(ω^2*2)
0[][][]0[][][]0...{0[][][]がn個}....0[][][]0 = ω^(ω^2*n)
0[][][][]0 = ω^ω^3
0[][][]...{[]がn個}...[][]0 = ω^ω^(n-1)
816 :
2016/11/30(水) 13:43:31.08 ID:OkNBdN0X
0[[]]0 = ω↑↑3 = ω^ω^ω
0[[]]0[[]]0 = ω^(ω^ω*2)
0[[]][]0 = ω^ω^(ω+1)
0[[]][][]0 = ω^ω^(ω+2)
0[][[]]0 = ω^ω^(ω*2)
0[][][[]]0 = ω^ω^(ω*3)
0[[]][[]]0 = ω^ω^ω^2
0[[]][[]]...{[[]]がn個}...[[]][[]]0 = ω^ω^ω^n
0[[][]]0 = ω↑↑4 = ω^ω^ω^ω
0[[[]]]0 = ω↑↑5 = ω^ω^ω^ω^ω
0[[[][]]]0 = ω↑↑6 = ω^ω^ω^ω^ω^ω
0[[[[]]]]0 = ω↑↑7 = ω^ω^ω^ω^ω^ω^ω
0[[[[][]]]]0 = ω↑↑8 = ω^ω^ω^ω^ω^ω^ω^ω
0[[[...[[]]...{[]のn個(n≧1)の入れ子}...]]]0 = ω↑↑(2*n-1)
0[[[...[[][]]...{[]のn個(n≧0)の入れ子}...]]]0 = ω↑↑(2*(n+1))
0[0]0 = ω↑↑ω = ε_0
817 :
2016/11/30(水) 19:39:54.27 ID:cmknUhG3
最後の0[0]0 = ω↑↑ω = ε_0
ってなんか飛躍してない?
ルールが突然変わったかのように見えてしまう。
まあ、括弧じゃ表せないってことなんだろうけど。
818 :
2016/11/30(水) 22:46:35.35 ID:bWcxGjkt
f(x)=「√2の十進表記の小数点第1〜n位が十進表記にはじめて登場する2の階乗の数」っていうのは増加量どれくらいかな。
819 :
2016/11/30(水) 23:30:39.19 ID:QsGOxIHR
√2=1.41421356…
f(1)=4(2^2)
f(2)=419,4304(2^22)
f(3)=23,6118,3241,4348,2260,6848(2^71)
…ってことか?
820 :
2016/12/01(木) 09:56:19.39 ID:N1A2sZAH
>>817
そうカッコだけだとε_0より小さい全ての順序数しか表せないんで
次のステージとしてカッコ内に先に定義された順序数を入れ子にすれば
ε_0より大きな順序数も表せるかなと考えた。
ただしどこまで大きく表せるかは検証してない。
821 :
2016/12/01(木) 13:01:54.14 ID:+dRcYNmN
>818
意味が分からなかったけど、階乗は累乗の間違い?
822 :
2016/12/01(木) 13:15:02.12 ID:+dRcYNmN
一般化して、ふたつの数列の列S,Mがあり、Sのm番目の要素S_mがM_nの中にあるとき、
f(m)=M_n

簡単な例

S={0,00,000,...} M={0!,1!,2!,...}

f(0)=120,f(1)=10!,f(2)=15!
823 :
2016/12/02(金) 08:25:16.26 ID:KGxO91vj
>>819
そう
>>821
そう
>>822
なるほど
824 :
2016/12/02(金) 08:29:18.98 ID:KGxO91vj
数字列がランダムだとするとnが1増えても確率が1/10になるだけだから10^10^n程度か
825 :
2016/12/02(金) 19:59:12.22 ID:Mq4/+sOm
乱数と期待値で定義される関数とは違って厳密な証明がほしいところ。
826 :
2016/12/02(金) 20:10:51.57 ID:Mq4/+sOm
ってよく読んだらランダムって書いてたんだな。すまん
827 :
♯眠い
2016/12/03(土) 17:16:00.46 ID:tp4Rqyju
test
828 :
132人目の素数さん
2016/12/03(土) 17:17:03.61 ID:tp4Rqyju
ちゃんと、半角の#のはずだったんだが。
839 :
2016/12/03(土) 20:28:50.36 ID:favnptuA
λ式によるm(n)変換、ただしm(0)=n+1とする。

m(0)=λn.λf.λx.f(nfx)
m(1)=λm_0.λn.nm_0n
m(k+2)=λm_{k+1}.λm_k.nm_{k+1}m_k

F_5(n)=λn.m(n)m(n-1)...m(0)n
850 :
2016/12/03(土) 20:37:58.88 ID:z0G6MLi6
最近巨大数の本買って興味が出て、wikiとか読んでみたけどさっぱりわからんです
このスレの話しを理解しようとしたら、数学的な前提知識はどれくらい必要ですか?
861 :
2016/12/04(日) 01:35:39.13 ID:mH7eJXPX
このスレの話にもいろいろあるし、中学生も読んでいれば数学者も読んでいて、
一律の前提知識があるという仮定には無理がある

Wikiであれば、関連する記事を読みまくるとか、
Wikiだと体系的にまとめられてないので巨大数論PDFを最初から読んでいって
わからないところがあれば何がわからないのかを明確にして質問するとか、
自分からなにかわかろうとしなければ何もわからないと思う

そして、わからない人が書き込まないと誰も書き込まなくなってスレが沈静化するので、
なにか書き込むのはいいことだろうと思う
866 :
2016/12/04(日) 01:39:40.40 ID:mH7eJXPX
「ここがわからない」と書けばだれかが教えてくれるかもしれないけど、
ただ漠然とWikiを読んでわからないと書いても、どうしようもないわけで
877 :
2016/12/04(日) 11:32:05.07 ID:il7+4Zqx
>>861
>>866
ありがとう、なんか集合論?の話しとか、基数?とかはまずわからんです
取り敢えずpdfの奴読むところから始めてみる
888 :
2016/12/04(日) 16:54:46.08 ID:YQRsXSVK
>>877
おい


俺より先のページで悩むなよ
899 :
2016/12/04(日) 19:55:53.16 ID:il7+4Zqx
>>888
すまねぇ……
900 :
2016/12/04(日) 22:13:27.96 ID:jNsocdg8
0番目の基数、1番目の基数、2番目の基数、・・・と基数を作っていく。もちろん無限番目の基数というのも考えることもできる。
それまでに作った基数Aを使ってA番目の基数と言うのも作ることができる。

以上のやり方で作れる基数の限界がΩ収束点

また、これまで作った基数をすでに作ったある基数Aの大きさだけ並べてそのA番目となる
基数として新しい基数を作ることもできる。

以上のやり方で作れる基数の限界が到達不可能基数
901 :
2016/12/05(月) 02:25:58.55 ID:zHwktBu1
hyper(ω,ω,ω)ってε_1でいいの?
902 :
2016/12/05(月) 12:31:26.78 ID:w5LqWTNw
http://ja.googology.wikia.com/wiki/%E7%B7%A9%E5%A2%97%E5%8A%A0%E9%96%A2%E6%95%B0
この緩増加関数だとω^ω^ωがn^n^nに対応しているので、
これと対応させてhyper(ω,ω,ω)がψ(ω,0)と考えるのはどうかな
903 :
2016/12/06(火) 14:24:18.31 ID:otVqWId6
ψ(ω,0) ≒ ω→ω→ω
ψ(1,ω,0) ≒ ω→ω→ω→ω
ψ(2,ω,0) ≒ ω→ω→ω→ω→ω
ψ(3,ω,0) ≒ ω→ω→ω→ω→ω→ω
ψ(4,ω,0) ≒ ω→ω→ω→ω→ω→ω→ω
ψ(5,ω,0) ≒ ω→ω→ω→ω→ω→ω→ω→ω
ψ(6,ω,0) ≒ ω→ω→ω→ω→ω→ω→ω→ω→ω

ψ(ω,ω,0) ≒ ω→ω→...ω個...→ω→ω てなるの?
904 :
132人目の素数さん
2016/12/08(木) 18:54:58.11 ID:889tvehq
ε_0の次の順序数は、ε_0+1じゃないの?
905 :
2016/12/09(金) 00:02:04.39 ID:zrmJg8PJ
すべての自然数をちょうど一回使った列(並び順はいろいろ入れ替えられる)を考える。
このような列をあつめてさらにそれを辞書順にならべる。

これらの列と順序数の対応はどのようなものになるだろうか?
906 :
2016/12/09(金) 08:18:51.64 ID:J1PhUIjs
ω!
907 :
2016/12/09(金) 14:52:56.68 ID:jvJLsgyT
>>905
ω^ω
908 :
2016/12/09(金) 17:45:09.48 ID:J1PhUIjs
ω!=ωなのかな?
909 :
2016/12/09(金) 18:38:34.07 ID:jvJLsgyT
もしかして

n! 〜 √2πn(n/e)^n[1+1/2n+1/8n^2]^(1/6)

で ω! = ω^ω てこと?
910 :
2016/12/09(金) 19:07:10.83 ID:J1PhUIjs
ωの対称群だからωの階乗かと思った
ω以外の総乗がωだから2ωになるのか?
911 :
2016/12/09(金) 20:39:37.50 ID:Uiqbq0wO
まず

>すべての自然数をちょうど一回使った列(並び順はいろいろ入れ替えられる)

の濃度がΩ
912 :
2016/12/09(金) 22:13:47.66 ID:74W7lfF2
アレフ1じゃなくて?
913 :
2016/12/09(金) 22:38:15.93 ID:Lsj8W0ZY
アレフは無限だから
極限順序数とは違う
914 :
2016/12/09(金) 22:49:31.71 ID:Lsj8W0ZY
無限って何だよアレフは濃度だ
915 :
2016/12/10(土) 03:35:32.03 ID:5HdnYMvz
>>911
> >すべての自然数をちょうど一回使った列(並び順はいろいろ入れ替えられる)
>
> の濃度がΩ

濃度Ωって何?
有限でない濃度を表す数学記号としてはアレフ ? に
適当な順序数を添え字として付けたものを使うのが普通だが
916 :
2016/12/10(土) 09:07:09.39 ID:39sz4fIJ
順序数崩壊関数じゃΩを使う。Ωが必ずしも連続体濃度でなければならない理由もないが。
あと正確にはこの場合Ωが意味するのはアレフ1ではなくベート1。
連続体仮説を認めれば結果的にアレフ1=ベート1とはなる。
917 :
2016/12/11(日) 00:23:31.89 ID:Tc1s+pXK
「順序数を」崩壊させる関数なんだから、最小の非可算順序数と呼んで差し上げたい
まあ、同じだけど
918 :
2016/12/12(月) 21:59:38.76 ID:C3Ci9Uuv
BB(1000)を出力するチューリングマシンを特定するためには
今未解決になってる問題のほとんどを解決する必要がある?
919 :
2016/12/12(月) 22:04:50.39 ID:QGb70+CY
特定は無理でしょ
920 :
2016/12/12(月) 23:00:26.96 ID:C3Ci9Uuv
いますぐには無理でも100年後には特定できるかもしれないだろ。
921 :
2016/12/12(月) 23:23:02.42 ID:4HhfCHOl
可能性はあるがほとんど0に近いレベルでは
922 :
2016/12/12(月) 23:25:23.64 ID:4HhfCHOl
ラヨ数の値はどんなに時間があっても求めることができない。ビジービーバー関数の大きい値ははどうだか
923 :
2016/12/12(月) 23:56:12.27 ID:QGb70+CY
理論的には不可能じゃないけど、さすがにBB(1000)は無理じゃないかと思う
924 :
2016/12/13(火) 00:56:49.56 ID:sV/ioZsM
見積もりで
宇宙が保持し得る情報量は
10^90ビット

地球の情報容量は
10^56ビット

程度しかない。
925 :
2016/12/13(火) 03:30:04.87 ID:aUqnMl7U
計算量とか情報量の問題じゃないよね。計算で攻めたら停止性問題があるので無理なのは明らか。
証明できるかどうかの問題。そして、さすがにBB(1000)の証明は人間には無理じゃないかと。
926 :
2016/12/13(火) 04:49:32.35 ID:wXipH4nz
>>924
空間なら有り余ってるから
粒子一つ一つの位置情報を最小単位にすればもっと多くの情報を保持できる
927 :
2016/12/13(火) 07:41:04.52 ID:sV/ioZsM
>>926
それでも
10^170
には届かないか
928 :
2016/12/13(火) 07:47:53.05 ID:wXipH4nz
順列で (宇宙の広さ)P(粒子の総数) だからテトレーションクラスにはなるんじゃない?
929 :
2016/12/13(火) 11:31:03.31 ID:r2Um27P7
>925
停止性問題は任意のチューリングマシンが停止するかどうかを
計算で求めることができないということであって、有限個のチューリングマシンの停止性を計算
で求めることはできる。よって一応計算量と情報量の問題ではある。
930 :
2016/12/13(火) 15:22:33.54 ID:aUqnMl7U
>>929
その有限個のチューリングマシンである1000状態チューリングマシンの中に、
停止しないものが含まれれているので、停止ないものがすべて停止しないと
「証明」しないとビジービーバーは決定できない。
931 :
2016/12/13(火) 15:28:18.24 ID:aUqnMl7U
また、現実的に計算はできないようなチューリングマシンを計算が終了することを
証明することもあるので、結局は計算量の問題というよりも証明できるかどうかという
問題にしかならないな
932 :
2016/12/13(火) 15:47:13.46 ID:sV/ioZsM
いや、すべて停止しないのを停止しないと証明できても、停止するものが停止するまでのステップ数が記録できるスケールを超えてたら計算しようがないので、記憶容量とかも大切。
933 :
2016/12/13(火) 17:57:25.86 ID:wXipH4nz
記録座標に無理数使えば有限範囲で無限に記録できるんじゃない?
アナログ最強
934 :
2016/12/13(火) 18:27:15.64 ID:hQ5HQSpf
プランク長より精度必要じゃね
935 :
2016/12/13(火) 20:42:07.54 ID:aUqnMl7U
で、現実的には計算できないものを計算できると示すためには、結局計算できるという
「証明」をするしかないでしょ。グラハム数だって誰も計算できないけれど、
計算できることが証明できるから存在することがわかるわけで。
936 :
2016/12/13(火) 20:42:58.48 ID:r2Um27P7
>929
ちょっと論点がずれてたわ、すまん。

「計算で求めた」ように見せるだけなら、あらかじめ入力された有限個のそれぞれのチューリングマシン
に対し最初から停止するかしないかの答えを用意しておけばいいので、可能と言えば可能。
ただそれがなんであるかを知ることができるかどうかは別問題。
937 :
2016/12/13(火) 20:46:56.59 ID:aUqnMl7U
BB(1000)を人類が決定できるか?という問題は、まともにチューリングマシンを動かしても
現実的に無理だと言うのは誰でもわかるので、すべてのTMに対して計算できる、できないという
決定をできるか、という問題に帰着する。

あらゆるTMに対してその証明を与えるアルゴリズムはないけれど、BB(1000)までに対してそれができない
とは言えないため、可能性はゼロだとは言えないけれど、まあたぶん無理でしょ、という話。
938 :
2016/12/13(火) 20:59:13.27 ID:r2Um27P7
正確には計算できる、できないといより証明を計算で与えることができるか(形式的証明が存在するか)
という問題じゃと。また矛盾した体系からはいかなる命題も証明できてしまうため、求められているのは
無矛盾な論証体系からの証明だが、その体系が十分強ければ無矛盾性の真の証明はできなくなってしまう。

これもうわかんねぇな
939 :
2016/12/13(火) 20:59:57.91 ID:r2Um27P7
訂正
問題じゃと⇒問題かと
940 :
2016/12/14(水) 10:51:12.43 ID:kjfqcV7P
f(0,x) = BB(x)
f(y+1,0) = f(y,BB(1000))
f(y+1,x+1) = f(y,f(y+1,x))
941 :
2016/12/19(月) 19:28:23.17 ID:jdcWtsfk
計算不可能な関数に制限を設けて計算可能な領域に引きずり落とすというテクニックもある
942 :
2016/12/19(月) 21:21:57.10 ID:O7H4P6L5
超越整数みたいな感じ?
943 :
2016/12/21(水) 22:56:10.86 ID:2sVKlP3t
チューリング完全でない言語を対角化するなりある範囲のプログラムの定義不能性
を自動的に証明するなり、好きにすればよい
944 :
2016/12/21(水) 22:56:51.21 ID:2sVKlP3t
定義不能性じゃなくて計算不能性だった
945 :
2016/12/22(木) 01:41:39.84 ID:lWGOs9HI
チューリング完全でない言語の対角化、というのはローダー数だね
プログラムの計算不能性を自動的に証明する、というのは意味がわからないけど
946 :
2016/12/22(木) 22:46:46.40 ID:VhpxlRJR
BEAFのプログラムソースどこかにない?
パイロットとか副操縦士とかいわれてもよくわからん。
947 :
2016/12/23(金) 03:05:34.28 ID:mulCK6iM
いきなり定義読んでもわからないので、BEAF入門の記事あたりから
948 :
2016/12/23(金) 03:07:41.93 ID:zZj0EspU
あれからいろいろ考えてみたけど一つの形になったかも
うまくいってればε0あるかなぁ
最低でもω^ω^ω^ωはいってると思いたい

$a=0

def f(x)
return enum_for(:f,x) if !block_given?
x-=1
$a+=1
yield true
return if(x<0)

y=$a
while(y>0)
y-=1
f(x){ p $a}
end
end

def g(x,i)
return enum_for(:g,x,i) if !block_given?
$a+=1
x-=1
yield true
return if(x<0)

y=nil
if(i<=0)
then
y=f($a)
else
y=g($a,i-1)
end

begin
while(y.next)
g(x,i){p $a}
end
rescue
end
end

x=g(9,9){p $a}
949 :
2016/12/23(金) 11:55:50.43 ID:PMe1xwKL
ラヨ数のプログラムソースどっかにない?
950 :
2016/12/23(金) 11:59:33.48 ID:PMe1xwKL
BB(1000)はコラッツ予想も含んでないかな
951 :
2016/12/23(金) 15:52:35.79 ID:EfZ5efPZ
しくじってるっぽい
くそう
952 :
2016/12/24(土) 09:10:56.40 ID:jTilf4n2
微小数を追究して、その逆数から巨大数を得る事はできない?
巨大数に1を足せども引けども焼け石に水だが、微小数に1を加算/減算するととてつもない変化量になる
この差異をどうにかして手数の増加に繋げたい
953 :
2016/12/24(土) 16:49:49.97 ID:CZuBTdm1
加算減算じゃ微小数の操作には大きすぎて向かないだろうね
オコジョ数は近似値との差を微小数にしているんじゃなかったかな
954 :
2016/12/24(土) 20:00:28.92 ID:5H9Noa9p
微小数の追究も結局巨大数の追究と変わらないのだ。微小数をスタート地点とする
巨大数はまれだなんぎゃば
955 :
2016/12/25(日) 03:15:17.07 ID:otYoKtz5
拡張型Eシステムのあのクリスマス飾りみたいなハイペリオンの帯や、
10をベースとする関数なのに命名基準が100単位なのが気に入らなくて
自分好みにいろいろ叩いていったらなぜかBEAFが出来上がった
な、何を言っているのかわからねーと思うが(ry

BEAFってかなりスマートな構造してたんだね
今までなんとなく分かったつもりだったけれどしっかり理解できた
956 :
2016/12/26(月) 23:01:37.24 ID:yNmuMPcP
BIG FOOT が作者が思ってたほど大きくなかったようだが今のところ一番大きい
名前の付いた数であることは確かなようだ。
957 :
2016/12/28(水) 01:54:23.55 ID:CrGWohEQ
基礎論の強さっていうのは、
公理がより多いほど証明できる命題がより多くて
より少ない記号で特定の大きさ以上の巨大数の指命ができるけど、
うまくつくらないとやり過ぎるとラッセルのパラドクスみたいに自己矛盾が生じやすい、
っていう理解でおk?
958 :
2016/12/29(木) 20:37:31.96 ID:n+Oz33lt
Kurikaeshi no hyokihou wo tsukuttemita yo

a,k|0->f(k) = a
a,k|n+1->f(k) = f(( a,k|n->f(k) ))

Enzanshi Hyoki no rei: a{n+1}b = a,k|b->a{n}k

Phi kansu no rei:
phi(0,b) = omega^b
phi(a+1,0) = phi(
phi(a+1,b+1) = (phi(a+1,b)+1),k|omega->phi(a,k)

epsilon_1 = phi(1,1) = omega^omega^...^(epsilon_0+1)
959 :
2017/01/01(日) 22:37:32.79 ID:P+h0bxp+
過去ログを見てみたが為になるな
熱い議論が展開してて面白過ぎる
960 :
2017/01/04(水) 10:20:24.97 ID:0dVQs3l5
スレ立て出来ぬ。すまん。誰かお願い
スレ立て別に950じゃないならおk
961 :
2017/01/04(水) 14:41:17.11 ID:L+joR6Xn
次スレなんてまだまだ立てる必要ないよ。
962 :
2017/01/04(水) 18:43:28.38 ID:80McB6Uf
このスレも巨大関数並みのレス増加率なら面白いのに
963 :
2017/01/04(水) 19:03:32.37 ID:RGzW2Ekr
宇宙全部がサーバーでも追いつかんだろう
964 :
2017/01/04(水) 19:25:51.07 ID:PR8N4svl
勢いが3↑↑↑↑3とかか
いいなそれ
965 :
2017/01/04(水) 22:51:04.28 ID:02ryg0U5
サーバーの増設率がBB(BB(1000))/sec
966 :
2017/01/04(水) 23:15:09.36 ID:W7MrQHxn
FOOT

oodles
* 抽象的な何か
* 広義の集合みたいなの
* oodlesの演算は∈,=, それと論理演算(∃,∧,?とか)
* 式の真理値をタルスキの定義によって定める:
* * "x∈y": xがyの要素なら真
* * "x=y": xとyが同じoodleなら真
* * "φ∧ψ": "φ"と"ψ"が真なら真
* * "?φ": "φ"が真でないなら真
* * "∃x:φ(x)": "φ(x)"が真になるようなoodleがあれば真

oodle ranks
* わからん
967 :
2017/01/05(木) 00:48:18.80 ID:4vtLZkJz
oodle ranks
フォン・ノイマン宇宙の階層を拡張したやつ。定義はフォン・ノイマン宇宙と「だいたい」同じ
968 :
2017/01/05(木) 18:03:52.06 ID:LbTcPrue
増加率の曲線自体を幾何学の観点からあれこれしてスーパー増加率にする
969 :
2017/01/12(木) 13:25:11.62 ID:IluHtFDi
バシク行列は強配列表記よりも右側でいいかな、解析結果見る限り>例の表
強配列表記は2階算術の強さは超えてないようだ。解析結果が正しければ。
970 :
2017/01/12(木) 13:32:06.20 ID:IluHtFDi
と思ったらバシク行列微妙に定義変更されて弱体化された?
971 :
2017/01/12(木) 17:13:27.68 ID:/wyz5h+9
あの表の小一次数列数の位置付けが若干違う。ハーディー階層でω^ωと評価されるのでFGHだとωになる。
972 :
2017/01/14(土) 04:04:59.72 ID:if507sG+
バシク行列は、結局まだ計算終了の証明が完成していないので、位置付けはかなり不安定
973 :
2017/01/14(土) 11:58:20.25 ID:8SB0dB56
計算終了の証明以前に定義があやふやなのだわ。
直接順序数に対応させる力技ですくなくともBEAFなみの力を持ってるのは確かなようだ。
BEAFもテトレーション配列より先はあやふやだけど。
974 :
2017/01/14(土) 14:23:25.25 ID:if507sG+
定義はプログラムがあるんだからはっきりしてるでしょ
975 :
2017/01/14(土) 14:26:20.38 ID:if507sG+
でも、修正したバージョンの定義がどうなっているのかがあやふやか
976 :
2017/01/17(火) 20:47:38.83 ID:5OvsB2ra
計算不能関数ってRCA_0では定義できないの?
977 :
2017/01/17(火) 22:08:55.78 ID:bBhLDo2n
計算不能関数なのでその関数の全域性をRCA_0では証明できない
978 :
2017/01/17(火) 22:11:03.76 ID:bBhLDo2n
ビッグフットを超えるリトルビッゲドンが出た
979 :
2017/01/17(火) 22:38:33.30 ID:5OvsB2ra
>>977
thx 定義自体はできるのもあるってことです?

最近順序数でかいの多すぎてよくわかんないです
バッハマンハワードまでしか理解できてないです
RathjenのΨ関数とかマーロ基数がわかんないのでわかる方解説お願いできませんか
980 :
2017/01/17(火) 23:27:42.59 ID:bBhLDo2n
全域性が証明されていないということは、全域関数が定義されたことにはならない
981 :
2017/01/17(火) 23:28:49.64 ID:bBhLDo2n
RCA_0 という理論体系を考えている以上は、証明を離れた定義を与えるのは無理
982 :
2017/01/17(火) 23:59:12.53 ID:LlH1bWiB
TaranovskyのCを超えたところから順序数による評価そのものに限界を感じ始める。
正確には評価というより評価の表現
983 :
2017/01/18(水) 23:06:04.75 ID:AW3VJECs
RCA_0やらは証明するための前提条件でなにかを定義するものではないのでは。
そして定義できることと記述できることと証明できることはそれぞれ別だろう。
984 :
2017/01/19(木) 02:38:26.58 ID:V/9wQX49
リトルビッゲドン分かる人解説お願い
985 :
2017/01/19(木) 03:28:03.75 ID:V/9wQX49
>>983
全域関数f(n)を定義するためには、f(n)がすべての自然数で定義されている必要がある。
でも、全域性が証明できないということは、定義されているかどうかわからないという
ことだから、定義したことにはならない。
986 :
2017/01/19(木) 15:28:40.06 ID:h1QSJgrO
定義するとはどういうことかを定義する必要があるが、循環論法になって議論にならないな。
定義されているかどうか分からないからといって定義できてないというわけではなく、
定義できているかどうかということを確かめようがない、ということだろう。

というか、定義するのは論理式であってRCA_0ではない。
987 :
2017/01/19(木) 15:39:53.90 ID:h1QSJgrO
別に985が間違ってるということではなく、「定義」という言葉の定義が互いにずれているようだ。
「「定義」という言葉の定義」という言葉の定義はずれていないことを望む。

どうでもいいけどこの議論は
X=「X」という言葉の定義
で対角化することができる。
988 :
2017/01/19(木) 16:16:07.49 ID:E4EyRNy1
「全域関数か部分関数かわからないビジービーバー関数のようなもの」であれば定義できる可能性があるけど、
そもそも証明しないんならRCA_0もZ_2も同じなので、わざわざRCA_0を指定する意味がない。
989 :
2017/01/19(木) 17:00:12.29 ID:OtJDogH/
最大の双子素数の大きい方は、自然数nである
という定義が、そもそも意味を持つかということで、
存在しないかもしれないのに定義できたというかどうかは、
まさに定義の定義だけれと、だとしたら常識的には
定義できてないと考えるのが自然だろう。
990 :
2017/01/20(金) 14:20:34.98 ID:Edrcljmk
存在するかどうかというのは、「存在する」が真であるかどうかという問題であって、
証明可能性とは別の問題だと思います。。
この分野では従来の直感に反することがよく起こってきたから、常識で判断するのはちょっと危険かと。
この場合は常識云々というよりは存在論や認識の話になるのやもしらんが。

RCA_0では証明できなくてZ_2では証明できる論理式があるかもしれないので、
指定する意味はある(かもしれない)。
RCA_0の時点で矛盾していればたしかに意味はない。
そして完全に純粋に言語だけで無矛盾性を証明する(ことを認識する)ことはできない、
つまり理性のみで完全に正しいと言える証明は存在しない。
そういう意味で理性のみを便りにする立場から、証明できなければ定義できたことにはならない
と主張するのであれば、何物も定義することができないということになる。

直感って大事ね。
991 :
2017/01/20(金) 15:39:48.39 ID:xpTs7JyG
> RCA_0では証明できなくてZ_2では証明できる論理式があるかもしれないので、
> 指定する意味はある(かもしれない)。

そもそもの質問は「計算不能関数をRCA_0で定義できるかどうか」なので、
もしそれをあらわす論理式ができたとして、その論理式が全域関数であることを
証明する、しないを問題にするのであればRCA_0を指定する意味ある。

定義する段階では証明する、しないは関係ない、という立場であれば、
Z_2で定義できていればRCA_0でも定義できているということになるから、
はじめからZ_2で定義できるかどうかを聞けばいい。
992 :
2017/01/20(金) 15:55:59.13 ID:xpTs7JyG
内包公理ってのは「ある式を満たす集合が存在するよ」ってことだから、
その式を満たす集合が存在するかどうかは関係なく定義できる、
という立場だったら、RCA_0もZ_2も同じだよね。
993 :
2017/01/20(金) 15:59:18.13 ID:xpTs7JyG
同じというのは「定義できるかどうか」という問いに対してね。
証明できるかどうかとなると、もちろん違う。
994 :
2017/01/20(金) 23:39:48.10 ID:cKrQZH+b
995 :
2017/01/22(日) 23:12:06.89 ID:MZPwabB4
先に双子素数のたとえに触れるべきでしたが、

最大の双子素数の大きい方aが存在するということが今は証明できてないから
aを定義できていない。
しかし後で証明できたとしたらaを定義できたことになる。
となったら矛盾になりませんか。

これは言葉のあやの問題だと思うので、これで貴方は間違っていると主張するわけではありません。

私は「定義する」という言葉を普遍的な意味で使っていますが、時間を踏まえて
言葉を使う線形論理学的な(?)考え方では確かに矛盾しません。

お考えを伺いたいです。
RCA_0を指定する是非については993で腑に落ちました。
996 :
2017/01/22(日) 23:41:56.39 ID:MZPwabB4
Z_2で定義できていれば〜のくだりを読み間違えていた。
Z_2のモデルが存在すればそれはRCA_0のモデルでもあるから、最初から
Z_2のモデルの中にそういうのがあるかを問えばいい、ということか。

確かにその通りだけど、私は無矛盾性の強さと証明不能性からそこまで自信を
持てないです・・・
997 :
2017/01/23(月) 01:58:52.37 ID:TzBT9HVn
>>995
逆に、後で存在しないことが証明されたら定義できてなかったことになるから矛盾するでしょ。
そんな不安定な状態のものをはっきりと定義できたとするのは常識的には不自然だけど、
常識とは無関係にそういうものだと定義という言葉を定義するのであれば、その定義であるという
断りを入れて使うことまでは否定しない。
998 :
2017/01/23(月) 02:10:40.99 ID:TzBT9HVn
最大の素数をaとする、という定義は定義になってないよね。
それは、最大の素数が存在しないことが証明されているから。
一方、最大の双子素数の大きい方をaとすえう、という定義が今は許されるとして、
それが存在しないことが将来的に示されたらその定義は無効になるので、
存在するかどうか分からないものを定義できるという考え方が、
時間によって定義の意味が変わるという意味では普遍的でない。
999 :
2017/01/23(月) 02:21:42.89 ID:TzBT9HVn
少なくとも、定義できると言い切ってしまうと、存在するという意味だと誤解されるので、
存在するとすればそのように定義する、
定義する。ただし、存在しないかもしれない。
定義できるかどうかは分からない
などの、誤解のない表現を使うことが重要。

まとめると、証明できないということが重要なのであって、
定義できたかどうかはどうでもいい。
1000 :
2017/01/23(月) 02:22:32.18 ID:TzBT9HVn
1001 :
1001
Over 1000 Thread
このスレッドは1000を超えました。
もう書けないので、新しいスレッドを立ててくださいです。。。
life time: 427日 0時間 17分 8秒

巨大数探索スレッド過去ログ